Nothing Special   »   [go: up one dir, main page]

General Surgery Personal Notes by DR Shariful Halim

Download as pdf or txt
Download as pdf or txt
You are on page 1of 189
At a glance
Powered by AI
The document discusses different types of ulcers including tubercular, pyogenic and malignant ulcers. It also covers bed sores and head injuries.

The clinical characteristics of a tubercular ulcer include sites in the neck, axilla or groin with an undermined edge and serous discharge. The floor has pale/apple jelly like granulation and the base is mildly indurated with enlarged matted draining lymph nodes.

A tubercular ulcer is treated with category I anti-tubercular chemotherapy (2HRZE+4HR) along with regular dressing.

Sharifs personal notes

GENERAL SURGERY

Ulcer:

1. What is ulcer? Classify ulcers.

An ulcer is a discontinuity /solution/ breach of continuity of an epithelial surface which occurs due to
sloughing of inflammatory necrotic tissue.

Classification:

1. Non-specific ulcer: No specific infection.


Ischemic ulcer: Foot ulcer in PVD ie Buerger s disease.
Neuropathic ulcer: Foot ulcer in DM.
Decubitus ulcer: Pressure sore/ Bedsores.
Traumatic ulcer.
Peptic ulcer.
Apthous ulcer.
2. Specific ulcer: Ulcer due to specific infection
Tubercular ulcer.
Syphilitic ulcer.
Herpes labialis.
3. Malignant ulcer:
Basal cell carcinoma.
Squamous cell carcinoma.
Carcinoma of the GI tract ie adenocarcinoma.
`
2. What are the clinical characteristics of a tubercular ulcer? (11).

Ans:

a. Site: Neck, axilla, groin.


b. Edge: Undermined.
c. Discharge: Serous.
d. Floor: Pale/ apple gelly like granulation.
e. Base: Mildly indurated.
f. Lymphadenopathy: Enlarged matted draining Ln.
3. How will you treat a tubercular ulcer? (12).

Ans:

a. Anti-tubercular chemotherapy: Category I (2HRZE+ 4HR).


b. Regular dressing.

4. Write short note on Bed sore/Decubitus ulcer/Pressure sore.

Ans:

1 SHARIFUL HALIM, K66, DMC


Sharifs personal notes

Definition: Gangrene of the skin caused by local pressure.

Common site:

a. Heel.
b. Lateral malleolus.
c. Greater trochanter.
d. Ischium.
e. Sacrum.
f. Occiput.

Stages:

a. I: Non-blanchable erythema.
b. II: Partial thickness ulcer.
c. III: Full thickness ulcer without involving the fascial plane and deeper structures.
d. IV: ulcer involving fascial plane and deep structures.

Cause:

Prolonged bed ridden patients such as:


Unconscious.
Quadriplegic.
Paraplegic.
After major surgical procedure.

Risk factors:

Anemia.
Malnutrition.
DM.
Old age.
Chronic diseases.
Smoking.
Immunosuppression due to any cause.

Treatment:

Regular dressing.
Keeping the ulcer area bare and dry.
If the ulcer area granulates well then skin grafting or flap coverage.
Broad spectrum antibiotics.
Optimization of general wellbeing:
Correction of anemia.
Correction of malnutrition.
Control of DM.
Treatment of other comorbid conditions.

2 SHARIFUL HALIM, K66, DMC


Sharifs personal notes

Prevention of bed sore:

Regular 2 hourly change of posture.


Optimization of general wellbeing.
Use of pneumatic/ pressure relieving mattresses and bed.
Regular checking of common areas of bed sore.
5. Describe different stages of a non-specific ulcer.

Ans:

Stage of extension: Floor covered with slough and base is indurated.


Stage of transition: Floor cleared and induration diminished.
Stage of repair: Transformation of granulation tissue into fibrous tissue leading
to scar formation.
6. Describe naked eye appearance of pyogenic, tubercular, syphilitic & malignant ulcers.

Ans:

Venous ulcer:

Site: Medial aspect of lower leg, commonly above the medial malleolus.
Edge: Sloping.
Floor: Pale granulation tissue.
Base: Slightly indurated.
Discharge: Seropurulent.
Surrounding region: blackening, edema, hair loss, shininess, loss of skin texture.

Pyogenic:

Edge: sloping.
Floor: Necrotic debris.
Discharge: Purulent.
Base: Slightly indurated but not fixed.

Syphilitic:

Edge: Punched out.


Floor: Wash leather slough.
Base: Indurated.

Malignant:

Squamous cell carcinoma:


Edge: Raised and Everted.
Floor: Greyish white slough, may bleed on touch.
Discharge: Serosanguinous or blood stained.
Base: Indurated, may be fixed with underlying structure.
Draining LN: May be enlarged and hard.
Basal cell carcinoma:
Site: Face above the line joining the angle of mouth to ear lobule.

3 SHARIFUL HALIM, K66, DMC


Sharifs personal notes

Edge: Raised and rolled.


Floor: deep.
Discharge: Serosanguinous/ blood stained.
Base: Indurated, may be fixed with underlying structures.
Draining LN: May be enlarged.

Q. Write down different types of ulcer edges with their cause.

Ans:

Sloping: Healing and venous ulcer.


Punched out: Ischemic, peptic, syphilitic ulcer neuropathic.
Undermined: Tubercular, pressure sore.
Rolled: Basal cell carcinoma.
Everted: Squamous cell carcinoma.

SINUS

Q. What is sinus? Classify sinus.

Ans:

Sinus:

It is a blind tract connecting an epithelial surface with a cavity lined by granulation tissue.

Classification:

A. Anatomical sinus: Cavernous sinus, PNS.


B. Pathological sinus:
Congenital: Pre-auricular, post-auricular, umbilical, urachal, sacro-coccygeal.
aAcquired: Pilonidal, hidradenitis supporativa, post- surgical, tuberculosis,
osteomyelitis.

FISTULA

Q. What is fistula? Classify it.

Ans:

Fistula:

It is an abnormal tract joining 2 epithelial or endothelial lined cavities/surfaces lined by healthy or


unhealthy granulation tissue or epithelial tissue.

Classification: According to etiology and onset:

A. Congenital:
Tracheoesophageal fistula.
B. Acquired:

4 SHARIFUL HALIM, K66, DMC


Sharifs personal notes

Inflammatory: Perianal fistula, tubercular, Crohn s disease.


Neoplastic: Rectovesical/rectovaginal fistula.
Traumatic: VVF.
Iatrogenic: Arteriovenous fistula, biliary fistula, pancreatic fistula, gastro-jejuno-
colic fistula, gastro-jejunostomy.

Q. What are the causes of persistence of a fistula?

Ans:

A. Local causes:
Foreign body.
Continuous discharge.
Less blood supply.
Chronic infection.
Epithelialization.
Malignant change.
B. Systemic causes:
Anemia.
Malnutrition.
Vitamin and mineral deficiency.
DM.
Immunosuppression.
Smoking.
Chronic diseases of kidney, liver.

Cyst

1. Define & classify cyst.

Ans:

Cyst: It is a fluid filled cavity bounded by a wall.

Classification of cyst:

A. Congenital:
- Dermoid cyst.
- Cystic hygroma.
- Branchial cyst.
- Thyroglossal cyst.
- Polycystic kidney.
B. Acquired:
- Retention: Sebaceous cyst, mucus retention cyst of mouth.
- Infective: Hydatid cyst.
- Neoplastic: Ovarian cyst.

5 SHARIFUL HALIM, K66, DMC


Sharifs personal notes

2. Compare between cyst and abscess.

Ans:

Points Cyst Abscess


Definition It is a fluid filled cavity bounded Localized collection of pus lined
by a wall. by pyogenic membrane.
Pain Absent Present
Fever Absent Present
Tenderness Absent Present
Pus pointing Absent Present
Consistency Cystic Soft to firm.
Transillumination Positive Negative
Lining Epithelium/endothelium Pyogenic membrane.

3. How will you differentiate between sebaceous cyst & dermoid cyst?

Ans:

Points Sebaceous cyst Dermoid cyst


Common Site Scalp, face, scrotum, vulva. External angular dermoid, post-
auricular.
Punctum Present Absent
Fixity to overlying skin Fixed Not fixed
Moulding Positive Negative
Bony indentation Negative Positive

4. Write a SN on sebaceous cyst.

Ans:

Definition: It is a cystic swelling that develops due to obstruction of the duct of sebaceous gland.

Sites: Scalp, face, scrotum, vulva.

Clinical features:

Punctum: Bluish dot in the middle of the swelling.


Fixed with overlying skin.
Consistency: Cystic.
Transillumination: Negative.
Moulding test: Positive.
Fluctuation test: Positive.

Treatment:

Excision of the cyst under L/A or G/A.

Complication:

6 SHARIFUL HALIM, K66, DMC


Sharifs personal notes

Infection.
Abscess formation.
Rupture.
Sinus formation.
Calcification.

TUMOR

Q. How will you differentiate benign and malignant tumor clinically?

Point Benign Malignant


History Long Short
Enlargement Slow Rapid
Weight loss, anemia, Absent Present
anorexia
Surface Smooth Irregular.
Consistency Soft to firm Hard.
Fixity Not fixed May be fixed with underlying
structures of overlying skin
Margin Well defined Ill defined
Draining LN Not enlarged May be enlarged
Features of metastasis Absent May be present.

Q. Write down the different modalities of treatment of a malignant tumor?

Ans:

Surgery.
Chemotherapy.
Radiotherapy.
Immunotherapy.
Hormone therapy.
Gene therapy.

Q. Mention the indications of radiotherapy.

Ans:

Head neck tumor.


Esophageal carcinoma.
Basal cell carcinoma.
Carcinoma cervix.
Breast cancer.

Q. Mention some Radioresistant tumor.

Ans:

Glioma.
Melanoma.

7 SHARIFUL HALIM, K66, DMC


Sharifs personal notes

Marjolins ulcer.
SCC.
Sarcoma.

Q. Write down the role of chemotherapy.

Ans:

Selective toxicity/curative.
Neoadjuvant.
Adjuvant.
Metastasis.
Palliative therapy.

Q. What are the routes of spread of cancer?

Ans:

Local: Invasion.
Systemic: Lymphatic, blood, nerve sheath.

Wound & injury

1. Define wound.

Ans:

It is a break in the integrity of the skin or tissue often which is associated with disruption of function.

2. Classify wound.

Ans:

Classification of surgical wound according to contamination:

Clean wound:
Incised, clean, healthy wound without any tissue loss.
Usually primary suturing done, healing by primary intention.
Example: Surgical incision wounds ie in hernirrhaphy, surgeries in brain, joint,
heart, thyroid and breast.
Infective rate <2%.
Clean contaminated wound:
Incision is clean but the operative structure may contain micro-organisms within
itself.
Wound is initially clean but contaminated later.
Examples: Appendicectomy, cholecystectomy, bowel surgeries- ie resection
anastomosis of intestine, partial gastrectomy & Gastro-Jejunostomy,
Infective rate: 10%.
Prophylactic antibiotic is needed.

8 SHARIFUL HALIM, K66, DMC


Sharifs personal notes

Contaminated/untidy wound:
Open fresh accidental wounds i.e. lacerated wound in RTA.
Large gut surgery where bowel is opened.
Wound infection.
Operations in natural orifices: Per-oral, Per-anal, Per-vaginal.
Antibiotic is given according to C/S; if C/S not done then broad spectrum antibiotic
with anaerobic coverage.
Dirty infected wounds: Huge load of micro-organisms.
Abscess drainage.
Pyocele.
Empyema GB.
Fecal peritonitis with frank pus in the peritoneal cavity.

3. What are the differences between tidy/clean & untidy/contaminated wound?

Ans:

Points Tidy/clean wound Untidy/contaminated wound


Type Incised Crushed or avulsed.
Presence of infective No Yes.
organism
Tissue type Healthy Often devitalized.
Tissue loss Minimum More
Infective rate <10% About 30%
Example Herniotomy incision Open fresh accidental wounds.

4. How will you treat an untidy wound of hand?

Ans:


Thorough cleaning/toileting of the wound.

If it is a lacerated wound: Primary suturing is done after excision if possible; if not possible
delayed primary closure is done.
If it is a crushed or devitalized wound: Wound debridement is done & wound is kept open
for 2-6 days when edema is allowed to subside. Then delayed primary suturing is done.
Repair of structures: i.e. vessels, nerves.
Skin cover is given if required: esp. in delayed primary closure.
Analgesics.
Broad spectrum antibiotics.
Administration of fluids & electrolytes as needed.
Tetanus toxoid: .5ml IM to deltoid or ATG- Anti-Tetanus-Globulin injection is given.
5. Write the management of lacerated wound in the leg following RTA.

Ans:

Primary survey & resuscitation: Maintenance of airway, breathing & circulation.


Thorough cleaning/toileting of the wound.
Repair of structures: ie vessels, nerves.

9 SHARIFUL HALIM, K66, DMC


Sharifs personal notes


Lacerated wound: Excision & primary suturing if possible; delayed primary closure if not
possible.
Crushed wound/Devitalized wound: Wound debridement is done & wound is kept open for
2-6 days when edema is allowed to subside. Then delayed primary suturing is done.
Skin cover is given if required esp. in delayed primary closure.
Analgesics.
Broad spectrum antibiotics.
Administration of fluids & electrolytes as needed.
Tetanus toxoid: .5ml IM to deltoid or ATG- Anti-Tetanus-Globulin injection is given.
6. What are the methods of wound closure?/ Classify & compare different types of healing.

Method of wound Characteristics Example/ Situation where done.


closure/healing
Primary closure/Healing Wound edges Surgical incisions.
by primary intention. opposed. Linear incised
Scar: Minimal, linear, wounds.
smooth.
Secondary Wound let OPEN. Major trauma.
closure/Healing by Heals by granulation, Burn.
secondary intention. contraction & Wound sepsis.
epithelialization.
POOR SCAR.
Delayed primary Wound initially left Primary
closure/Healing by open. contaminated
tertiary intention. Edges later apposed wounds with less
when healing tissue loss.
conditions favorable. Wound infection.
Scar is good. Ulcer.

7. Write down the stages/phases of wound healing.

Ans:

1. The inflammatory stage/phase:

Starts immediately after wound lasts for 2-3 days ( B&L).

The wound is filled with coagulated blood and cell debris.

Secretion of different chemical mediators by platelet & injured endothelium (PDGF,


TGF-Beta, IL-1, TNF-Alpha).

Primary vasoconstriction followed by sustained vasodilation & increased blood


flow to the area.

At first the neutrophil comes to the site followed by macrophage on second day.

Macrophages clean up the cell debris and foreign material.

2. The proliferative stage/phase:

10 SHARIFUL HALIM, K66, DMC


Sharifs personal notes

rd rd
From 3 day to 3 week.

Macrophages stimulate the formation of granulation tissue.

It contains fibroblasts and angioblasts.

Fibroblasts start to form collagen type III & GAGs.

Angioblasts starts to form new blood vessels.

Epidermal cells form a bridge that seals the defect.

3. The remodeling/maturation stage/phase:


rd
From 3 weeks onwards.
Replacement of collagen type III by type I until a ratio of 4:1 is achieved.
Realignment of collagen along the line of tension.
Decreased wound vascularity.
Wound contraction due to fibroblast & myofibroblast activity.

Q. What are the components of granulation tissue?

Ans:

Fibroblasts.
Collagen fibers.
GAGs.
Angioblasts.
Newly formed blood vessels.
8. What are the differences between primary & secondary healing?

Ans:

Trait Primary Secondary


union/intention/ union/intention/healing/closure
healing/closure

Margin of wound Apposed together A large gap between the margin

Tissue loss Less More


Inflammation, exudation and tissue Less More
damage.

Granulation tissue Minimum Larger amount of granulation


tissue

Healing and scar tissue Quickly, small, linear & Takes time, large ugly scar
smooth scar

Infection and wound contraction No May be present

11 SHARIFUL HALIM, K66, DMC


Sharifs personal notes

9. Write a short note on delayed primary healing/tertiary healing/delayed primary


closure/tertiary closure.

Ans:

Definition:
It is a type of closure/healing where wound is left open initially but edges are opposed later
when healing conditions are favorable.
Principle:
Devitalized tissue, edema & infection of wound are to prime factor responsible for delaying
wound healing.
In delayed primary closure:
Devitalized tissues are debrided.
Time is allowed for the edema to subside.
Infections are controlled.
Wound is closed with sutures or covered with skin graft.
Scar formation:
Better than secondary closure.
Indication:
Primary contaminated wounds with less tissue loss, mixed tissue wounds.
10. Enumerate the factors that delays wound healing (Almost same answer for factor of healing of
an ulcer, fistula, sinus, gangrene, RF for wound dehiscence, burst abdomen, prevention ugly
scar).

Ans:

Local factors:

Poor apposition.
Infection.
Presence of Devitalized tissue.
Presence of Foreign body.
Poor arterial supply.
Venous stasis.
Increased tissue tension.

Systemic factor:

Malnutrition.
Anemia.
Vitamin deficiency: Vit-A, C.
Mineral deficiency: Zn, Cu, Mn.
DM.
Medications: Steroids.
Immunosuppression: Due to cytotoxic drugs, AIDS.
Smoking.
Jaundice: Less ability of liver to synthesize protein.
Uremia.
11. How do you prevent an ugly scar? (Correlate with factors responsible for delayed healing).

12 SHARIFUL HALIM, K66, DMC


Sharifs personal notes

Ans:

Prevention of an ugly scar:


Adequate apposition of wound margin by:
Primary closure.
Delayed primary closure.
Control of infection.
Removal of devitalized tissue or foreign body.
Adequate treatment of:
Malnutrition.
Vitamins & mineral deficiencies.
DM.
Anemia, jaundice, uremia.
Avoid: Smoking, Steroids.
12. What are the differences between hypertrophic scar & keloid?

Ans:

Points Hypertrophic scar Keloid


Sex Equal in both More in female.
Race No racial relation Common in black.
Growth Limited to scar margin Extend beyond scar margin.
Site of predilection Anywhere in the body Chest, upper arm, ear, lower
neck, sternum, face, shoulder.
Relation with wound Only larger infected wounds No relation with wound size
Treatment Good response to steroid Poor response
Recurrence Uncommon Very common.

13. What is wound disruption/dehiscence? What are the factors responsible for wound disruption?

Ans:

Definition:

It is a complication of wound where wound suddenly gives away with pain & serosanguinous
discharge.

Example:

Disruption of external wound: Closure of skin, mucosa, cornea.


Disruption of internal wound: Closure of fascia, tendon, ligaments.

Factors responsible for wound disruption:

Local:
Poor closure technique.
Sudden strains: Coughing, vomiting, BPH, constipation.
Infection.
Increased tension deep to sutures.

13 SHARIFUL HALIM, K66, DMC


Sharifs personal notes

Wound necrosis.
Wound hematoma.
FB.
Devitalized tissue.
Systemic:
Malnutrition.
Anemia.
Vitamins & mineral deficiency.
DM.
Immunosuppression.
Steroids.
Elderly.
Smoking.
Liver failure, Renal failure.
14. How will you manage a case of burst abdomen?

Ans:

Immediate management:

1. ABC resuscitation.

2. Clinical assessment.

3. Inspection of wound.

4. IV access & administration of IV fluids and basic blood tests.

5. Nil orally/NPO.

6. Reassurance of patient.

7. If Bowel is exposed: saline soaked sterile towel is used to cover them.

Further definitive management:

Decision for re-intervention: depends on exposed structures and cause for wound
disruption.

Exposed bowel:

o Surgical intervention and repair.

o Using tension sutures.

Gross infection wound/abscess

o May elect to leave wound exposed.

o Pack and dress to allow healing by secondary intention or delayed primary


closure.

14 SHARIFUL HALIM, K66, DMC


Sharifs personal notes

Haematoma:

o Evacuated

o Stoppage of bleeding direct/pack wound.

15. Enumerate different complications that may arise in a surgical wound.

Ans:

Local:

Infection.
Dehiscence.
Compartment syndrome.
Gangrene, Gas gangrene.
Tetanus.
Hypertrophic scar.
Keloids.
Contracture.

Systemic:

Shock.
Fever.
Bacteremia.
Septicemia.
Renal failure.
Toxemia.

DIC.
16. How will you diagnose & manage wound infection?

Ans:

Diagnosis:

Clinical features:
Symptoms: Pain at wound site, purulent or serous foul smelling discharge,
delayed wound healing, fever, malaise.
Signs: Discoloration of granulation tissue, red, swollen & edematous wound,
tender on palpation, tachycardia.
Investigations:
CBC with ESR.
CRP.
Pus: For Gram staining & culture sensitivity.
Wound swab: For Gram staining & C/S.
Tissue biopsy & wound culture.

Management:

15 SHARIFUL HALIM, K66, DMC


Sharifs personal notes

Wound care:

o Cleansing: With large quantities of soap and boiled water for 10 minutes, and
then irrigate the wound with normal saline.

o Debridement: This is done to clean and remove objects, dirt, or dead skin and
tissues from the wound area. Caregivers may also drain the wound to clean out
pus.

o Wound cover: With damp saline disinfected or clean gauze and cover the packed
wound with dry dressing. Change the packing and dressing at least daily.

Broad spectrum antibiotics with anaerobic coverage.

Analgesics.

Other treatment: Controlling or treating the medical condition that causes poor wound
healing helps treat wound infection such as diabetes or high blood pressure. Improving
nutrition and health.

Negative pressure therapy: This is also called vacuum-assisted closure (VAC). VAC helps to
increase blood flow and decrease the number of bacteria in the wound.

17. CRUSH syndrome.

Ans:

It is serious surgical emergency occurring as a result of crushing of muscles causing extravasation of


blood & release of myoglobins into the circulation leading to ATN & ARF.

Causes:

Earthquakes.
Mining & industrial accidents.
Air crash.
RTA.
Tourniquet.

Pathogenesis:

Increased pressure in the muscle.


Muscle ischemia.
Release of myoglobin.
ATN & ARF.
Infection, gangrene formation, septicemia.

Investigations: Std. investigations.

Treatment:

16 SHARIFUL HALIM, K66, DMC


Sharifs personal notes

ABC resuscitation.
Initial aggressive volume load using saline about 1-1.5 liters/hour.
Mannitol: To improve urine output.
Relieving tension in the muscle compartment by multiple parallel deep incisions
(fasciotomy).
Alkalanization of urine by giving sodium bicarbonate: Promotes the excretion of acid
hematin.
Hemodialysis: If patient condition deteriorates.
Other measures:
Catheterization.
Oxygen therapy.
Antibiotics.
Blood transfusion.
Correction of severe hyperkalemia.

ATLS

Q. What is triage?
Ans:
Screening and classification of sick wounded people in a mass casualty to determine the priority
of their need and proper place of their treatment for efficient use of medical resource.
Phase:
Pre-hospital triage.
At the scene of trauma.
On arrival of the receiving hospital.

Type of situation:

Multiple casualties: the number and severity of the injury don t exceed the capability and
facility.
Mass casualty: See below.

Triage procedure:

T. Sieve: Separate the dead and the walking from the injured.
T. sort: Sort and classify the injured.

18. What are the mass casualties? How do you determine the priority?

Ans:

Mass casualty:

It is a triage situation where-


Number & severity of the casualties exceed the capability & facilities available to staff.
These with greatest chance of survival are prioritized.

17 SHARIFUL HALIM, K66, DMC


Sharifs personal notes

Determination of priority:

It is done according from more priority to less like.

Most priority: Pt. with greatest chance of survival.


Then, Pt. with life threatening injuries.
Then, these with poly-trauma.

A common color coding system followed to determine priority from more to less is given below:

Red: Immediate treatment. Can t wait.


Yellow: Urgent tt. Can wait ie 30 minutes in most cases.
Green: Delayed tt. Less serious injury. Not endangered by delay.
Blue: Expectant management. Survival not likely.

Black: Deceased.
19. What is ATLS?

Ans:

The elaboration of ATLS is Advanced Trauma Life Support.

It is a standardized protocol of immediate management of trauma victims formulated by Committee


of Trauma of the American College of Surgeons.

20. Name the components of ATLS.

Ans:

ATLS has 3 main components:

1. Primary survey & simultaneous resuscitation: Identify what is killing the patient & treat it.
2. Secondary survey: Identify all other injuries.
3. Definitive care: Development of a definitive management plan.
21. Write steps of primary survey & resuscitation of ATLS.

Ans:

The mnemonic for steps of primary survey & resuscitation is ABCDE.

A= Airway maintenance & cervical spine protection.

B= Breathing & ventilation.

C= Circulation & control of hemorrhage.

D= Disability: Neurological status.

E= Exposure: Assessment for other injuries & environmental control.

22. Name some complications that can be fatal following a trauma.

18 SHARIFUL HALIM, K66, DMC


Sharifs personal notes

Ans:

Fatal complications following a trauma:

Airway obstruction.
Excessive bleeding & shock.
Pericardial temponade.
Tension pneumothorax.
Head injury.
Massive Hemothorax.
Rupture of liver, spleen.
Pulmonary contusion.
ATN.

Suture material:

Q. Write down the criteria of ideal suture material?


Ans:
Cheap.
Available.
Adequate tensile strength.
Minimum tissue reaction.
Non-allergic.
Non-infective.
Feasible to use.
Non- carcinogenic.
Easy to sterilize.
Q. Classify suture material?
Ans:

According to source:

Natural: Catgut, silk.


Synthetic: Vicryl, prolone, Dexon.

According to absorbability:

Absorbable: Catgut, Vicryl.


Non-absorbable: Silk, prolene.
Delayed absorbable: Dexon.

INFECTION

Q. What is hospital infection?

Ans:

19 SHARIFUL HALIM, K66, DMC


Sharifs personal notes

Infection acquired at hospital stay which generally presents after 3 days of admission or within 2 days
of discharge.

1. What are the common organisms of surgical infection?

Ans:

Streptococcus pyogenes.
Pneumococcus.
Klebsiella.
E. coli.
Pseudomonas.
Proteus.
Staphylococcus aureus.
Anaerobes.
Q. What is asepsis?
Ans: It is a process of prevention/reduction of contamination of surgical wound.

Q. How will you control hospital infection?

Ans:

Pre-OT care:

Isolation of infected patient.


Correction of anemia.
Correction of malnutrition.
Control of DM.
Gut sterilization.

OT:

OT isolated from general facilities.


Sterilization of OT.
Proper ventilation.
Sterilization of all instruments and other OT facilities that come in contact with the patient.
Scrubbing.
Gloving.
Gowning.
Masking.
Minimum numbers of staff.

During operation:

Proper painting and draping.


Less tissue injury.
Proper method of wound closure.
Proper disposal of contaminated articles.

20 SHARIFUL HALIM, K66, DMC


Sharifs personal notes

After operation:

Isolation of high risk patient.


Regular sterile dressing.
Prophylactic antibiotics.

ABSCESS, BOIL, CARBUNCLE, CELLULITIS

Abscess:

1. Define abscess.

Ans:

Localized collection of pus in a cavity covered by pyogenic membrane.

2. Compare abscess with cellulitis.

Ans:

Point Cellulitis Abscess


Demarcation Diffuse- no sharply demarcated Well localized.
edge
Pus Not present initially Formed pus
Spread Spreads Doesnt spread
Fluctuation Negative Positive
Immunosuppression + -
Septicemia Can occur Doesn t occur
Main mode of treatment Antibiotics, elevation, glycerin I&D
dressing.
Incision as treatment Not practiced until pus forms. Essential

3. Describe the management of a case of abscess in the groin. /Write down the principles of
treatment of abscess. /Write SN on drainage of abscess.

Ans:

Initially broad spectrum antibiotics are started.


Under regional block anesthesia after cleaning & draping, abscess is aspirated &
presence of pus is confirmed.
Skin is incised adequately, in the line parallel to neurovascular bundle in the most
dependent position.
Pyogenic membrane is opened using sinus forceps ( Esp. in groin, axillary region) & all
loculi are broken up. Abscess cavity is cleared off pus & washed with saline.
A drain is placed (Gauze drain or corrugated rubber drain).
Wound is not closed. Rather it is allowed to granulate & heal. Sometimes secondary
suturing or skin grafting is required.
Pus is sent for C/S.

21 SHARIFUL HALIM, K66, DMC


Sharifs personal notes

Antibiotics are continued.


Analgesics and antipyretics.
Treatment of any underlying cause.

Q. How will you treat a case of ischiorectal abscess?

Ans:

Incision and drainage of abscess: The only difference is cruciate incision, rest from the
previous question.

Other points about abscess:

Types:
Pyogenic.
Pyemic.
Metastatic.
Cold abscess.
Structure: From outside inwards- pyogenic membrane, granulation tissue, loculi, pus.
Common organisms:
Commonest: Staph aureus.
Others: Strep. Pyogenes, Gram ve, anaerobes.
Factor precipitating abscess formation: Immunosuppression.
Symptoms:
Fever with chills & rigors.
Throbbing pain.
Localized swelling.
Sign:
Red edematous swelling with visible pointing pus.
Increased temperature over the area.
Tenderness.
Smooth, soft & fluctuant.
Common sites:
Perianal & Ischiorectal region.
Axilla.
Breast.
Hands.
Tonsils.
Internal organs & spaces: Liver, lungs, brain, intra-abdominal abscesses.
Investigations:
CBC with ESR.
Pus for Gram stain & C/S.
Internal abscess: X-Ray, USG, CT scans according to site.
Complication of an abscess:
Bacteremia, septicemia & pyemia.
Local destruction of tissues.
Erosion into adjacent vessels & life threating hemorrhage, e.g. as in pancreatic
abscess.
Sinus & fistula formation.

22 SHARIFUL HALIM, K66, DMC


Sharifs personal notes

Antibioma formation.
Specific complication according to deep sites.
Features of a formed abscess:
Excruciating pain.
Visible pus.
Pointing tenderness.
Fluctuation.
Abscesses should be formed before draining, exceptions are:
Parotid, breast, axilla, ischiorectal, thigh abscess.
Difference between pyogenic & cold abscess:

Point Pyogenic Cold abscess


Signs of inflammation Present Absent
Responsible organisms Pyogenic Tubercular bacteria
Incision used for drainage Dependent Nondependent
Suturing after drainage Not done Done
Drain tube placement Done Not done (risk of sinus
formation)

Q. What is a boil?

Ans:

It is an acute staphylococcal infection of a hair follicle.

Characteristics:

Often opens at its own & subsides.


Common in back, neck, thigh.
Boil in eyelash called stye.
Suppuration can occur in a boil.
Treatment: Antibiotics- Cloxacillin/amoxicillin.
Complications: Cellulitis, Hidradenitis.

Carbuncle:

1. What is carbuncle?

Ans: It is an infective gangrene of skin & subcutaneous tissue.

It is a necrotizing infection of skin and subcutaneous tissues composed of a cluster of furuncles/boils,


usually due to staph aureus infection with multiple draining sinuses.

Symptoms:

Pain.
Fever.

Signs:

23 SHARIFUL HALIM, K66, DMC


Sharifs personal notes

Local: Erythema, edema.


Tenderness.
Blackish discoloration.
Multiple boils.
Discharging sinus.

Investigations:

CBC with ESR.


Pus for microscopy C/S.
FBS/ 2HRABF/HbA1c.
2. Write the management of carbuncle on the nape of the neck.

Ans:

Incision and debridement: Cruciate incision is given along with debridement of all dead
tissues. Excision is done later.
Wound is kept open.
Regular dressing.
Administration of broad spectrum antibiotics: Penicillins- Flucloxacillin, cephalosporins-
Cephradine or depending on C/S.
Skin grafting/Secondary suturing: Once wound granulates well.
Analgesics.
Control of DM using insulin if present.
Control of other comorbid conditions.

Q. What are the complications of carbuncle?

Ans:

Septicemia
Meningitis.

Cellulitis:

2. Define cellulitis.

Ans:

It is a spreading inflammation of subcutaneous & fascial planes (Deep subcutaneous tissue).

3. What are the causative organisms of cellulitis?

Ans:

Common: Streptococcus pyogenes.

Others: Klebsiella, Pseudomonas, E. coli.

4. What are the dangerous areas of cellulitis?

24 SHARIFUL HALIM, K66, DMC


Sharifs personal notes

Ans:

Orbital cellulitis.
Area of upper lip & lower part of nose.
Ludwigs angina/ cellulitis of neck.
Hand.

Q. What are the risk factors of cellulitis?

Ans: Std. infectious risk factors.

5. How will treat a case of cellulitis of neck (Ludwigs angina)?

Ans:

IV broad spectrum antibiotics.


Early surgical decompression: Under G/A mylohyoid muscles of both sides are cut.
Incised wound closed by suturing once infection is controlled completely.

Other important points about cellulitis:

Common in: People who are immunosuppressed.


Area: Face, lower limb, scrotum.
Sequel:
Pyogenic abscess.
Bacteremia, septicemia, pyemia.
Local gangrene.
Symptoms:
Fever.
Pain.
Sign:
Increased temp.
Tachycardia.
Red, shiny area with stretched/tense warm skin.
Tenderness over the skin.
Tender regional lymphadenopathy.
Management:
Elevation of limb: To reduce edema.
Broad spectrum antibiotics: Penicillins-Flucloxacillin, cephalosporins-Cephradine.
Dressing: Often glycerin dressing is used to reduce edema thru hygroscopic
action.
If suppuration occurs: I&D.
Control of DM.

Gangrene:

Q. What is gangrene? Write down some common causes of gangrene.

Ans:

25 SHARIFUL HALIM, K66, DMC


Sharifs personal notes

Gangrene: Death of macroscopic portion of tissue, often with putrefaction.

Common causes of gangrene:

Buergers disease.
Diabetic foot.
Atherosclerosis.
Trauma.
Embolization.
Raynaud s disease.
Carbuncle.
Gas gangrene.

Q. How will you differentiate dry and moist gangrene?

Ans:

Points Dry gangrene Moist gangrene.


Cause Arterial occlusion Arterial and venous occlusion
Type of necrosis Coagulative Liquefactivie
Line of demarcation Present Absent
Infection Absent Present
Spread Slowly Rapidly
Discharge Often absent Seropurulent or blood stained

Q. What are the factors responsible for a diabetic gangrene?

Ans:

Neuropathy.
Vasculopathy.
Reduced immunity.

Gas gangrene:

Q. What is gas gangrene?

Ans:

It is an infective gangrene of the muscle and deep subcutaneous tissue usually caused by clostridial
organism.

Q. What are the organisms responsible for gas gangrene?

Ans:

Clostridium perfringes/welchii.
Clostridium histolyticum.
Cl. Sordeli.
Cl. Septicum.

26 SHARIFUL HALIM, K66, DMC


Sharifs personal notes

Q. Write down the management of a case of gas gangrene.

Ans:

History:

Lacerated or crushed injury.


Contaminated wound.

Symptoms:

Pain at the wound site.


Swelling.
Fever.
Features of shock.

Signs:

Wound is under tension.


Swelling.
Tenderness.
Skin: Khaki brown coloured.
Crepitus may be felt.
Signs of shock.

Investigations:

X-Ray of the local part.


Discharge for C/S.
CBC with ESR.
S. electrolyte.
S. creatinine.
LFTs.

Treatment:

General:
Oxygen inhalation, Ventilatory support if needed.
Hyperbaric oxygen.
IV fluids.
Blood transfusion.
Urinary catheterization.
Tetanus prophylaxis.
Specific:
Anti-gas gangrene serum/Polyvalent antiserum: 25000U IV and repeated after 6
hours.
Inj. benzyl penicillin 20 lacs 4 hourly + Inj metro 500mg TDS+ Inj. gentamicin.
Wound:

27 SHARIFUL HALIM, K66, DMC


Sharifs personal notes

- Liberal incision.
- Debridement of dead tissues.
Amputation in severe cases.

Tetanus:

1. Describe prophylaxis against tetanus.

Ans:

Tetanus toxoid:
In adults: 0.5ml IM TT in 0, 1 & 6 months. Booster dose should be given once in
every 4 years or after any significant trauma.
In infants: Pentavalent - 6, 10, 14 weeks.
Reproductive female: 0, 1, 6m, 1yr, 1 yr.
th th
Pregnant mother: Given in 20 , 24 weeks of pregnancy.
ATG:
500 U IM as a prophylaxis in:
RTA.
Severe burns.
Crush injuries.
Penetrating wounds.
War wounds.
Wounds of the head & face.
2. What are the C/F & management of a case of tetanus?

Ans:

C/F:

Symptoms:
Jaw stiffness.
Pain & stiffness in the neck & back muscles.
Anxiousness, sweating.
Dyspnea.
Convulsions.
Dysphagia.
Headache.
Signs:
Trismus.
Risus sardonicus (smiling facies/saternic smile).
Opisthotonus.
Neck rigidity.
Hyper-reflexia.
Tonic clonic convulsions.
Treatment:
General:
Isolation.
Avoidance of noise & light.

28 SHARIFUL HALIM, K66, DMC


Sharifs personal notes

IV fluids.
Oxygen by nasal cannula/ ET intubation/ IPPV.
Urinary catheterization.
NG tube & suction.
TPN.
Suction of airway.
Specific:
ATG 3000 U IM or ATS 50000 U IM & 50000 U IV.
Inj. penicillin 20 lacs QDS.
Inj. Gentamicin & metronidazole.
Inj. TT .5 ml IM- To deltoid muscle.
IV diazepam/ phenobarbitone.
Wound care: Debridement, drainage & local injection of ATG.

Important point:

Predisposing factors of tetanus:


Trauma with laceration, soil contamination, deep wounds, crushed devitalized tissue.
Improper sterilization of surgical instruments & OT, ward, Labor room.
Cause of death:
Respiratory failure, ARDS, Aspiration pneumonia.
Severe carditis.
Other complications:
Fracture of bones.
Hematoma.
DVT, PE.

Hemorrhage

1. Define & classify haemorrhage.

Ans:

Haemorrhage: Extravasation of blood is called haemorrhage.

Classification:

I. Based on time of onset:


a. Primary: At the time of injury or operation.
b. Reactionary: Within 24 hours of surgery.
c. Secondary: 7-14 days after surgery/primary hemorrhage.
II. Based on the source of bleeding:
a. Arterial: Bright red, spurting like jet.
b. Venous: Dark red, steady & continuous flow.
c. Capillary: Rapid, bright red bleeding/.
III. Based on visibility:
a. Revealed: Visible external hemorrhage.
b. Concealed: Internal hemorrhage.

29 SHARIFUL HALIM, K66, DMC


Sharifs personal notes

c. Revealed on concealed: Hematuria, hematemesis, melena, hemoptysis.


IV. Based on time:
a. Acute: Sudden, severe.
b. Chronic: ie PUD.
c. Acute on chronic: Hematemesis due to PUD.
2. Give 5 examples of revealed hemorrhage.

Ans:

Bleeding after skin cut.


Hematemesis.
Hematochezia.
Hemoptysis.
Hematuria.
Menstrual bleeding.
Epistaxis.
3. What are the different methods of controlling hemorrhage in surgical practice?

Ans:

Methods of controlling hemorrhage:

i.
Pressure:
a. Pressure by gauze: Over the site of hemorrhage.
b. Digital pressure: over the proximal artery.
c. Tourniquets: at least 2 inches above the bleeding site.
ii. Elevation: Of affected area above heart level.
iii. Rest: Pt. should lie down; affected part of the body is immobilized/rested.
iv. Local hemostatic agent:
a. Gelatin sponge.
b. Collagen sponge.
c. Topical thrombin.
d. Oxidized cellulose.
v. Operative techniques:
a. Pressure by artery forceps.
b. Diathermy.
c. Ligation of vessel.
d. Excision of bleeding source e.g. splenectomy.
4. How will you investigate & treat a case of bleeding following circumcision of 7 years old boy?

Ans:

Investigations:

BT.
CT.
aPTT.
PT.
CBC with PBF.

30 SHARIFUL HALIM, K66, DMC


Sharifs personal notes

Factor VIII and IX measurement in blood.

Management:

The child probably has hemophilia. The tt is given below:

Immediate: Blood transfusion, Tranexamic acid, DDAVP/Desmopressin acetate.


FFP/Fresh Frozen Plasma or cryoprecipitate.
Recombinant factor VIII or IX replacement.
Advice:
Avoidance of contact sports: ie football, handball, hockey, wrestling.
5. Name the situations where you apprehend reactionary hemorrhage.

Ans:

Tonsillectomy.
Thyroidectomy.
Haemorrhoidectomy.
Prostetectomy.
Cholecystectomy.
Major abdominal surgery.
6. How do you manage a patient with massive hematemesis?

Ans:

Immediate resuscitation:

Short history about episodes and amount of blood lost.

Quick assessment of vitals:

o Pulse: Rapid and thready.

o BP: hypotension.

o RR: increased.

o Temperature: decreased

o Other features of hypotension: sweating, cold clammy extremities,


confusion, disorientation.

Establishment of IV access with 2 large bore (16G) IV cannula:

o IV crystalloid: Ringer s lactate, Hartman s solution, DNS, NS is given.

o IV colloid can be given.

o Blood is collected and sent for grouping and cross matching.

o IV omeprazole.

31 SHARIFUL HALIM, K66, DMC


Sharifs personal notes

o Grouped and cross matched blood is transfused- about 2 units.

o If patient is in shock- oxygen thru facemask is given.

o Other options: Tranexamic acid, octeotride.

o If bleeding continues- urgent upper GI endoscopy & control of bleeding


i.e. by laser photocoagulation, electrocauterization, bipolar diathermay,
injection of sclerosing agent, local inj dilute adrenaline, if varices- EVL:
Endoscopic variceal ligation.

General management after resuscitation:

Monitoring:

o Vitals check in regular interval.

o Catheterization and monitoring of urine output.

o For new episode of hematemesis.

Identifying the coz of hematemesis:

o Symptoms of CLD.

o Symptoms of PUD.

o If CLD and PUD are excluded, then other causes can be sought out.

o In every case, if emergent endoscopy is not done before, it should be


done now to identify the cause of hematemesis.

7. How will you manage a case of intra-abdominal hemorrhage following blunt abdominal
trauma?

Ans:

Immediate resuscitation & simultaneous quick preparation for surgery:


1. Resuscitation:
Maintenance of ABC.
Quick assessment of BP & pulse.
Establishment of IV access with 2 large bore IV cannula.

IV crystalloid at hand: Ringer s lactate, Hartmann s solution, DNS, NS is given.

IV colloid can be given: i.e. Albumin 4.5%, Haemacel, Dextran, Plasma infusions.

Blood is collected and sent for grouping and cross matching.

Grouped and cross matched blood is transfused- about 2 units.

2. Emergency Preparation for surgery & Definitive management:

32 SHARIFUL HALIM, K66, DMC


Sharifs personal notes

Broad spectrum IV antibiotic.


Foley catheterization.
CVP monitoring.
Analgesics: Morphine 10-20mg IV/IM.
Exploratory laparotomy & control of bleeding: Ligation of bleeding vessel, vessel
anastomosis, if ruptured spleen- Splenectomy.
8. Give 5 examples of internal hemorrhage.

Ans:

Hemorrhage into any body cavities are internal hemorrhage:

a. Epidural hemorrhage.
b. Subarachnoid hemorrhage.
c. Haemothorax.
d. Cardiac temponade.
e. Intra-abdominal hemorrhage: ruptured spleen, liver, ectopic gestation.
f. Hemorrhage due to fracture of femur.
9. What are the C/F of internal hemorrhage?

Ans:

Symptoms:

Sweating.
Increased thirst.
Air hunger.
Weakness/lethargy.
Restlessness.
Altered level of consciousness: Drowsiness, confusion, coma.
Dry mouth.
Pallor.

Signs:

Pulse: Rapid thready.


BP: Low BP.
RR: Increased.
Cold clammy periphery.
Urine output: Reduced <.5ml/kg/hour.
Sunken eye.
Dry mouth.

Shock

1. Define shock.

It is a systemic state of low perfusion which is inadequate for cellular respiration.

33 SHARIFUL HALIM, K66, DMC


Sharifs personal notes

2. Classify shock.

Ans:

Classification of shock:

Hypovolemic shock.
Cardiogenic shock.
Obstructive shock.
Distributive shock.
Endocrine shock.
3. Mention 5 causes of hypovolemic shock.
Hemorrhage: External or internal.
Excessive vomiting: ie in intestinal obstruction.
Non-remitting diarrhea.
Excessive sweating: ie in hot climates.
Major burn.
4. Write the pathogenesis of hypovolemic shock.
5. What are the principles of management of hypovolemic shock?

Ans:

Immediate resuscitation:
Maintenance of ABC.
Quick assessment of BP & pulse.
Establishment of IV access with 2 large bore IV cannula.

IV crystalloid at hand: Ringer s lactate, Hartmann s solution, DNS, NS is given.

IV colloid can be given: i.e. Albumin 4.5%, Haemacel, Dextran, Plasma infusions.

Blood is collected and sent for grouping and cross matching.

Urinary catheterization.

Tt of the cause:
If bleeding- Control of hemorrhage & blood transfusion.
Vomiting: NPO, NG suction, anti-emetic drugs.
Diarrhea: No further mx other than fluid resuscitation.
If burn: Dressing of burn.

6. What are the C/F & management of septic shock?

Ans:

C/F of septic shock:

Symptoms:

34 SHARIFUL HALIM, K66, DMC


Sharifs personal notes

Sweating.
Increased thirst.
Palpitation.
Air hunger/SOB.
Weakness/lethargy.
Altered level of consciousness: restlessness, agitation, drowsiness, confusion, coma.
Fever or hypothermia.

Signs:

Pulse: Rapid, >90/min.


BP: Low BP.
RR: Increased: >20/min.
Periphery: May be warm.
Urine output: Reduced <.5ml/kg/hour.
Sunken eye.
Dry mouth.

Treatment:

IV fluids.
Oxygen thru face mask or mechanical ventilation if needed.
IV broad spectrum antibiotics.
Inotropes: Dobutamine.
Vasopressor: Dopamine, Adrenaline, NAdrenaline.
IV hydrocortisone.
Recombinant activated protein C (Drotreogin alpha).

Fluid & electrolytes

1. What are the common IV fluids used in surgical practice?

Ans:

Crystalloid:
Ringers lactate.
Hartmanns solution.
.9%, .45%, .18% NS.
5% Dextrose in NS.
5% DA.
Isolyte: P, G, M.
Colloids:
5% Albumin.
Haemacel.
Hetastarch.
Dextran 40/70.

35 SHARIFUL HALIM, K66, DMC


Sharifs personal notes

2. Outline the fluid regimen for a first 3 days for a patient who undergone a major abdominal
surgery. How do you manage the usual turnover of body fluid after major surgery?

Ans:

Principle of post-operative fluid regimen:

Operation is a trauma which increases release of counter-regulatory hormones causing retention of


salt & water. So in the earlier post-operative period the fluid & salt requirement is less. But it
increases later.

i.First day/24 hours:


Only glucose containing fluid like 5% DA is enough. Pts daily requirement is calculated
from his wt. & any ongoing losses are estimate before replacement. Commonly 2L of
5% DA.
No Sodium, Chloride or potassium containing fluids are given.
ii. Second day/up to 48 hours:
1L liter of NS in 2L of 5% DA is added.
The extra volume of daily requirement is replaced by 5% DA.
iii. Third day/up to 72 hours:
Along with the regimen of second day, 1mmol/kg body wt. Potassium is given in
divided doses thru the 5% DA solution. Any possible ongoing potassium losses are
calculated & given along with daily requirement of 1 mmol/kg/day.
3. What is the usual turnover of body fluid in adults of average body built per 24 hours?

Ans:

Usual turnover of body fluid in adults of average body built per 24 hours.

Intake (ml) Output (ml)


2500ml 2500ml
From beverage: 1200 Urine 1500ml
From foods: 1000 Insensible loss (skin, lungs) 900ml
Metabolic water 300ml Feces 100ml

nd
4. Describe the fluid regimen for an adult in the 2 POD of partial gastrectomy.

Ans:

Total daily requirement is calculated from his body wt., age & any ongoing losses.
1L liter of NS and 2L of 5% DA is given first.
The extra volume of daily requirement is replaced by 5% DA.
5. What are the fluid & electrolyte changes in GOO? How will you treat them?

Ans:

Fluid & electrolyte changes in GOO:

36 SHARIFUL HALIM, K66, DMC


Sharifs personal notes

Hypochloremia.
Hyponatremia.
Hypokalemia.
Hypocalcemia.
Hypomagnesemia.
Metabolic alkalosis.
Dehydration/Hypovolemia.
Paradoxical aciduria.

Treatment of fluid & electrolyte changes:

0.9% NS is often enough not only to correct Hyponatremia & Hypochloremia, but also mild
to moderate hypokalemia.
If severe hypokalemia, hypocalcemia or hypomagnesemia occurs replacement of these
electrolytes is needed thereafter.
6. What are the causes & complications of hyperkalemia?

Ans:

Causes:

Renal failure.
Diabetic ketoacidosis.
Metabolic acidosis.
Rapid infusion of potassium.
Tissue destruction: ie Burns, trauma, tumor necrosis, crush injury.

Complications:

Ventricular tachycardia.
Ventricular fibrillation.
Heart block.
Sudden cardiac arrest.
Muscle paralysis

TPN: Total Parenteral Nutrition.

Definition: It is Process by which all nutritional requirements are given only thru IV route, not thru
GIT.

Indications:

Failure or contraindication for any enteral nutrition for 7-10 days.


High output abdominal fistulas, duodenal, biliary, pancreatic fistulas.
Major abdominal surgeries of liver, pancreas, biliary, colonic.
Severe burns.

37 SHARIFUL HALIM, K66, DMC


Sharifs personal notes

High risk of aspiration.


MOF, head injury.

Technique of insertion:

Peripheral line: It can be done by PICC- Peripherally Inserted Central venous Cather or
conventional short cannula in wrist veins.
Central line: Can be done thru SC vein in infra-clavicular route, External/Internal jugular
vein.

Components used in TPN:

CHO: e.g. Dextrose 50% solution.


Fat (Emulsion containing long chain TAG) & Amino acids.
Vitamins, trace elements & minerals/electrolytes.

Complications:

1. Technical/During or following insertion/removal:


Bleeding.
Pneumothorax.
Air embolism.
Thrombosis/Thrombophlebitis.
Sepsis.
2. Biochemical:
Electrolyte imbalance: Hyponatremia, hypokalemia, hypophosphatemia.
Hyperosmolarity.
Hyperglycemia.
Hypercholesterolemia.
Dehydration.
a. MODS.

Definition:

It is progressively becoming irreversible injury of all tissues like kidneys, lungs, liver & GIT.

Cause:

Severe trauma.
Burns.
Sepsis.
Multiple transfusions.
Acute pancreatitis.
Rhabdomyolysis (e.g. in trauma)

Risk groups:

Elderly.
Diabetics.
Alcoholics.

38 SHARIFUL HALIM, K66, DMC


Sharifs personal notes

Pts. With liver disease.


Malnourished.

C/F:

Renal failure: Oliguria.


Liver failure: Jaundice.
Heart failure: Hypotension.
Lungs: Severe respiratory distress & hypoxemia not responding to vigorous resuscitation
efforts.

Types:

Primary MODS: Due to well defined cause like pulmonary contusion, rhabdomyolysis, and
multiple transfusions.
Secondary MODS: Occurs as a result of host response in SIRS.

Treatment:

Admitted in ICU & given:


Ventilator support.
Haemodialysis.
Transfusions.
Antibiotics.
TPN/ enteral nutrition.
b. Metabolic acidosis.

Ans:

Definition: It is an excess acid or base deficit. The cut-off value is a HCO3- level of <21 mmol/L.

Causes:

Increase in fixed acid:


DKA.
Renal failure.
Intestinal strangulation.
Loss of base:
Diarrhea.
Gastrocolic fistula.
Intestinal fistula.

Features:

Anorexia, muscle weakness, vomiting.


Air hunger/Kussmaul s breathing: Rapid, deep, noisy breathing.
Features of shock/dehydration: Cold clammy skin, tachycardia, hypotension.
Altered level of consciousness.

Investigations:

39 SHARIFUL HALIM, K66, DMC


Sharifs personal notes

ABG analysis.
S. electrolytes.
ECG.
S. creatinine & BUN.
RBS.

Treatment:

Correction of hypoxia: Oxygen thru nasal cannula or face mask.


Infusion of sodium bicarbonate: IV 50mmol of 8.4% Sodium bicarbonate. (Astrup formula
for calculation is: Total base deficit/sodibicarb required in mEq/L= Base deficit* Body wt.*
0.3).
Correction of fluid & electrolytes imbalances: ie hypovolemia, hyperkalemia with standard
measures.
Specific therapies for treating the cause: ie Insulin in DM.

Blood transfusion

1. What are the blood products used in clinical practice?

Ans:

Whole blood.
Packed RBC.
FFP.
Cryoprecipitate.
Platelet concentrate.
Factor VIII & IX concentrate.
Platelet rich plasma.
PCC: Prothrombin complex concentrate.
Human albumin 4.5%.
2. Write the complication of massive blood transfusion.

Ans:

Definition of massive blood transfusion:

Replacement of whole blood volume in <24 hours.


Single transfusion of >2.5 L of blood continuously.

Complications:

Volume overload & heart failure.


Coagulopathy: Due to dilutional thrombocytopenia & lack of factor V&VII.
Severe electrolyte imbalance: Hypocalcemia, hyperkalemia, acidosis.
Citrate toxicity.
ARDS, DIC.
Hypothermia.
Citrate toxicity.

40 SHARIFUL HALIM, K66, DMC


Sharifs personal notes

Incompatibility & transfusion reactions.


3. What do you mean by blood component therapy? Mention their names & indications.

Ans:

Blood component therapy: Treatment of any patient with components/fractions prepared from whole
blood is called blood component therapy.

Names & Indications:

Names Indications
Packed RBC Severe anemia.
Human albumin 4.5% CLD, Burns, Severe protein loss, other shock
states
FFP (Contains all coagulation factors) CLD, hemophilia, DIC, Coagulopathy following
Massive blood transfusion.
Cryoprecipitate (Rich in factor VIII & Hemophilia, vWD.
fibrinogen)
Factor VIII & IX concentrate Hemophilia, vWD.
Platelet concentrate (250*10^9/ml) Thrombocytopenia & drug induced
hemorrhage (aspirin, clopidogrel).
PCC: Prothrombin complex concentrate Emergency reversal of warfarin therapy.
(Contains Factors II, IX, X

4. Define transfusion & infusion.

Ans:

Transfusion: Infusion of blood or blood products which are obtained from another person is called
Transfusion.

Infusion: Process of delivery of any product (Fluid, medication) thru IV route is called infusion.

Actually transfusion is a type of infusion.

All transfusions are infusion but all infusions are not transfusion.

5. What do you mean by safe blood transfusion? /Describe the steps how you can transfuse blood
with safety.

Ans:

Provision of all of the following points encompasses safe blood transfusion:

Safe Donor: Voluntary non-remunerated/non-paid blood donors from low risk


populations without any systemic disease or recent history of systemic disease.
Proper Screening: For infectious disease- Hep-B, C, HIV, Malaria, syphilis.
Correct grouping & cross matching.
Appropriate clinical use of drug & use of alternative to transfusion when possible.
Strict aseptic transfusion.
Quality control system covering all stages of transfusion process.

41 SHARIFUL HALIM, K66, DMC


Sharifs personal notes

6. Mention the indications of blood transfusion.

Ans:

Indications:

Acute blood loss/hemorrhage: i.e. following trauma; a loss of >15% blood is a very
important indication of blood transfusion.
During major surgery: Hepatobiliary surgery, Abdomino-Perineal surgery, thoracic surgery.
As a prophylactic measure prior to surgery.
Following major burns.
Chronic severe anemia.
7. Mention the hazards/complications of blood transfusion.

Ans:

Transfusion reactions:
Pyrexial reaction/Febrile reaction: Due to pyrogenic ingredients in blood.
Allergic reaction.
Incompatibility: Major & minor reactions with fever, rigors, pain, hypotension.
Immunological sensitization.
Transmission of infection:
Hepatitis B, C.
HIV.
Malaria.
Syphilis.
Others: CMV, EBV.
Air embolism.
Thrombophlebitis.
Congestive cardiac failure.
TRALI: Transfusion related acute lung injury.
Coagulation failure:
Dilution of clotting factors.
DIC.
Dilutional thrombocytopenia.
Electrolyte imbalance: Hypokalemia, hyperkalemia, hypocalcemia, hypomagnesemia.
8. Write SN on Mismatched blood transfusion.

Ans:

Definition:

Hemolytic transfusion reaction due to incompatibility of major or minor blood groups.

Types:

Major blood group incompatibility: ABO & Rh.


Minor blood group incompatibility: Other blood groups.

Pathogenesis:

42 SHARIFUL HALIM, K66, DMC


Sharifs personal notes

Incompatible blood groups.


Antigen-antibody reaction, activation of complement.
Acute hemolysis.
Hemoglobinemia, Hemoglobinuria.
Acute renal failure.
DIC.

C/F:

Dyspnea.
Chest pain.
Sweating.
Fever with chills.
Low BP.
Low urine output.
Pain at the site of transfusion.
Palpitations.
Jaundice.

Treatment:

Stoppage of transfusion immediately.


Blood is sent for 2 labs for rechecking.
Oxygen inhalation.
IV corticosteroids: Hydrocortisone/ Dexamethasone.
IV antihistamines: Give chlorphenamine 10-20 mg by slow IV injection.
IV fluids & electrolyte replacement: e.g. Ringer s lactate, Hartmann s solution.
IV diuretic: Mannitol 20g/100ml in 5 minutes, Frusemide 120mg IV.
Hemodialysis: If renal failure develops.
If DIC suspected: FFP is given.
Ventilatory support, cardiac defibrillator.

BURN

1. Define & classify burn.

Ans:

Burn: A burn is a tissue damage caused by heat, chemicals, electricity, sunlight or radiation.

Classification of burn:

a. According to the thickness of skin involved:


st
I. 1 degree burn:
Only epidermis is involved.
It is painful & epidermis looks red.

43 SHARIFUL HALIM, K66, DMC


Sharifs personal notes

No blister forms.
Heals rapidly in 5-7 days without scarring.
nd
II. 2 degree burn:
Two types:
Superficial partial thickness burn:
o Involves up to papillary layer of dermis.
o Blistering occurs.
o Underlying tissue is pink and moist.
o Capillaries blanch under pressure.
o Painful.
o Sensation is normal.
o Healing needs 2 weeks.
Deep partial thickness burn:
o Involves up to reticular dermis.
o Dermis is not pink & moist. (Pale and Dry).
o No blanching effect on capillaries.
o Sensation is reduced.
o Leads to hypertrophic scarring.
o Consider for skin graft.
rd
III. 3 degree burn:
Also called full thickness burn.
Whole of the dermis is destroyed.
Clinically: Skin is hard, leathery or charred black.
Painless & insensitive.
Requires grafting.
th
IV. 4 degree burn:
Involvement of underlying tissue: Muscles, bones.
2. What are the differences between superficial (partial thickness) & deep (partial thickness) burn?

Ans:

Facts Superficial burn (only Deep burn.


superficial/superficial partial
thickness)
Involvement of skin Up to papillary dermis Up to reticular dermis.
Blistering May Occur Doesnt occur
Color of underlying tissue Pink Pale
Moisture Moist Not moist/dry
Capillary Blanching Blanches with pressure Doesnt blanch.
Sensation Intact Reduced
Healing Needs 2 weeks 3-4 weeks
Skin grafting Not required May be required.

3. What are the complications of burn?

Ans:

Local:

44 SHARIFUL HALIM, K66, DMC


Sharifs personal notes

Airway obstruction: In head neck burn.


Eschar formation & distal ischemia esp. in limb.
Cosmetic problem: Ectropion.
Hypertrophic scar, keloid.
Contracture.
Stricture: i.e. esophagus, pyloric stenosis.
Marjolins ulcer.

Systemic:

Shock.
Renal failure.
Pulmonary edema & ARDS.
Septicemia.
Curlings Ulcer in GIT.
Toxic shock syndrome.
Wt. loss.
4. How do you manage a case of 30% burn in an adult?

Ans:

1. General/ immediate management:


Hospitalization.
ATLS: Management of ABC.
Continuous irrigation of the burnt area with tap water for at least 20 minutes.
Fluid resuscitation:
First 24 hours: Crystalloids are given. Calculation of fluid requirement by
Parklands formula ie Total fluid requirement in 24 hours= % of TBSA burnt* wt. in
kg*4ml. Half of 24 hours requirement is given in first 8 hours. Next half in later 16
hours. Choice of fluid is Ringer s lactate. But Hartmann s solution or normal saline
can be used.
Next 24 hours: Colloids are given (.4ml* Wt.in kg* TBSA) ie plasma, haemacele,
and dextran.
Broad spectrum antibiotics.
Analgesics.
Urinary catheterization.
Administration of tetanus toxoid.
Insertion of Ryle s tube: For aspiration initially and feeding in the later period.
2. Local management:
Cleaning of burnt area with povidone iodine.
Open method: Application of Silver sulfadiazine/silver nitrate/ Mafenide acetate/
sulfamylon.
Closed method: Burn dressing at regular interval under G/A using:
Vaseline impregnated gauze.
Hydrocolloids
Paraffin gauzes.
Biological dressings like amniotic membrane, synthetic biomembrane.

45 SHARIFUL HALIM, K66, DMC


Sharifs personal notes

In deep dermal wound: Tangential excision of wound with skin grafting.

5. How does burn affect fluid & electrolyte balance? /Write down the pathophysiological changes
in massive burn.

Ans:

In burn coagulation necrosis of skin & SC tissue occurs.


Burnt skin or tissue activates a Web of inflammatory cascade ie activation of
complement system, Hageman factor, AA pathway, thrombin & Kallikrein pathway.
The release of vasoactive substances due to activation of these pathways increases
vascular permeability which causes escape of intravascular fluid to extravascular
space not only in the burnt area but also in the distal areas.
Leads to hypovolemia, shock & MODS.
6. How do you resuscitate a patient of 30 years old & 50 kg weight bearing 30% scald burn from
hot water?

Ans:

ATLS: Management of ABC.


Continuous irrigation of the burnt area with tap water for at least 20 minutes.
Fluid resuscitation:
First 24 hours: Crystalloids are given. Calculation of fluid requirement by Parklands
formula ie Total fluid requirement in 24 hours= % of TBSA burnt* wt. in kg*4ml. In
this patient: 30*50*4= 6L. 3L is given in first 8 hours. Next 3L in later 16 hours.
Choice of fluid is Ringer s lactate. But Hartmann s solution or normal saline can be
used.
Next 24 hours: Colloids are given ie plasma, haemacele, and dextran.
Antibiotics, analgesics.
Urinary catheterization.
Administration of TT.
Insertion of Ryle s tube.
7. Write a short note on: Burn wound care.

Ans:

Cleaning of burnt area with povidone iodine.


Open method: Application of Silver sulfadiazine/ silver nitrate/ Mafenide acetate/
sulfamylon.
Closed method: Burn dressing at regular interval under G/A using:
Vaseline impregnated gauze.
Paraffin gauzes.
Hydrocolloids.
Biological dressings like amniotic membrane, synthetic biomembrane.
In deep dermal wound: Tangential excision of wound with skin grafting.
Skin grafting:
Done once the area granulates well.
Small area: SSG- Split Skin Graft.
Larger area: MESH SSG- Mesh Split Skin Graft.

46 SHARIFUL HALIM, K66, DMC


Sharifs personal notes

Face & Ear: Full thickness graft.


Grafting of cultured skin if available.
If there is a circumferential Eschar over a limb: Escharotomy.
8. Write a short note on: SKIN GRAFT.

Ans:

Definition: It is transfer of skin from one area (Donor area) to the required defective area (recipient
area). It is an autograft.

Types:

1. Partial thickness graft:


Also called SSG/Split-thickness skin Graft/ Thiersch graft.
It involves removal of full epidermis+ part of the dermis from the donor area.
It may be: Thin, intermediate & thick SSG.
Common donor area: Thigh.
Indication: Well granulated ulcer, clean wound which cant be apposed ie after
mastectomy.
Contraindication: Can t be done over bone, tendon, cartilage, joint.
Disadvantage: Contracture of graft, infection, graft failure Blunting of sensation,
dryness of kin, seroma.
2. Full thickness graft:
Also called Wolfe graft.
Includes both epidermis+ full dermis.
Used over: Face, joints, hand, eyelid.
Common donor area: Post-auricular area, supraclavicular area, groin crease area.
Advantage: No contracture, sensation better than SSG, functional results better
than SSG.
Disadvantage: Only for small area, cant be used to cover ulcers.
9. Write a SN on: Post burn contracture.

Ans:

It is a common complication of burn which occurs along or over:

Joint.
Eyelids.
Lips.
Neck.
Cheeks.
Elbow.
Knee.

Pathology:

Scar shortening occurs.


May be intrinsic by loss of tissue.
May be extrinsic by pull during healing phase contraction.

47 SHARIFUL HALIM, K66, DMC


Sharifs personal notes

Contraction proceeds towards position of comfort until it meets or closely reaches


opposite surface.

Complication of burn contracture:

Ectropion of eyelid causing keratitis & corneal ulcer.


Disfigurement.
Microstomia: Narrowing of mouth.
Restricted movement of the neck.
Restricted joint movement: Knee, neck, elbow.
Hypertrophic scar formation.
Repeated breaking of scar & infection.
Pain & tenderness over the scar contracture.
Marjolins ulcer: SCC occurring in a scar ulcer due to repeated breakdown.

Treatment for contracture:

Surgical release of contracture: Using Z plasty, skin graft or different flaps.


Proper physiotherapy & rehabilitation.
Pressure garments: To prevent hypertrophic scars.
Management of itching over the scar: Antihistamines & moisturizing creams.

ANAESTHESIA

1. What are the types of regional anaesthesia?

Ans:

Topical.
Infiltration block.
Field block.
Nerve block.
Epidural.
Spinal.
2. What do you mean by pre-anaesthetic assessment?

It is the process/system of recognition of general medical & specific anesthetic risk factor to facilitate
implementation of pre-emptive measures & improvement of patient safety.

Another: Assessment of the patient prior to anesthesia to see whether he/she is fit for anaesthesia.

3. How will you assess a patient of 50 years for cholecystectomy with G/A in a patient suffering
from intestinal obstruction?

Ans:

48 SHARIFUL HALIM, K66, DMC


Sharifs personal notes

Patient assessment:

History:

General Disease: DM, HTN, BA, PTB, COPD, Heart disease, Stroke, CLD, anemia.
Drug therapy: Steroid, Anti-HTN, sedatives, antibiotics, anti-epileptics.

Examination:

G/E: Nutrition, Anemia, Jaundice, dehydration, cyanosis; vitals: Pulse, BP, RR, temperature.
Systemic examination:
Examination of CVS, Respiratory system, GIT, MKS, Nervous system & look for
above mentioned diseases.

Investigations:

CBC with ESR.


RBS, HbA1c.
Blood urea.
BUN.
S. creatinine.
S. electrolytes.
Urine R/M/E.
CXR.
ECG.
Blood grouping.
ABG analysis.
HBsAg.
PT/INR, BT.
4. How will you prepare a case of DM for major surgical procedure?

Ans:

History:

Dx of DM confirmed or not, documents are needed.


Drugs taken to control DM.
DM controlled or uncontrolled.

Examination:

G/E & Examination of CVS, Nervous system, eye.

Investigations:

RBS, FBS.
HbA1c.
Urine R/M/E.
RFTs: S. creatinine, S. urea, S. electrolytes.
CXR, ECG.

49 SHARIFUL HALIM, K66, DMC


Sharifs personal notes

Control of DM:

Adequate glycemic control must be achieved 2/3 days before surgery.


Long acting insulin is replaced with short acting insulin.
ODA are stopped & replaced with short acting insulin.
Blood sugar is kept <10mmol/L.
Patient is operated early in the morning.
Before operation he/she is infused with 500ml 10% DA with 10 U of short acting insulin with
20mmol of potassium.
5. Define G/A. What are the stages of G/A?

Ans:

It is the achievement of unconsciousness, pain relief & muscle relaxation, while ensuring the
maintenance of tissue perfusion & oxygenation.

Or

A global reversible CNS depression characterized by a triad of:

Unconsciousness.
Adequate Analgesia.
Adequate muscle relaxation.
6. What are the common complications of G/A that can develop during & immediately after
operation?

Ans:

CVS:

Myocardial depression & cardiac arrest.


Cardiac arrhythmias.
HTN./hypotension.

Respiratory:

Laryngeal & bronchospasm.


Respiratory failure.
ARDS.
Pneumothorax.
Later: Atelectasis, pneumonia, lung abscess.

Others:

Anaphylaxis.
Malignant hyperthermia.
Hypothermia.
Later: DVT.

50 SHARIFUL HALIM, K66, DMC


Sharifs personal notes

7. Enumerate the post-operative pulmonary complications. How do you prevent post-operative


pulmonary complications?

Ans:

Post-operative pulmonary complications:

Aspiration.
ARDS.
Atelectasis.
Pneumonia.
Bronchospasm.
Laryngeal spasm.
Falling of tongue backwards blocking the airway.

Prevention of pulmonary complication:

Deep breathing e.g. by a breathing exerciser.


Coughing.
Postural drainage.
Incentive spirometry.
8. What are the complications of local anaesthetic agents?

Ans:

CVS: depression and potential for arrhythmia, postural hypotension.


CNS: Giddiness, Headaches, Sleepiness, restlessness, tinnitus.
Local complications: Bleeding & hematoma formation, nerve injury, infection. (Bleeding,
injury, infection).
9. What are the advantages of local anaesthetic agents mixed with adrenaline?

Ans:

Adrenaline causes local vasoconstriction. So a decrease in blood flow occurs.

This slows down the absorption of local anesthetics and prolongs their action along with reduction of
systemic toxicity.

10. Write short note on:


a. Spinal anaesthesia.

Ans:

It is the injection of local anesthetic into the subarachnoid space causing loss of sympathetic tone,
sensation & motor function.

Position of the patient: Lateral decubitus position with flexed head, hips & knees.

Site of injection: Between 3-4/4-5 Lumbar vertebra.

Drugs used:

51 SHARIFUL HALIM, K66, DMC


Sharifs personal notes

Lignocaine 5% in 6% Dextrose, 2ml.


Bupivacaine .5% in 5% Dextrose, 3 ml.

Indication:

Hernia, hemorrhoids.
C/S, Hysterectomy.
Orthopedic surgery of femur, tibia.
Prostetectomy.

Disadvantage:

Headache.
Meningism & meningitis: Aseptic or bacterial.
Root damage: Pain, paresthesia, anesthesia, paralysis.
Cord damage.
Bradycardia & Hypotension.
Adhesive arachnoiditis.
b. Epidural anaesthesia:

Ans:

It is the injection of local anesthetic into the epidural space causing loss of sympathetic tone, sensation
& motor function.

Needle used: Touhy needle.

Drugs:

2% xylocaine with adrenaline.


.5% bupivacaine.

Advantages:

Can be used for continuous repeated prolonged anesthesia.


Can be used for postoperative analgesia.
Can be kept for several days.
c. Regional/Local anesthesia.

Ans:

Definition:

It is the process where analgesia, muscle relaxation, loss of reflex in localized area is achieved
without amnesia and unconsciousness.

Routes of administration:

Surface.
Infiltrative.
Field block.

52 SHARIFUL HALIM, K66, DMC


Sharifs personal notes

Nerve block.
Spinal
Epidural.

Indications:

Minor surgery: Draining abscess, excision of sebaceous cyst, excision of Lipoma,


circumcision, episiotomy.
Scopies: Laryngoscopy, bronchoscopy, endoscopy.
Surgery of arm, hand.

Contraindications:

Local infection.
Un-cooperative patient.

M/A:

Blocks inactivated voltage gated sodium channel- prevents propagation of action potential no pain
transmission and local muscle relaxation.


Local complications: Bleeding & hematoma formation, nerve injury, infection. (Bleeding,
injury, infection).
CVS: depression and potential for arrhythmia, postural hypotension.
CNS: Giddiness, Headaches, Sleepiness, restlessness, tinnitus.
11. What are the complications of spinal anaesthesia? How will you prevent them?

Ans:

Complications of spinal anesthetics:

Headache.
Meningism & meningitis: Aseptic or bacterial.
Root damage: Pain, paresthesia, anesthesia, paralysis.
Cord damage.
Adhesive arachnoiditis.
Bradycardia & Hypotension.

Prevention of these complications:


Strict aseptic procedure.

Infallible LP.

Using finger needle during LP.

Adequacy of anesthesia: Adequate doses, intrathecal vasoconstrictor.

Accurate determination of level: Patients description, motor, subjective response to testing,
skin temperature.
12. What is pre-operative analgesia? Give examples.

Ans:

53 SHARIFUL HALIM, K66, DMC


Sharifs personal notes

Process of providing analgesics usually 1h before operation.

Examples:

Pethidine 50mg./
Morphine 10mg./
Diazepam 10mg./
Midazolam 1-2.5 mg.

Q. Enlist general anesthetics.

IV: thiopental, midazolam, morphine, pethidine, fentanyl,. propofol, ketamine.


Inhaled: halothane, isoflurane, desflurane, sevoflurane, nitrous oxide.

Q. What do you mean by Preanesthetic medication? Name some drugs used in this purpose.

Ans:

Preanmed: use of drugs before anesthesia to make it more pleasant and safe is called anesthetic
adjunct or preanmed.

Preanmed Action

Diazepam, midazolam. Reduce anxiety, anterograde amnesia.

Atropine Reduction/drying of bronchial secretion.

Omeprazole , pantoprazole To reduce gastric acid

Broad spectrum antibiotics Prevent infection

Domperidone, ondansetron. Prevent vomiting

Laxatives For emptying GIT

Q. What are the common imaging techniques used in medical practice?

Ans:

X-ray.
CT.
MRI.
USG.
Radionuclide imaging.

Q. What are the indications of emergency USG?

Ans:

Acute appendicitis.

54 SHARIFUL HALIM, K66, DMC


Sharifs personal notes

Acute Cholecystitis.
Acute pancreatitis.
Female: PID, Rupture ovarian cyst, rupture of ectopic pregnancy.

Ileostomy:

Definition:

Surgical opening made in ileum to bypass the intestinal contents to the exterior.

Types:

End.
Loop.
End-loop.
Modified loop.

Indications:

Colorectal cancer.
Crohn s disease.
UC.
Hirschprung disease.
FAP.

Complications:

Hemorrhage.
Infection.
Prolapse.
Retraction.
Excoriation.
Diarrhea.
Stenosis.

CATHETERIZATION:

1. Describe the procedure of aseptic catheterization/Write a short note on catheterization.

Ans:

Definition:

Insertion of a hollow tube thru urethra into bladder for removing urine.

Purpose/indications:

55 SHARIFUL HALIM, K66, DMC


Sharifs personal notes

Relieving urinary retention.


Accurate measurement of urine output esp in shock states.
Measurement of residual urine.
Emptying bladder before, during and after operation.
In treatment of VVF.

Sizes:

Child: 8-10 Fr.


Female: 14-16 Fr.
Male: 20-22 Fr.

Types:

Intermittent catheter: ie used for only 5-10 minutes.


Indwelling catheter/retention catheter: used for longer duration.
Suprapubic catheter.

Steps of aseptic catheterization:

Explanation to the patient about the procedure.


Provision of privacy & adequate lighting.
Position:
Female: Dorsal recumbent with knees flexed & feet about 2 feet apart.
Male: Supine.
Cleaning genital area.
Washing hands.
Put sterile gloves.
Swab the entire glans with clean cotton.
Apply sterile lubricant/local anesthetic jelly in the penile urethra & catheter tip.
Wait for few minutes.
Hold the penis at 90 degree angle & start passing catheter.
When urine starts to flow advance 1-2 inch beyond.
Inflate the balloon using 10cc syringe.
Gently pull the catheter until inflated balloon is snug against bladder neck.
Connect catheter to drainage system.

Complication:

Hemorrhage.
Injury.
Infection.
Stricture.
Mis-puncture.

GIT

Q. Classify salivary gland neoplasm.

56 SHARIFUL HALIM, K66, DMC


Sharifs personal notes

Ans:

Benign: Pleomorphic adenoma, Adenolymphoma/Wertheim s tumor.


Malignant: Mucoepidermoid CA, Adenocarcinoma, SCC.

Q. Describe the epidemiology of salivary gland neoplasm.

Ans:

90% of salivary gland neoplasms affect the parotid gland.


90% of parotid neoplasms are benign.
90% of benign parotid neoplasms are pleomorphic adenoma.
85% of SL neoplasms are malignant.
50% of SM gland tumors are malignant.

Tongue:

Q. What are the predisposing factors of tongue ulcer/cancer?

Ans:

Sharp teeth.
Spices
Smoking.
Spirit- alcohol.
Syphilis.
Sepsis.
TB.

Q. Write down the characteristics of a malignant tongue ulcer.

Ans:

Symptoms:

Painless lump.
Excessive salivation.
Fetor oris.
Difficulty in articulation.
Altered speech.
Ankyloglossia.
Lump in the neck.

Signs:


rd
Site: Common in the margin of the tongue at the anterior 2/3 .
Margin: Irregular.
Floor: Covered with necrotic debris and may bleed on touch.
Edge: Everted.
Base: Indurated.

57 SHARIFUL HALIM, K66, DMC


Sharifs personal notes

Neck LN: May be enlarged.

Q. How will you manage a case of tongue cancer?

Ans:

Symptoms:

Painless lump.
Ulceration.
Excessive salivation.
Fetor oris.
Difficulty in articulation.
Altered speech.
Ankyloglossia.
Dysphagia.
Bleeding.
Lump in the neck.

Signs:

Ulceration.

rd
Site: Common in the margin of the tongue at the anterior 2/3 .
Margin: Irregular.
Floor: Covered with necrotic debris and may bleed on touch.
Edge: Everted.
Base: Indurated.
Neck LN: May be enlarged.

Investigations:

Incisional/excisional biopsy.
Swab for Gram staining and C/S.
To see extension:
X-Ray mandible.
MRI.
CT scan.
CXR.
USG of W/A.
Treatment plan:
Management of the primary lesion:
rd
Anterior 2/3 :
<2cm: Wide local excision/ Laser vaporization.
>2 cm: Mandibulectomy with partial glossectomy and
reconstruction followed by radiotherapy by following
approaches:
Trans-oral.

58 SHARIFUL HALIM, K66, DMC


Sharifs personal notes

Lip splitting.
Visor incision.
rd
Posterior 1/3 : Radiotherapy.
Management of the lymph nodes:
Palpable and mobile: Antibiotics if infected; block dissection if
malignant.
Palpable and fixed: Radiotherapy.

Q. What are the pre-malignant lesions of the tongue?

Ans:

Leukoplakia.
Speckled leukoplakia.
Erythroplakia.
Chronic hyperplastic candidiasis.

PAROTID:

Pleomorphic adenoma:

Q. How will you proceed to confirm your diagnosis?

Ans:

I will do FNAC- For tissue diagnosis.


X ray of local part- To see bony erosion.

Q. What are the complications of operation?

Ans:

Hemorrhage and hematoma formation.


Fistula.
Temporary facial nerve injury.
Permanent facial nerve injury.

Q. What is your treatment plan?

Ans:

1. Counseling:
2. Operation:
Superficial parotidectomy with preservation of facial nerve (Pateys operation).

Adenolymphoma:

Synonym: Papillary cyst adenoma/ Wertheims tumor.


Treatment: Superficial parotidectomy with preservation of facial nerve.

Carcinoma of parotid gland:

59 SHARIFUL HALIM, K66, DMC


Sharifs personal notes

Treatment:
Total conservative parotidectomy: With preserving facial nerve.
Radical parotidectomy with facial nerve grafting.
If tumor is irresectable: Partial resection with postoperative radiotherapy or
radiotherapy alone.

ESOPHAGUS

Q. What are the symptoms of esophageal diseases?

Ans:

Dysphagia.
Odynophagia.
Retrosternal chest pain.
Regurgitation.
Vomiting.

Q. Tell five causes of esophageal dysphagia?

Ans:

Achalasia cardia.
CA esophagus.
Corrosive stricture.
Myasthenia gravis.
Bulbar palsy.

Q. How will you investigate a case of esophageal dysphagia?

Ans:

Barium swallow X-Ray of esophagus.


Upper GIT endoscopy and biopsy.
Esophageal manometry.

Q. What are the common FBs in the esophagus? How will you manage them?

Ans:

Common FB in the esophagus:

Coin.
Battery.
Food bolus.
Small toys.
Marbles.
Dentures.

Management:

60 SHARIFUL HALIM, K66, DMC


Sharifs personal notes

Do an X-Ray of soft tissue of neck (A/P and lateral view).


Removal of FB under G/A.

Q. What are the causes of esophageal perforation?

Ans:

Instrumentation e.g. rigid esophagoscopy.


Sharp foreign body.
Esophageal carcinoma.
Corrosive injury.
Trauma.
Esophageal ulcer.

Q. Write down some esophageal motility disorders?

Ans:

Disorder of pharyngo-esophageal junction:


Bulbar palsy: Stroke, MND, MS.
Myasthenia gravis.
Disorder of body of esophagus: Diffuse esophageal spasm.
Disorder of lower esophagus: Achalasia cardia and systemic sclerosis.

Q. What is achalasia cardia? What are the causes and pathology?

Ans:

Definition: It is a lower esophageal motility disorder occurring due to degeneration of ganglionic cells
of myenteric and meissners plexus resulting in failure of relaxation of LES and loss of peristalsis of
esophageal body.

Causes:

Idiopathic neuronal degeneration.


Disorders of dorsal motor nuclei of vagus- in Chagas disease, polio.
Diabetic neuropathy.
Infiltrative disease: Malignancy, amyloidosis, sarcoidosis.

Pathology:

Degenerative changes of nerves


Failure of relaxation of LES.
Dilation of proximal esophagus.
Dysphagia and regurgitation.

Q. Describe the C/F and management of achalasia cardia?

Ans:

C/F:

61 SHARIFUL HALIM, K66, DMC


Sharifs personal notes

Dysphagia: Earlier for liquid, later/less for solid.


Regurgitation of undigested food: esp. at night.
Heartburn: Esp. just after meal.

Investigations:

Barium swallow X- Ray of esophagus: Shows


Huge dilation of esophagus.
Smooth outline with sudden tapering giving it a BIRD BEAK appearance.
Upper GIT endoscopy and biopsy.
Esophageal manometry.

Treatment:

Conservative:
Medication: Muscle relaxants such as:
Isosorbide dinitrate.
Diltiazem.
Nifedipine.
Inj. of Botulinum toxin.
Dilation by:
Mercury bougies.
Pneumatic dilation.
Hydrostatic dilation.
Surgical: Hellers operation- Cardioseromyotomy.

Q. What are the types of esophagoscopy? Tell the indications and complications of rigid
esophagoscopy?

Ans:

Types of esophagoscopy:

Rigid.
Flexible.

Indications:

Diagnostic:
See lesions: Stricture, ulcer, carcinoma.
Take biopsy.
Therapeutic:
Removal of FB.
Botox injection.
Sclerotherapy.
Polypectomy.
Institution of feeding tube.

Complications:

62 SHARIFUL HALIM, K66, DMC


Sharifs personal notes

Injury:
To oral cavity.
To pharynx and larynx.
To the esophageal mucosa.
Esophageal perforation.

Q. How will you diagnose and manage a case of esophageal perforation during rigid esophagoscopy?

Ans:

Diagnosis during esophagoscopy:

Bleeding leading to obliteration of vision.


Sudden loss of resistance.
Patient passing to hypovolemic shock.
In post OT care, patient complains of:
Pain between shoulder blades.
Hemoptysis.
Cough.
Dysphagia.
Odynophagia.

Management of perforation of esophagus:

Assessment of vitals.
Correction of fluid and electrolytes: IV crystalloids with or without colloids infusion.
IV antibiotics.
Chest drain tube
Referral to chest hospitals for repair of perforation.

Q. A 30 years old male had a H/O suicidal corrosive ingestion with progressive dysphagia? What is the
cause of dysphagia and how will you manage it?

Ans:

Cause: Stricture due to corrosive injury.

Investigations:

Barium swallow X-Ray.


Upper GIT endoscopy and biopsy.
Esophageal manometry.

Treatment:

Hospitalization of patient.
IV fluids infusion.
Antibiotics.
Passing NG tube for feeding.
Steroids for 4-6 weeks.

63 SHARIFUL HALIM, K66, DMC


Sharifs personal notes

For stricture:
Esophagoscopy and dilation.
Esophageal reconstruction or bypass.
Gastrostomy and retrograde dilation.

Q. What is GERD? Write down its C/F and management?

Ans:

GERD

It is clinical condition characterized by abnormal retrograde flow of gastric contents into esophagus
resulting in symptoms and mucosal damage.

C/F:

Retrosternal burning pain.


Dysphagia.
Atypical symptoms:
Cough.
Dental carries.
Excessive watering of mouth.
Wheeze.
Hoarseness.

Investigations:

Ambulatory 24 hours pH monitoring.


Esophageal manometry.
Endoscopy of upper GIT and biopsy.
Barium swallow X-Ray.

Treatment:

Elevation of head in bed.


Avoiding foods at least 1-3 hours before bedtime.
Avoid:
Spicy food.
High fat diet.
Smoking.
Alcohol.
Caffeine.
Carbonated drinks.
H2 receptor blockers.
Increase the tone of LES: Metoclopramide.
Anti-reflux surgery: Nissens fundoplication.

Carcinoma esophagus

Common neoplasms of esophagus:

64 SHARIFUL HALIM, K66, DMC


Sharifs personal notes

Benign: Leiomyoma, polyp.


Malignant: Squamous cell carcinoma, adenocarcinoma.

rd
SCC affects the upper 2/3 of esophagus and Adenocarcinoma affects the lower third.
Risk factors:
Alcohol ingestion.
Smoking.
Chewing tobacco and betel nut.
Achalasia.
Consumption of salted fish.
Pickled vegetables.
Radiation.
Common symptoms:
Due to primary tumor:
Difficulty in swallowing/Progressive dysphagia: Initially for solid food, later for
liquid also.
Wt. loss.
Loss of appetite.
Regurgitation, Nausea & vomiting.
Pain in the chest/Retrosternal pain.
Due to metastasis:
Cough, breathlessness, hemoptysis.
Hoarseness of voice.
Jaundice.
Bone pain.
Focal symptoms.
Examination findings:
G/E: Malnourished, anemic, dehydrated, enlarged Virchow s node, jaundice.
Systemic examination: Nothing significant.
Spread of tumor:
Direct spread: To surrounding mediastinal structures.
Lymphatic spread: Discontinuous. Neck node, mediastinal nodes, and celiac nodes-
satellite nodule: has poor prognosis.
Blood borne metastasis: Liver, lungs, bone, brain, adrenals.

Investigations:

FOR Diagnosis:
Upper GIT endoscopy & biopsy.
Barium swallow X-Ray of esophagus:
Irregular filling defect.
Rat tail appearance.
Proximal dilation.
Shouldering effect.
For distant metastasis/To stage the patient:
CXR: Esophagus- Air column, air fluid level & also to see lung metastasis-
Consolidation, lung abscess, Pl. effusion.
USG of W/A: Liver mets, LN, Ascites.

65 SHARIFUL HALIM, K66, DMC


Sharifs personal notes

CT scan of Chest:
Origin & extent of tumor.
Surrounding structure.
Mediastinal LN.
Lung mets.
CT scan of abdomen: Liver mets, LN, Ascites.

Management:

Pre-op preparation:
Investigations of G/A: CBC with ESR, blood RFT, Urine R/M/E, LFTs, BT, PT, HBsAg.
Preparation of patient.
Correction of anemia.
Correction of malnutrition.
Correction of dehydration.
Correction of electrolyte imbalance.
Control of DM, HTN.
Operation:
For upper third: Radiotherapy + Mckeowns three stage operation.
For middle third: Radiotherapy + Ivor Lewis two stage operation.
For lower third: Partial/total esophago-gastrectomy with Roux-en-Y esophago-
jejunostomy.

Q. What are the premalignant conditions of esophagus?

Ans:

Reflux esophagitis.
Barretts esophagus.
Achalasia cardia.
Corrosive stricture.
Plummer Vinson syndrome.

Q. What are the important complications of carcinoma of esophagus?

Ans:

Aspiration pneumonia: The most common cause of death.


Fatal hemorrhage.
Esophageal perforation and Mediastinitis.
Malnutrition.
Tracheo-esophageal fistula.

Q. How will you assess nutritional status of a patient?

Ans:

Clinical assessment:
Medical history: Wt. loss, dietary intake, functional status/energy level.
Examination: SC fat, muscle wasting.

66 SHARIFUL HALIM, K66, DMC


Sharifs personal notes

Anthropometric assessment:
BMI: <18.5 indicate nutritional impairment.
Upper arm circumference: Male >25cm, Female >23 cm. To see skeletal protein reserve.
Biochemical assessment:
S. Albumin: <3g/dl indicate malnutrition.
S. transferrin: Indication of visceral protein synthesis.
Lymphocyte count: <1500/mm3 suggests impaired cellular defense mechanism.

STOMACH

1. 3 common causes of pyloric stenosis.


Ans:
a. Chronic/Long standing peptic ulcer disease.
b. Gastric antral carcinoma.
c. Infantile hypertrophic pyloric stenosis.
d. Other: Adult hypertrophic pyloric stenosis, pyloric mucosal diaphragm/membrane.
2. Outline the management of a case of infantile hypertrophic pyloric stenosis.
Ans:
a. Symptoms:
Projectile non-bilious vomiting esp. after feeding.
Poor feeding.
Weakness & lethargy.
Features of shock if present: Breathlessness, hurried respiration, sweating,
alteration of consciousness level.
b. Signs:
Visible peristalsis in the epigastrium from left to right after feeding.
Palpation: Olive shaped mass is felt in the epigastrium.
Signs of dehydration.
c. Investigations:
Barium meal study: Narrowed pyloric outlet seen as String sign/Railroad tract
sign.
USG of W/A: Gold standard test. Muscle thickness >4mm, length of pyloric canal is
>17mm. (N len\\gth: 14mm)
S. electrolytes & ABG: Hypokalemic, hypochloremic metabolic alkalosis. (Extra-
paradoxical aciduria, Hyponatremia, hypocalcemia, hypomagnesemia)
d. Conservative management:
NPO.
NG suction.
IV fluids: 120-150ml/kg/day of .45% NS with 5% DA containing 20mmol of K/L.
IV antibiotics: Inj. Ceftriaxone/Ceftazidime, Inj. Metronidazole.
Urinary Foley catheterization.
e. Surgical treatment:
Fredet-Ramstedts Pyloroseromyotomy under G/A by transverse right upper
quadrant incision.
May be done under LA: Paediatric surgeon ra bole.

67 SHARIFUL HALIM, K66, DMC


Sharifs personal notes

3. A 50 Y/O man presented with an epigastric lump.


a. Write down the probable differentials.
b. What investigations will help diagnosis?
a. Ans:
Gastric carcinoma.
Gastric lymphoma.
Hepatocellular carcinoma involving the left lobe of liver.
Pancreatic pseudocyst.
Carcinoma of head of pancreas.
b. Ans:
Endoscopy of upper GIT & biopsy for histopathology.
Endoscopic ultrasound.
Barium meal study/ X-Ray.
USG of W/A.
Multi-slice CT scans of abdomen with contrast film.
PET scan.
Laparoscopy: to see peritoneal mets.
LFTs: S. bilirubin, ALT, AST, ALP.
Markers: S. Alpha Feto-Protein, CEA, CA-19-9.

4. What are the congenital anomalies of stomach & duodenum?


Ans:
Congenital hypertrophic pyloric stenosis.
Duodenal atresia.
Pyloric membrane.
Hiatus hernia.
Microgastria.
Gastric duplication.
Duodenal duplication.
5. How will you diagnose & treat a case of pyloric stenosis/GOO in middle aged man?
Ans:

DIAGNOSIS:
a. Past history: Of PUD ie burning/gnawing intermittent epigastric pain sometimes
radiating to back relieved by taking food or medication.
b. Present history:
Vomiting:
Profuse.
Non-bilious.
Often contains a mixture of digested, semi-digested & undigested
food particle.
Foul smelling.
Sometimes induced by the patients to be relieved off epigastric
discomfort.
Epigastric discomfort/gastric fullness.

68 SHARIFUL HALIM, K66, DMC


Sharifs personal notes

Wt. loss.
Constipation.
Weakness & fatigue
c. Examination findings:
G/E:
Ill looking.
Malnourished.
Anemic.
Dehydrated.
BP: Low.
Pulse: Rapid.
Local examination:
Epigastric fullness.
Visible peristalsis from left to right in the epigastric region.
Succussion splash: Patient is usually seen 4 hours after taking meal.
d. Investigations:
Endoscopy of upper GIT with endoscopic biopsy & histopathology.
Barium meal X- Ray.
USG of W/A.
CT scan.
CBC & ESR.
Other investigations for G/A fitness.
e. Treatment:
Preparation/ conservative management:
Patient preparation:
Correction of anemia.
Correction of malnutrition.
Correction of electrolyte imbalance.
Control of DM, HTN and any infection.
Urinary catheterization.
Gastric preparation:
3 days preparation.
NPO.
NG suction and lavage:
o First suction of all residual food particles.
o Then, 200ml of normal saline every 2 hourly until clear
fluid comes out (Lavage).
o Then only plain water.
IV fluids & Electrolytes: .9% NS.
Prophylactic antibiotics: Inj. cefuroxime 1gm, inj.
metronidazole 500mg.
PPI & H2 blockers.
Avoidance of NSAIDs.
Surgical:
Bilateral Truncal vagotomy with Gastrojejunostomy.
Heineke-Mukulicz pyloroplasty
If CA: Appropriate resection.

69 SHARIFUL HALIM, K66, DMC


Sharifs personal notes

6. Write down the C/F, investigations & treatment of PUD.

Ans:

C/F:

Pain: Epigastric, gnawing, often intermittent & periodic, sometimes radiating to the
back, relieved by taking food or medication.
Vomiting.
Hematemesis & melena.
Alteration in wt.
Weakness, fatigue: Due to anemia.

Investigations:

Endoscopy of upper GIT with multiple endoscopic biopsy & histopathology.


Barium meal X-Ray.
ELISA for H. pylori.
Urease breath test.
CBC with ESR.
S. amylase.

Treatment:

Medical:
H2 blocker: Ranitidine, nizatidine, famotidine.
PPI: Omeprazole, esomeprazole, pantoprazole.
Eradication therapy: Combined PPI (20mg) + Clarithromycin
(500mg) +amoxicillin (1gm) BD for 1 week.
Avoidance of NSAIDs.
Surgical:
Truncal vagotomy & gastrojejunosotmy.
Selective vagotomy with drainage procedure.
Highly selective vagotomy.
Billroth II gastrectomy with gastrojejunostomy.

7. What are the types of vagotomy? Write down the complication of vagotomy.

Ans:

Types of vagotomy:

Truncal vagotomy & drainage: Section of the main trunks of the both vagus nerves (So
hepatic and celiac branches are also cut)
Selective vagotomy & drainage: Section of nerve of Latarjet of both vagi with
preservation of celiac and hepatic branches.

70 SHARIFUL HALIM, K66, DMC


Sharifs personal notes

Highly selective vagotomy: Only branches of body and fundus are cut with preservation
of branches of antropyloric region.

Complications of vagotomy:

Per-operative: Hemorrhage, injury to surrounding structures, pneumothorax.


Later:
Early satiety: Due to loss of receptive relaxation.
Post vagotomy diarrhea: Due to quick gastric emptying.
Bilious vomiting.
Dumping: early & late. Due to rapid gastric emptying and high osmotic load in small
intestine.
Gall stone formation.

8. What are the complications of PUD?


Ans:
Acute perforation of PUD.
Hematemesis & melena.
GOO.
Iron deficiency anemia.
9. How will you manage a case of acute perforation of PUD?

Ans:

History:

H/O PUD.
Sudden, severe generalized excruciating abdominal pain.
Features of shock:
Breathlessness.
Sweating.
Palpitation.
Anxiety.
Altered consciousness level.

Examination findings:

Features of shock: Rapid thready pulse, Low BP, cold extremities, tachypnea, sunken
eye.
Pt. doesnt move at all.
Hippocratic facies: if severe shock is present.
Abdominal examination:
Inspection: Abdomen doesnt move with respiration.
Palpation: Board like rigidity & muscle guarding.
Obliteration of upper border of liver dullness.
Bowel sounds: Dimished/absent.

71 SHARIFUL HALIM, K66, DMC


Sharifs personal notes

Investigation:

th
CXR P/A view erect posture (B/L 25 ) or plain X-Ray abdomen A/P view in erect posture
with both domes of diaphragm: Shows cresentic sub-diaphragmatic free gas shadow.
S. Amylase.
CT scan.

Treatment:

Conservative:
NPO.
NG suction.
IV fluids & electrolytes.
IV broad spectrum antibiotics.
PPI: ie IV omeprazole.
IV Analgesics.
Foley catheterization: as pt. is destined for emergency surgery.
Surgical:
Laparotomy by upper midline incision &:
Thorough peritoneal toileting.
Adequate closure/repair of the perforation.
Reinforcement with omental patch.
10. Write down the causes of hematemesis. How will you manage a case of hematemesis?

Ans:

Causes of hematemesis:

Bleeding PUD.
Gastritis/Gastric erosion.
Rupture of esophageal varices.
Esophagitis.
Mallory-Weiss tear.
Gastric carcinoma.
Esophageal carcinoma.

Management plan of hematemesis:

Immediate resuscitation:

Short history about episodes and amount of blood lost.

Quick assessment of vitals:

o Pulse: Rapid and thready.

o BP: hypotension.

o RR: increased.

o Temperature: decreased

72 SHARIFUL HALIM, K66, DMC


Sharifs personal notes

o Other features of hypotension: sweating, cold clammy extremities,


confusion, disorientation.

Establishment of IV access with 2 large bore IV cannula:

o IV crystalloid: Ringer s lactate, Hartman s solution, DNS, NS is given.

o IV colloid can be given.

o Blood is collected and sent for grouping and cross matching.

o IV omeprazole.

o Grouped and cross matched blood is transfused- about 2 units.

o If patient is in shock- oxygen thru facemask is given.

o Other options: Tranexamic acid, octeotride.

o If bleeding continues- urgent upper GI endoscopy & control of bleeding


i.e. by laser photocoagulation, electrocauterization, bipolar diathermay,
injection of sclerosing agent, local inj dilute adrenaline.

General management after resuscitation:

Monitoring:

o Vitals check in regular interval.

o Catheterization and monitoring of urine output.

o For new episode of hematemesis.

Identifying the coz of hematemesis:

o Symptoms of CLD.

o Symptoms of PUD.

o If CLD and PUD are excluded, then other causes can be sought out.

o In every case, if emergent endoscopy is not done before, it should be


done now to identify the cause of hematemesis.

Treatment according to the cause.

11. What are the gastric tumors?

Ans:

73 SHARIFUL HALIM, K66, DMC


Sharifs personal notes

Benign:

Gastric polyp: Metaplastic, Inflammatory, FAP.


Adenoma.
Leiomyoma.
Neurofibroma.
Lipoma.

Malignant:

Gastric adenocarcinoma.
Gastric lymphoma.
Carcinoid.
Gastric stromal tumor: Leiomyosarcoma.

12. What are the RFs of CA stomach?

Ans:


Environmental:
Diet:
Alcohol.
Excessive salt intake.
Deficiency of anti-oxidant.
Diet with certain preservatives in food as nitrate and nitrosamine.
Cigarette smoking.
Pre-cancerous lesion:
Gastric adenoma.
Gastric polyp.
Chronic autoimmune gastritis.
Chronic gastric ulceration.
Pernicious anemia.
Infection with H. Pylori.
Partial gastrectomies e.g. bypass surgery.
Genetic factors:
Blood group: A.
Family H/O gastric cancer.
Hereditary Non polyposis colon cancer syndrome.
13. Common sites of CA stomach in Low SES countries?

Ans:

Most common site: Pylorus and antrum.


Next: Body.
Least: Fundus and cardia.

74 SHARIFUL HALIM, K66, DMC


Sharifs personal notes

14. Classify gastric carcinoma.

Ans:

A. On the basis of depth of invasion:


Early gastric carcinoma: Neoplasm that is confined within submucosa. In Japanese
classification it has 3 types:
Type I: Protruding into lumen. Involves mucosa.
Type II: Superficial. Involves mucosa.
Type III: Excavated. Shallow deeply ulcer crater is present. Involves
mucosa and submucosa.
Advanced gastric carcinoma: Neoplasm that involves below the submucosa into
the muscular wall. In Bormann classification it has 4 types:
Type I: Fungating type.
Type II: Ulcer with clear margin.
Type III: Ulcer without clear margin. With infiltration into surrounding
mucosa.
Type IV: Diffuse infiltrative carcinoma/Linitis plastica which involves all
layers with marked desmoplastic reaction.
Type III and IV are incurable.
B. Histological type (Lauren): You may forget the previous one, but this one is important.
I. Intestinal gastric CA:
Cells are in glandular structure resembling colonic adenocarcinoma.
Prognosis good.
II. Diffuse gastric CA:
Overwhelming infiltration deeply into the stomach.
Bad prognosis.
15. Describe the clinical features of CA stomach.

Ans:

Symptoms:

Due to primary tumor:


Early case:
New onset dyspepsia over 40 years.
Anorexia.
Vague post prandial abdominal heaviness.
Advanced case:
Early satiety.
Abdominal distension.
Constant Epigastric pain: Not relieved by food or H2 blockers.
Lump in Epigastric region.
Hematemesis & melena.
Wt. loss.
Due to GOO in antral or pyloric carcinoma:
Non-bilious vomiting which often contains a mixture of digested & undigested food
particles & often induced by the patient.
Constipation.

75 SHARIFUL HALIM, K66, DMC


Sharifs personal notes

Due to metastasis:
Peritoneal mets: Abdominal distension due to ascites.
Liver mets: Yellow sclera, mucus membrane, urine.
Bone mets: Bone pain.
Lung mets: Cough, hemoptysis.
Other effects:
Thrombophlebitis/Trousseau s sign.
DVT.
Acathosis nigricans.

Signs:

G/E:
Ill looking.
Malnourished.
Anemia: Due to local tumor.
Jaundice: Liver mets or L. nodes around bile duct/Porta hepatis causing obstructive
jaundice.
Dehydration: Due to vomiting & less intake of fluid.
Palpable left supraclavicular lymph node: Troisers sign.
Abdominal examination:
Palpable mass in the epigastrium: Mass is irregular, tender and hard.
If GOO: Visible peristalsis from left to right in the epigastrium & Succusion splash.
Sister Mary-Joseph nodule around umbilicus.
Ascites.
Hepatomegaly if liver mets.
DRE: Metastatic nodule in Rectal shelf of Blummer.

16. What are the investigations needed to be done in a suspected case of CA stomach?

Ans:

To diagnose primary tumor:


Endoscopy of upper GIT with biopsy & histopathology.
Barium meal X-Ray.
USG of W/A.
Multi-slice CT scans of abdomen with contrast.
Endoscopic ultrasound.
To see metastasis:
LFTs: S. bilirubin, ALT, AST.
CXR P/A view.
Radionuclide bones scan.
PET scan.

To see associated complication of gastric cancer:


CBC with ESR.
Comment on PBF.

76 SHARIFUL HALIM, K66, DMC


Sharifs personal notes

S. albumin.
To assess the G/A fitness of the patient:
Cardiac function: ECG, Echocardiogram.
Renal function: S. creatinine, S. electrolytes, BUN, Urine R/M/E.
Hemostasis: BT, PT, INR.
Liver function: S. albumin, S. bilirubin, HBsAg.
Hematologic: CBC with ESR.
Endocrine: FBS.

17. How will treat a case of gastric carcinoma?

Ans:

Treatment modalities depend on:

Type of gastric cancer.


Stage of presentation.
Age of the patient.
General fitness and associated comorbidities of the patient.
Features of inoperability.

TREATMENT:

1. Preparation/ conservative management:


Patient preparation:
Correction of anemia.
Correction of malnutrition.
Correction of electrolyte imbalance.
Control of DM, HTN and any infection.
Urinary catheterization.
Gastric preparation:
3 days preparation.
NPO.
NG suction and lavage:
o First suction of all residual food particles.
o Then, 200ml of normal saline every 2 hourly until clear fluid
comes out (Lavage).
o Then only plain water.
IV fluids & Electrolytes: .9% NS.
Prophylactic antibiotics: Inj. cefuroxime 1gm, inj.
metronidazole 500mg.

77 SHARIFUL HALIM, K66, DMC


Sharifs personal notes

PPI & H2 blockers.


Avoidance of NSAIDs.

2. Surgery:
Curative: In Early gastric CA, Stage I.
I. IF antral carcinoma: Subtotal gastrectomy with enblock removal of
omentum with gastro-jejunostomy: Includes entire tumor and 7-8cm of
healthy margin in unstressed stomach to ensure adequate macroscopic
and microscopic excision.
II. IF body CA: Total/Radical gastrectomy with reconstruction esophago-
jejunostomy with jejuno-jejunostomy.
Resection of:
o Stomach.
o Spleen and hilar LN.
o Tail of pancreas.
o Lesser omentum.
o Greater omentum.
o LN that drains the stomach and celiac nodes.
III. If cardia CA: Esophago-gastrectomy plus splenectomy with intra-thoracic
esophagnojejunostomy. Common in western countries.
Palliative:
I. When there are signs of inoperability:
Fixity of tumor.
Gross local involvement of LN.
Presence of secondaries in liver, lung, bone.
Peritoneal seedling: Ascites.
Japanese N4 tier nodal metastasis.
Supra Clavicular and Sister Joseph nodule.
II. Palliative resection, bypass or exclusion operation is done.
Radiotherapy: Has a role in painful bony metastasis.
Chemotherapy.

18. How will you prepare a patient for partial gastrectomy suspected to be suffering from early
gastric carcinoma?

Ans:

Pre-operative investigations:

To diagnose primary tumor:

Endoscopy of upper GIT with biopsy & histopathology.

Barium meal X-Ray.

USG of W/A.

78 SHARIFUL HALIM, K66, DMC


Sharifs personal notes

Multi-slice CT scan of abdomen with contrast.

Endoscopic ultrasound.

To see metastasis:

LFTs: S. bilirubin, ALT, AST.

CXR P/A view.

Radionuclide bone scan.

PET scan.

To see associated complication of gastric cancer:

CBC with ESR.

Comment on PBF.

S. albumin.

To assess the G/A fitness of the patient:

Cardiac function: ECG, Echocardiogram.

Renal function: S. creatinine, S. electrolytes, BUN, Urine R/M/E.

Hemostasis: BT, PT, INR.

Liver function: S. albumin, S. bilirubin, HBsAg.

Hematologic: CBC with ESR.

Endocrine: FBS.

Pre-operative management:

Patient preparation:

o Correction of anemia.
o Correction of malnutrition.
o Correction of electrolyte imbalance.
o Control of DM, HTN and any infection.
o Urinary catheterization.

Gastric preparation:

3 days preparation.
NPO.
NG suction and lavage:

79 SHARIFUL HALIM, K66, DMC


Sharifs personal notes

o First suction of all residual food particles.


o Then, 200ml of normal saline every 2 hourly until clear fluid
comes out (Lavage).
o Then only plain water.
IV fluids & Electrolytes: .9% NS.
Prophylactic antibiotics: Inj. cefuroxime 1gm, inj.
metronidazole 500mg.
PPI & H2 blockers.
Avoidance of NSAIDs.

SMALL and LARGE INTESTINE

INTESTINAL OBSTRUCTION:

Q. Classify intestinal obstruction.

Ans:

According to site: Small BO, Large BO


According to duration: Acute, sub-acute, chronic, acute on chronic.
According to state of peristalsis: Dynamic, adynamic.

Q. What are the common causes of intestinal obstruction?

Ans:

Dynamic:
Extramural: Bands and adhesions, obstructed inguinal hernia, volvulus,
intussusception.
Mural: Benign and malignant strictures.
Luminal: Fecal impaction, worm bolus, gallstone.
Adynamic: Paralytic ileus, mesenteric vascular occlusion.

Q. What are the causes of large gut obstruction?

Ans:

Luminal: Fecal impaction.


Mural: Benign and malignant stricture.
Extramural: Volvulus.

Q. What are the causes of neonatal intestinal obstruction?

Ans:

Duodenal, ileal or jejunal atresia.


Meconium ileus.
Neonatal volvulus.
Hirschprung disease.

80 SHARIFUL HALIM, K66, DMC


Sharifs personal notes

Imperforate anus.

Q. How will you manage a case of intestinal obstruction?

Ans:

Symptoms:

Intermittent colicky abdominal pain.


Vomiting.
Abdominal distention.
Constipation:

Signs:

On G/E: Dehydration, tachycardia, low BP, tachypnea, alteration of consciousness level.


P/A/E:
Inspection: Abdominal distention, visible peristalsis.
Palpation: In malignant coz of obstruction a mass can be felt.
Auscultation: High pitched tinkles, peristaltic rushes.

Investigations:

Plain X-Ray of abdomen A/P view in erect posture.


S. electrolytes.
BUN.
S. creatinine.
ABG.
USG/CT: esp. if malignancy is suspected.

Treatment:

General management:
NPO.
NG suction.
IV fluid.
Broad spectrum antibiotics.
Analgesics.
Urinary catheterization.
Specific management: According to cause of obstruction, when conservative management
fails or the patient deteriorates.

Q. Write down the differences between small and large intestinal obstruction?

Ans:

Points Small intestinal obstruction Large intestinal obstruction

81 SHARIFUL HALIM, K66, DMC


Sharifs personal notes

Colicky abdominal pain Early and severe Less severe


Vomiting Early, profuse and contains Late and feculent
mainly undigested food
particles and intestinal
secretions
Abdominal distention Late, mild and central Early, pronounced and
peripheral.
Absolute constipation Late Early
Dehydration Rapid and marked Delayed and mild.
Plain X-Ray abdomen Central air fluid level and jejunal Peripheral air fluid lvel.
volvulae conniventes.

Q. What are the clinical features of strangulation?

Ans:

Symptoms:

The intermittent colicky pain becomes constant and severe.


Vomiting, constipation persists.
Fever.

Signs:

G/E: Pt. is very ill looking, raised temp., tachycardia, low BP, tachypnea.
P/A/E: Abdomen is tender with muscle guard and rigidity.

Q. How will you differentiate viable and non-viable gut?

Ans:

Points Viable gut Non-viable gut


Appearance Shiny Lusterless.
Color Pink Dark.
Peristalsis May be seen Absent
Consistency Firm Flabby and friable
Mesenteric artery Visible pulsation may be seen No visible pulsation.

INTUSSUSCEPTION:

Q. How will you manage a case of intussusception?

Ans:

Patient particulars: Children in their weaning period.


Symptoms:
Severe colicky abdominal pain manifested as recurrent screaming and drawing up
of legs.
Vomiting.
Passage of Red CURRANT gelly stool.

82 SHARIFUL HALIM, K66, DMC


Sharifs personal notes

Signs:
G/E: Dehydration, tachycardia, low BP.
P/A/E:
Inspection: Mild abdominal distention.
Palpation:
Sausage shaped mass.
Emptiness in right iliac fossa.
DRE: Blood stained mucus.
Investigations:
Plain X-Ray of abdomen.
USG of abdomen.
Barium enema: Claw sign.
Treatment:
General:
NPO.
NG suction.
IV fluid.
Antibiotics.
Analgesics.
Urinary catheterization.
Specific:
Hydrostatic reduction.
Laparotomy:
Reduction by Copes method.
If bowel is gangrenous: Resection of gangrenous portion and
end to end anastomosis.

VOLVULUS:

Q. What is volvulus?

Ans:

Segmental rotation of a portion of gut about its mesentery.

Q. How will you manage a case of sigmoid volvulus?

Ans:

Symptoms:

Constipation.
Abdominal pain.
Abdominal distention.
Feculent vomiting.

Sign:

83 SHARIFUL HALIM, K66, DMC


Sharifs personal notes

G/E: Dehydration, tachycardia, low BP, tachypnea.


P/A/E:
Abdominal distention and tenderness.

Investigations:

Plain X-Ray of abdomen A/P view in erect posture.


Barium enema.

Treatment:

General:
NPO.
NG suction.
IV fluids.
Broad spectrum antibiotics.
Analgesics.
Urinary catheterization.
Specific:
Passage of flatus tube.
If deflation occurs, laparotomy can be delayed for 3 to5 days.
No deflation- then immediate laparotomy:
Manual untwisting or
Resection of the redundant or gangrenous part of the colon followed by
end to end anastomosis.

HIRSCHPRUNG DISEASE:

Q. What is Hirschprung disease?

Ans:

It is a congenital disorder characterized by absence of parasympathetic ganglia in the myenteric


plexus of usually the distal hindgut.

Pathology: Failure of migration of neural crest cells from cranio-caudal direction.

Site: Anorectal or recto-sigmoid junction, any part of colon.

Q. How will you manage a case of Hirschprung disease?

Ans:

Clinical features:

Delayed passage of meconium.


Constipation.
Abdominal distention.
Vomiting.

84 SHARIFUL HALIM, K66, DMC


Sharifs personal notes

Abdominal pain.
Failure to thrive.

Investigations:

Rectal biopsy.
Barium enema.
Plain x-ray.

Treatment:

General: NPO, NG suction, IV fluids, broad spectrum antibiotics, analgesics, urinary


catheterization.
Specific:
Initial colostomy.
Rectal saline washout: For evacuation of intestine.
Operation:
Resection of the aganglionic segment and pull thru of ganglionic portion to
anus.

Tumors of the small intestine:

Benign:

Adenoma.
Polyp.
Carcinoid tumor.

Malignant:

Primary: Adenocarcinoma, carcinoid tumor, lymphoma.


Secondary: From breast, lung, melanoma, colon, kidney.

Tumors of large intestine:

Benign:

Polyp: Inflammatory, Metaplastic, hamartomatous (Peutz-Jegher, Juvenile), Neoplastic


(Adenoma).
Lipoma.
Neurofibroma.

Malignant:

Primary: Adenocarcinoma.
Secondary.

85 SHARIFUL HALIM, K66, DMC


Sharifs personal notes

CA colon

CA right colon is more common in women.


CA rectum is more common in women.
Distribution: CA rectum- 38%, CA sigmoid colon-21%, CA Cecum-12%.
Macroscopic pathology:
Annular: Produces obstructive symptoms.
Tubular, ulcerative, cauliflower: Produce bleeding.
Tubular variety takes the form of fungating mass & has the best prognosis.
RFs/Causes of CA colon:
Benign adenoma: esp. >2cm- has 50% chance of malignancy.
Inherited genetic factor: FAP, HNPCC.
Familial cancer syndrome: Lynch syndrome.
Diet:
Increased intake of saturated fat.
Increased calorie intake.
Decreased dietary calcium.
Decreased intake of dietary fiber.
Decreased intake of vitamins ACE.
Bile acid: Carcinogenic; patients after cholecystectomy has increased risk of CA
colon.
Bacteria: E. coli, H. pylori.
HPV: 16,18.
Predisposing situation/condition/Precancerous lesion:
IBD: UC, Crohn s.
Colorectal polyp.

Spread of CA colon:
Local: Circumferential.
Lymphatics: Via extramural lymphatics.
Hematogenous spread: Liver- Portal vein; Lung, bone, brain= By lumbar and
vertebral vein.
Transperitoneal: pelvic peritoneum, pelvic Cul-de-sac- Blummer shelf.
Symptoms:
Right colon/Ascending colon/Cecum:
Weakness, fatigue= Anemia.
Melena.
Vague right abdominal discomfort.
Wt. loss.
Palpable mass in RIF or right abdomen.
Transverse colon:
Mistaken as CA stomach because of location with anemia.
Left colon/Descending colon:
Pain: In the left side of abdomen, episodic colic or constant. May be
referred to suprapubic area. Occurs due to intestinal obstruction.
Alteration of bowel habit: Alternating constipation and increased
frequency of defecation.
Palpable lump in the left side of abdomen.

86 SHARIFUL HALIM, K66, DMC


Sharifs personal notes

Bleeding per rectum.


Mucus often passed with small blood clots.
Sigmoid colon:
Per rectal bleeding.
Alteration in bowel habit.
Colicky abdominal pain.
Tenesmus.
Passage of mucus & blood esp. in the early morning.
Things to look for in G/E:
Anemia, Jaundice, dehydration, edema, SC LN, edema, ascites.
Things to look for in Abdomen examination:
Ascites, enlarged liver.
Examination of the mass: Size, site, shape, number, surface, consistency, fixity.
DRE: Look for rectal shelf of Blummer.
Investigations:
For diagnosis:
Colonoscopy: To detect primary cancer, exact site & obtain biopsy.
Double contrast barium enema:
o Constant/irregular filling defect.
o Apple core deformity.
o Shouldering effect.
For staging of the disease:
USG of W/A: Liver mets, ascites, lymph nodes.
CT scan of W/A
CXR.
CT chest.
LFTs.
Prognostic purpose: S. CEA, N=<10.
Routine investigation for G/A and surgical fitness:
CBC with ESR.
Urine R/M/E.
RBS.
Blood urea, s. creatinine, electrolytes.
ECG.
BT, PT.
HBsAg.
DDx:
Lump in RIF: Appendicular lump, ileocecal TB, Crohns disease.
Left colon: Diverticular disease, IBD.

Staging of CA colon:
Modified Dukes classification: Almost like RECTAL CA.
A= Tumor confined to bowel wall.
B= Thru the bowel wall but not involving the free peritoneal surface.
C1= Regional LN.
C2= LN at the point of origin/division of blood vessels.

87 SHARIFUL HALIM, K66, DMC


Sharifs personal notes

D=Distant metastasis.
TNM staging:
T1= Within Submucosa.
T2= into but no thru Muscularis propria.
T3= thru the Muscularis propria but not thru the serosa.
T4= Breached/thru serosa.
N0= No LN.
N1= 1-3 Pericolic LN.
N2= 4/> Pericolic LN.
N3= Nodes along the supplying vascular trunk.
M0=No Mets.
M1= Mets Yes.

Management:
General preparation of patient:
Correction of nutritional impairment, anemia, fluid and electrolyte
imbalances, vitamin supplementation.
Gut preparation: Fleet enema.
Antibiotic prophylaxis:
rd
o 3 generation cephalosporin 1g inj. + Metro 500mg IV given
before induction & another dose post-operatively.
Surgery:
CA cecum and ascending colon:
o Right hemicolectimy with end to end ileotransverse
anastomosis.
o Resection of: Last 30cm of ileum and its mesentery, appendix,
rd
cecum, ascending colon + proximal 1/3 of transverse colon
with mesocolon.
o Vessel and LN: Ileocolic, right colic.

CA transverse colon:
o Transverse colectomy along with mesocolon and end to end
anastomosis between ascending and descending colon.
CA descending colon:
o Left hemicolectomy with anastomosis between transverse
colon and rectum.
rd
o Resection of left 1/3 of TC and TMC, descending and sigmoid
rd
colon, upper 3 of rectum, LN, blood vessels accompanying it.
CA sigmoid colon/pelvic colon:
o Sigmoid colectomy and end to end anastomosis between
descending colon and rectum.
o Dissection of local LN and vessels.
When growth is inoperable:
o Anastomosis between proximal and distal intestine.

88 SHARIFUL HALIM, K66, DMC


Sharifs personal notes

o Colostomy: Splenic flexure- Transverse colostomy, Pelvic colon-


Sigmoid colostomy

Adjuvant therapy:
Chemotherapy: 5FU, Folinic acid.

Q. Write a short note on colostomy?

Ans:

Definition: Artificial opening made in the large bowel to divert feces and flatus to the exterior.

Type:

According to duration: Temporary, permanent.


According to site: Sigmoid, transverse colostomy.
According to procedure: End, loop colostomy.

Indication:

De-function anastomosis.
Facilitate repair of high fistula in ano.
In case of penetrating injury to rectum and distal colon.
Malignancy in rectum and anal canal:

Complications:

Colostomy diarrhea.
Excoriation, irritation.
Infection.
Stenosis.
Retraction.
Prolapse.
Bleeding.

APPENDIX:

1. What are the differential diagnoses of a lump in the right or left iliac fossa?

Ans:

Appendicular lump.
Appendicular abscess.
Ileocecal TB.
CA colon.
Crohn s disease.
2. Write down the management of a case of acute appendicitis.

89 SHARIFUL HALIM, K66, DMC


Sharifs personal notes

Ans:

Patient particulars: Young male at their twenties.


Symptoms:
Vague peri-umbilical pain.
Constant pain in the right iliac fossa.
Nausea and vomiting.
Mild fever.
Signs:
G/E: Ill looking, dehydration, tachycardia, raised temperature, low BP.
P/A/E:
Pointing sign.
McBurney s point tenderness.
Rebound tenderness.
Rovsing s sign.
Psoas sign.
Obturator sign.
Investigations:
Blood for TC and DC of WBC (Or CBC with ESR)
USG of W/A with special attention to Ileocolic segment.
Urine R/M/E and C/S.
Treatment:
General:
NPO, NG suction, IV fluid, Broad spectrum antibiotics, analgesics, urinary
catheterization.
Specific: Emergency appendectomy/Appendicectomy.

Q. What are the important DDx acute appendicitis in a young male and an young female?

Ans:

Young male:

Regional enteritis.
UTI/ ureteric colic.
Acute pancreatitis.
Perforation of PUD.

Young female:

Twisted ovarian cyst.


Ruptured ectopic pregnancy.
Salphangitis/PID.
Endometriosis.

Q. What are the fates of acute appendicitis?

Ans:

90 SHARIFUL HALIM, K66, DMC


Sharifs personal notes

Resolution.
Appendicular lump.
Appendicular abscess.
Gangrenous appendix.
Burst appendix.
Peritonitis.

Q. How will you manage a case of appendicular lump?

Ans:

History: OF acute appendicitis about 2-3 days back.


Signs:
rd
Tender mass in the RIF on 3 day with Rigidity and muscle guard over the mass.
th
Mass become well circumscribed in the 5 day.
Treatment:
General: NPO, NG suction, IV fluid, Broad spectrum antibiotics, analgesics, urinary
catheterization.
Specific: Interval appendectomy 6 weeks later after complete recovery.

Q. What are the indications of appedicectomy?

Ans:

Emergency: Acute or recurrent appendicitis.


Interval appedicectomy: Appendicular lump and abscess.

RECTUM AND ANAL CANAL

1. Write 5 most common cause of bleeding per rectum.


Ans:
a. Hemorrhoids.
b. Rectal polyp.
c. Rectal carcinoma.
d. CA colon.
e. Bleeding disorder.
2. Write 4 most common cause of bleeding per rectum in childrens.
Ans:
a. Rectal polyp.
b. Intussusception.
c. Gastroenteritis.
d. Bleeding disorder.

91 SHARIFUL HALIM, K66, DMC


Sharifs personal notes

e. Rectal prolapse.
3. Write most important causes of painful anus.
Ans:
a. Anal fissure.
b. Abscess.
c. External/ Prolapsed hemorrhoids.
d. Infected fistula in ano.
e. Invasive CA rectum.
4. Name some common anorectal diseases.
Ans:
a. Hemorrhoids.
b. Anal fissure.
c. Perianal abscess.
d. Fistula in ano.
e. Rectal polyp.
f. Rectal Carcinoma.

ARM:

Q. Classify Anorectal malformations.

Ans:

Imperforated anus:
Low abnormalities: Covered, ectopic, stenosed anus, membranous stenosis.
High abnormalities: Anorectal agenesis, rectal atresia.
Sacro-coccygeal teratoma.
Post-anal dermoid.
Post-anal dimple.

Q. How will you interpret an invertogram?

Ans:

Distance between the site of metal detector and blind end of rectum:
>2.5 cm= High abnormality.
<2.5 cm= Low abnormality.

HEMORRHOIDs.

Risk factors:
Habitual constipation.
Straining.
Family history.
Bad defecatory habit.
Old age.

92 SHARIFUL HALIM, K66, DMC


Sharifs personal notes

Pregnancy.
Types/classification:
a. External or internal or interno-external.
st th
b. Internal hemorrhoid: Classified according to protrusion: 1 - 4 degree.
st
1 degree: Only bleeds, no protusion.
nd
2 degree: Protrudes but reduces spontaneously.
rd
3 degree: Protrudes but can be reduced manually.
th
4 degree: Irreducible.
c. According to etiology: 1ry- idiopathic, secondary due to TB, Carcinoma, Crohn s, UC.
d. According to position: 1ry position- 3,7,11 oclock; secondary position: In between these
are secondary positions. Position is evaluated in the lithotomy position.
Classic hemorrhoidal bleeding:
a. Fresh blood passed.
b. Not mixed with stool or mucus.
Best way of diagnosis: Proctoscopy.
Next best investigation: Colonoscopy to evaluate other causes of bleeding per rectum.
Treatment:
Depends on symptoms, degree, position and presentation.
Early stage- medical: Diet-high fiber diet, increased fluid, defecatory advice,
laxatives, roughage, analgesics, antibiotics, hip bath.
If doesnt improve:
Anti-hemorrhoid drugs: e.g. Normanal.
Anal canal softener.
Minimal invasive procedure:
Injection sclerotherapy: Phenol in almond oil.
Barrel banding.
Operative procedures:
Hemorrhoidectomy: Commonly done, hemorrhoids are cut and mucosa
kept open.
Stapling method / Procedure for prolapsed hemorrhoids
(PPH)/Longlow operation

Q. Write down the complications of hemorrhoids?

Ans:

Thrombosis of prolapsed hemorrhoid.


Rupture and excessive bleeding.
Gangrene.
Ulceration.
Infection.

93 SHARIFUL HALIM, K66, DMC


Sharifs personal notes

ANAL FISSURE:

Definition: Longitudinal split in anoderm of distal anal canal.


Severe TEARING/excruciating/burning pain during each defecation.
Few drops of blood may be present as streaks outside the stool.
Dx:
If severe pain: Direct inspection of fissure by aparting anal canal.
EUA: Examination under anaesthesia.
Sentinel piles/guarding piles: May be found at the lower end of the fissure.
Treatment:
Conservative management: Like hemorrhoids.
If severe pain:
Narcotic analgesics may be given.
Local anesthetic application:
2% Jasocaine mixed with oil based cinchocin ie Erean, newparcanal,
anastat.
If no improvement: Surgery done.
Fissurectomy with lateral internal anal sphincterectomy.

FISTULA IN ANO:

Definition: Fistula that connect Anorectal lumen with exterior.


Classification:
According to sphincter/Parks: intersphincteric, transphincteric (high/Low),
extrasphincteric, suprasphincteric.
Another classification: High level- Above pubic crest, low level- below pubic crest.
High level is difficult to approach from perineum.
Differentiation between high & low variety: FISTULOGRAM. * Next Proctoscopy and
Sigmoidoscopy.
Fistulogram also helps in:
Differentiating fistula & sinus (Blind tract).
Branching pattern evaluation.
Assessment of sphincter involvement.
Clinical examination in fistula: DRE- may find external opening, puckered skin
tag/excoriation of perianal skin, fecal soling, internal opening, abscess, fistula as a
complication of CA rectum & Proctoscopy.
Treatment:
Always surgical:
Low variety: Removal of the tract by Fistulectomy/ fistulotomy in
sphincter involvement.
Here no suturing is done & spontaneous healing occurs.
In high variety of fistula in ano SETON procedure is done where lower
half is cut & a non-absorbable suture (Prolene, silk, wire) is kept at

94 SHARIFUL HALIM, K66, DMC


Sharifs personal notes

upper half which causes foreign body reaction & consequent closure of
the tract.

PERIANAL ABSCESS:

More common due to:


Dependent part.
Less vascular.
More fat.
More source of infectious agents.
Variety:
Submucosal.
Subcutaneous.
Ischiorectal.
Pelvic.
Symptoms:
Local: Boggy feeling, severe pain.
Systemic: Fever, anorexia, malaise.
Treatment:
Incision & drainage.
No antibiotic before draining.
Wide opening of abscess by cruciate incision- Excision of triangular flaps- rhomboid
shape- free drainage of pus and spontaneous healing.
Pus is sent for C/S.
Tissue is sent for histopathology: to see TB, Chron s, UC.

Rectal polyp:

If single: Polypectomy.
Multiple: excision/resection of the involved bowel.

CA RECTUM
Very common.
Average age: 50-70 years; but can occur at any age after 20 years.
Premalignant/predisposing conditions:
FAP.
Adenoma.
UC.
Dukes staging:
Modified Duke s classification:
A= Tumor confined to rectal wall.

95 SHARIFUL HALIM, K66, DMC


Sharifs personal notes

B= thru the rectal wall (or outside) but no regional LN involved.


C1= Regional LN- Local para-rectal.
C2= LN up to the point of origin/division of blood vessels.
D=Distant metastasis.
TNM staging:
T1= Within Submucosa.
T2= into but no thru Muscularis propria.
T3= thru the Muscularis propria but not thru the serosa.
T4= Breached/thru serosa.
N0= No LN.
N1= 1-3 Peri-rectal LN.
N2= 4/> Peri-rectal LN.
N3= Nodes along the supplying vascular trunk.
M0=No Mets.
M1= Mets Yes.

Symptoms:
Bleeding per rectum: At the end of defecation; may or may not be mixed with
stool. Bright red or coffee ground.
Sense of incomplete defecation/ TENESMUS: Painful straining to empty bowel
without resultant evacuation.
Early morning Spurious diarrhea: It is a false diarrhea in which patient try to empty
the rectum several times a day but passes only liquid stool.
Alternating constipation and diarrhea (esp. Early morning bloody diarrhea.)
Rectal pain: Due to
Intestinal obstruction.
Invading sphincter muscle.
Invade sacral plexus.
Invasion of prostate and bladder.
Features of metastasis: Hepatomegaly, jaundice, ascites.
General features: Wt. loss, 3A= Anemia, anorexia, asthenia.
Physical examination:
G/E: Wt. loss, anemia, jaundice, dehydration, ascites, SC LN.
Abdominal examination: Ascites, hepatomegaly.
DRE: To see lower and middle third and upper third can be tipped with finger.
Growth in the rectum: Fixed, surface irregular, Margin irregular, hard in
consistency.
On withdrawing the finger: Mucus and blood.
Proctoscopic examination: If possible biopsy done.
Investigations:
For diagnosis:
Sigmoidoscopy and biopsy/Proctoscopy and biopsy.
Colonoscopy: To exclude synchronous lesion.
Barium enema: if colonoscopy is not performed:
o Irregular filling defect.
o Apple core appearance.
o Upper and lower shoulder effect.

96 SHARIFUL HALIM, K66, DMC


Sharifs personal notes

For staging:
USG of W/A.
CT scan of abdomen: To detection lesion, see invasion.
CXR.
LFTs.
Bone scan.
For prognostic purpose:
S. CEA: Presence of increased CEA 6 weeks after surgery indicates
residual disease, Rising CEA indicates recurrence.
Routine investigations for G/A fitness and operability.
Treatment:
Pre-operative preparation:
Counseling of the patient about treatment options and probable
complications. Stoma, lifestyle, professional advice.
Correction of anemia, nutritional imbalance, fluid and electrolyte
imbalance.
Tt. Of any comorbid condition.
Mechanical bowel preparation.
Prophylactic antibiotics.
Insertion of urethral catheter.
Surgical treatment options:
Curative operation:
rd
o Tumor involving upper 2/3 of rectum: Anterior resection; can
be normal which is 8 cm from anal verge, low anterior 6cm and
ultra-low anterior 4 cm from anal verge
rd
o Tumor involving the lower 1/3 of rectum: SCAPR=
Synchronized combined Abdomino-Perineal Resection.
Palliative operation:
o Hartmans operation: Old feeble patient unfit for surgery.
Rectum excised including tumor and permanent colostomy
done.
o Palliative colostomy.
Chemotherapy:
5FU+ Folinic acid.
Radiotherapy: External, intra-cavitary or interstitial.
Immunotherapy: Levamesole, cetuximab, bevacizumab.

Q. Which structures are removed in APR/SCAPR?

Ans:

Parts removed:
Part of pelvic colon and mesocolon.
Rectum with mesorectum.
Anus and surrounding skin.
Levator ani and related fascia.
Ischiorectal fat.

97 SHARIFUL HALIM, K66, DMC


Sharifs personal notes

Blood vessels sacrificed:


Inferior mesenteric artery distal to left colic artery.

Q. What is CEA? Name some conditions where CEA is elevated? Write down the interpretation of
different blood levels of CEA.

Ans:

CEA: CarcinoEmbryonic antigen.

It is a complex glycoprotein synthesized by tumor cell and embryonic epithelium.

Elevated in:

CA of colon, rectum, pancreas, gastric, breast, lungs.


Alcoholic liver cirrhosis.

Interpretation:

Normal: 0-4.
Significant: >10 ng/ml.
Residual disease: Elevated 6 weeks after therapy.
Recurrence: Rising CEA.
More CEA elevation with anaplastic varieties.

HEPATOBILIARY SYSTEM

ACUTE CHOLECYSTITIS:

Q. How will you manage a case of acute Cholecystitis?

Ans:

Symptoms:

Pain in the right upper abdomen which may refer to right scapula.
Nausea and vomiting.
Fever.

Signs:

On G/E:
Pt. is ill looking, dehydration, tachycardia, low BP, tachypnea, increased
temperature.
P/A/E:

98 SHARIFUL HALIM, K66, DMC


Sharifs personal notes

Tenderness, muscle guard and rigidity in right upper abdomen.


MURPHYs sign is positive.

Investigations:

USG of W/A with special attention to hepatobiliary system.


LFTs: S. bilirubin, AST, ALT, ALP, PT.
CBC with ESR.

Treatment:

General/conservative: NPO, NG suction, IV fluid, broad spectrum antibiotics, analgesics,


urinary catheterization.
Specific:
If pt. doesnt improve or deteriorates: Emergency cholecystectomy.
If pt. respond to conservative tt. Elective cholecystectomy 6 weeks after recovery.

Q. Write down the steps of laparoscopic cholecystectomy.

Ans:

First: Pt. lies supine.


Induction of G/A.
Painting and draping of abdomen.
Establishment of pneumo-peritoneum.
Initial port: Open sub-umbilical cut down.
2 other port: One in sub-xiphoid and another in right subcostal area.
Pt. is now placed in reverse Trendelenberg position.
Confirmation of anatomy of GB and Biliary tree.
Clipping and division of cystic duct and cystic artery.
GB removed by sharp dissection.

Q. What are the port related complications of lapchol?

Ans:

Hemorrhage and hematoma.


Injury to surrounding structures.
Infection.
Seedling of cancer cells in abdominal wall.
Abdomino-peritoneal adhesion.

Q. Outline the management of a case of empyema of GB?

Ans:

Symptoms:

Pain in the right upper abdomen which may refer to right scapula.
Nausea and vomiting.

99 SHARIFUL HALIM, K66, DMC


Sharifs personal notes

Swinging Fever.

Signs:

On G/E:
Pt. is ill looking, dehydration, tachycardia, low BP, tachypnea, increased
temperature.
P/A/E:
Tenderness, muscle guard and rigidity in right upper abdomen.
Tender lump in right upper abdomen.

Investigations:

USG of W/A with special attention to hepatobiliary system.


LFTs: S. bilirubin, AST, ALT, ALP, PT.
CBC with ESR.

Treatment:

General/conservative: NPO, NG suction, IV fluid, broad spectrum antibiotics, analgesics,


urinary catheterization.
Specific:
If pt. doesnt improve or deteriorates: Emergency cholecystectomy.
If pt. respond to conservative tt. Elective cholecystectomy 6 weeks after recovery.

Q. Write down the difference between Mucocele and empyema of GB.

Ans:

Traits Mucocele Empyema.


Definition Enlargement of GB due to Enlargement of GB due to
accumulation of clear mucinous accumulation of purulent
secretion resulting from secretion.
obstruction of cystic duct.
Pain - +
Fever - +
Vomiting - +
Palpation Non-tender lump in the right Tender lump in the right upper
upper abdomen abdomen.

GALL STONES

Q. What are the causes/ RFs of Gall stones?

Ans:

5F: Female, fatty, forty, fertile, fair.


Infection.
Abnormal function of GB: Abnormality of absorption, secretion and emptying.

100 SHARIFUL HALIM, K66, DMC


Sharifs personal notes

Hemolytic anemia.

Q. What are the types of gall stones?

3 common types:

a. Cholesterol stones: single, white.


b. Pigment stones: multiple, black.
c. Mixed stones: multiple, brown.

Q. What are the complications of gall stone disease?

Ans:

In GB: Acute Cholecystitis, chronic Cholecystitis, Mucocele, empyema, perforation, gangrene,


cancer.
In Bile duct: Choledocolithiasis, cholangitis, acute pancreatitis.
Liver and other: hepatic abscess, septicemia.
Intestinal obstruction.

BILIARY SYSTEM:

Q. What are the indications of exploration of common bile duct?

Ans:

USG evidence of stone in CBD.


Palpable stone.
Dilation of CBD.
Failure of stone extraction by ERCP prior to cholecystectomy.
Recurrent jaundice.
Raised ALP.

Q. A 40 Y/O lady presented with abdominal pain, fever and jaundice?

1. What are the probable DDx?

2. What are the investigations needed to be done to differentiate them?

Ans:

1. DDx:
Acute Cholecystitis with choledocolithiasis.
Cholangitis with choledocolithiasis.
Acute viral hepatitis.

2. Investigations:
LFTs: S. bilirubin, ALT, AST, viral markers, PT, ALP.
USG of HBS.

101 SHARIFUL HALIM, K66, DMC


Sharifs personal notes

CBC with ESR.


ERCP.

Q. How will you manage a T-tube?

Ans:

T tube is attached with a saline bag.


Daily observation of bile flow: Quantity, color, leaking, smell.
Progressive clamping:
th
7 POD= 4hours.
th
8 POD= 12 hours.
th
9 POD= 24 hours.
Observation of the patient of the patient during clamping for:
Pain.
Fever.
Jaundice.
Itching.
Oozing.

th
11 POD: T- tube cholangiography= See stricture and stone.

th th
Removal of T-tube on 12 or 13 POD.

Jaundice:

Hemolytic:
Unconjugated bilirubin increased in blood.
Normal or dark colored stool
Normal urine.
Pallor due to anemia.
Splenomegaly.
Investigations:
S. bilirubin usually <6mg/dl.
Hepatic/hepatocellular:
Viral hepatitis, acute alcoholic hepatitis, chronic active hepatitis, liver cirrhosis.
Mechanism: Failure of uptake of unconjugated bilirubin, failure of transport of
conjugated bilirubin into canaliculi.
Investigations:
S. bilirubin: both conjugated and unconjugated are markedly raised.
SGPT: Grossly elevated.
ALP: Slightly raised.
PT: Raised.
Obstructive/Cholestatic jaundice:
Causes:
Lumen:
o Gall stone.
o Round worm.
Wall:

102 SHARIFUL HALIM, K66, DMC


Sharifs personal notes

o Congenital atresia.
o Traumatic stricture.
o Cholangitis- After operation of biliary tree.
o Tumor of the bile duct.
Outside the wall:
o CA of head of pancreas.
o CA of ampulla of vater.
o Enlarged LN at porta hepatis due to metastasis.
S. bilirubin: Conjugated bilirubin.
ALP: Markedly raised.
GGT, 5-Nucleotidase: 5 Nucleotidase is most reliable because it is not
elevated/influenced by bone disease.
PT: Raised.

Choledocolithiasis

Types:
Secondary stone: 60% mainly cholesterol stone from GB.
Primary stone: Pigment stone 40%. Forms due to:
Stasis in biliary tree (stricture due to trauma or tumor), recurrent
cholangitis.
Biliary tumor causes huge dilation of CBD about 2-3 cm proximal to
obstruction.
Less dilation due to stricture or Choledocolithiasis.
Clinical features:
Pain: Features of biliary colic.
o Recurrent.
o Sudden, severe colicky.
o In the right hypochondrium.
o Radiate to inferior angle of scapula or tip of right shoulder.
o Aggravated by taking fatty food.
Nausea and vomiting.
Jaundice: Commonly intermittent, may be persistent if complete
prolonged obstruction
Fever: Intermittent. May have chills with rigor if cholangitis develops.
In G/E:
Pt. is toxic, ill looking and unwell.
Jaundice.
Dehydration.

103 SHARIFUL HALIM, K66, DMC


Sharifs personal notes

Scratch mark all over the body.


Increased temperature.
In abdominal examination:
Tenderness in right hypochondriac region.
GB is not palpable.
Investigations:
LFTs:
o S. bilirubin: Conjugated, raised usually <10mg/dl.
o ALP: Markedly increased.
o SGPT, PT: Increased.
o S. albumin: Decreased.
USG of hepatobiliary system and pancreas:
o Demonstrate stone in bile duct with or without stone in GB
(Bright echogenic structures in the lumen of GB casting
posterior acoustic shadows).
o Dilation of bile duct.
o Any pathology of pancreas.
Complications of Choledocolithiasis:
Prolonged obstruction: Cholangitis, liver abscess, septicemia.
Acute pancreatitis.
Hemorrhage or hemobilia.
Management:
Pre-operative management:
Nutritional improvement: High CHO diet to improve glycogen
storage.
Correction of anemia.
Correction of fluid and electrolyte imbalance.
Correction of coagulopathy: Administration of vitamin K or
FFP.
Urinary catheterization.
rd
Antibiotic prophylaxis: 3 generation cephalosporin with
metronidazole.
Bowel preparation: Lactulose to urea splitting organism.
IV 10% glucose about 12-24 hours before operation.
Operative management:
Minimal invasive technique: Endoscopic sphincterotomy with
stone extraction by Dormia Basket followed by LapChol.

104 SHARIFUL HALIM, K66, DMC


Sharifs personal notes

Can t be done if:


o Stone >2cm.
o Stenosis or stricture of CBD proximal to sphincter.
Surgery: Open cholecystectomy with choledocolithotomy with
T-tube placement.

Q. When acute surgical intervention taken in obstructive jaundice?

Ans:

If the patient presents with obstructive jaundice and cholangitis which is not improved within 2-4
days of conservative treatment should undergo surgery.

Q. What are the complications of surgery done in obstructive jaundice?

Ans:

Intra-operative:

Hemorrhage: Due to coagulopathy.


Injury to the bile duct.
Precipitation of hepatic failure:
Glycogen depletion.
Anesthetic drug effect.
Operative hypovolemia.

Postoperative complications:

Reactionary hemorrhage.
Septicemia: Reflux of infected bile into blood.
Encephalopathy.
Hepatorenal shut down:
Hypoperfusion due to hypovolemia.
Drug effect.
Septicemia.
Wound infection.
Wound dehiscence.
Biliary stricture.

Q. What are the causes of post-operative jaundice?

105 SHARIFUL HALIM, K66, DMC


Sharifs personal notes

Ans:

Bile duct injury.


Complete ligation of bile duct.
Residual stone in CBD.
Viral hepatitis.
Hematoma.

Q. How will you determine ALP of liver or bone origin? What are the other sources of ALP?

Ans:

Liver: Increased 5- Nucleotidase. Here ALP is secreted from biliary canaliculi.

Bone: Increased S. calcium and S. phosphate.

Other: Placenta, small intestine.

Q. What is Courvoisiers law?

Ans:

In obstruction of CBD due to stone, distension of GB seldom occurs; the organ is usually
already shriveled.
In obstruction from other causes, distension is common by comparison.

Q. What is Charcots triad?

Ans:

Fluctuating jaundice.
Recurrent pain due to stone.
Intermittent fever with rigor: Due to cholangitis.

Benign bile duct stricture:

Causes:
Congenital: Biliary atresia.
Postoperative: Cholecystectomy, choledocolithotomy, gastrectomy, hepatic
resection.
Inflammatory: Stone, cholangitis, ascariasis.

106 SHARIFUL HALIM, K66, DMC


Sharifs personal notes

Trauma.
Idiopathic inflammatory stricture.
C/F:
H/O cholecystectomy.
Intermittent jaundice.
Features of cholangitis: Pain, fever with chills and rigors, jaundice.
Treatment:
Minimally invasive: Endoscopic balloon dilation followed by stenting.
Invasive:
Roux-en-Y hepatico-jejunostomy.

Cholangiocarcinoma
Treatment:
Pre-operative preparation: Same as patient of Choledocolithiasis.
Operative treatment:
Curative operation:
o Lower zone: Radical Whipples operation.
Palliative treatment:
o Endoscopic or percutaneous transhepatic stenting.

Carcinoma of GB:

Treatment:
Pre-operative preparation.
Operation:
Simple cholecystectomy: In stage I disease.
Extended/radical cholecystectomy: stage II-III.
Chemotherapy.

PANCREAS

Acute pancreatitis:

Definition:
Acute inflammation of pancreas.
Causes:
BAPPI.
Biliary stone: Causes bile reflux, reflux of activated enzyme, reflux of infected
fluid.

107 SHARIFUL HALIM, K66, DMC


Sharifs personal notes

Alcoholism: Direct toxicities.


Post-operative: Gastrectomy.
Post-ERCP: Ductal disruption, enzyme extravasation.
Idiopathic.
Others: Hypercalcemia, drug induced.
Affects men in young age and women in older age.
Symptoms:
Severe agonizing pain in the epigastrium which is constant, radiating to back and
not responding to usual doses of analgesics.
Nausea, vomiting, retching.
Features of dehydration and shock: Sweating, labored breathing, anxious, altered
consciousness level, weak/fatigued.
Signs of G/E:
Tachypnea, tachycardia, hypotension.
Decreased urine output.
Altered mental status.
Mild icterus: Gall stone pancreatitis.
Acute swinging pyrexia: Suggest cholangitis.
Signs in Abdominal examination.
Gray turners sign/Cullens sign: Due to bleeding into fascial plane.
Tenderness, muscle guarding and rigidity in upper abdomen.
Mild abdominal distension.
DDx:
Perforated peptic ulcer.
Acute Cholecystitis.
MI.
Left Lower lobe pneumonia.

Investigations:
Laboratory investigations:
CBC with ESR: Moderate leukocytosis and raised ESR.
S. amylase:
o Increased within 24-48 hours.
o Normal 50-300 IU/dl.
o >1000 IU/dl is diagnostic.
S. lipase: Most specific, elevates several days after attack.
Urinary amylase: 5000IU/24hour is abnormal.
S. calcium level: Decreased due to calcium combined with fatty acid.
Imaging studies:
Plain X Ray abdomen A/P view:
o Sentinel ileus/Generalized ileus.
o Colon cut-off sign: Distension of right transverse colon with
abrupt collapse of left transverse and descending colon due to
colonic spasm adjacent to pancreatic inflammation.
o Calcified gall stone and calcified pancreas.
CXR:

108 SHARIFUL HALIM, K66, DMC


Sharifs personal notes

o Left sided pleural effusion.


o ARDS.
USG of W/A:
o Swollen pancreas.
o Free peritoneal fluid.
o Gall stone disease (GSD).
o Dilation of CBD.
CT scan:
o Swollen pancreas.
o Free peritoneal fluid.
o GSD.
o Dilation of CBD.
Laparotomy:
o Diagnostic dilemma.
o Acute hemorrhagic necrotizing pancreatitis.
Management:
Conservative:
Mild to moderate cases: Normal ward.
Severe cases: HDU or ICU.
Bed rest.
NPO (7 days for mild to moderate cases, 14 days for severe cases)
NG suction.
IV fluids.
IV antibiotics.
Narcotic analgesics: Pethidine 100mg IM or IV.
Antispasmodics.
O2 inhalation.
Foley catheterization.
Monitoring: Urine output, ABG analysis, invasive monitoring.
Surgical:
Laparotomy Indication:
o Diagnostic dilemma.
o Acute hemorrhagic necrotizing pancreatitis.
o GSD.
o Pt. not responding to conservative treatment within 48 hours.
ERCP indication:
o Stone in ampulla of vater.
o Features of severe cholangitis.
o Abnormal LFTs

Q. What are the bad prognostic factors of acute pancreatitis?

Ans:

History/Symptoms:
Features of shock.
H/O DM, Previous surgery.

109 SHARIFUL HALIM, K66, DMC


Sharifs personal notes

Age >55 years.


Obese.
Initial attack in an alcoholic patient.
Signs:
Signs of shock.
Gray turner/Cullen sign.
Investigations:
Arterial PO2: <60mm of Hg.
S. calcium: <2 mmol/L.
RBS: >10mmol/l.
BUN: >5mg/dl.

Q. Why shock occurs in acute pancreatitis?

Ans:

Pain.
Sequestration of fluid:
Systemic extravasation of fluid into the interstitial space: Due to flushing of
inflammatory mediators to systemic circulation.
Localized extravasation of fluid in retroperitoneal space due to inflammation and
peritoneal burn.

Q. What are the complications of acute pancreatitis?

Ans:

Local:
Pancreatic necrosis.
Abscess.
Pseudocyst.
Ascites.
Pleural effusion.
Ileus.
Systemic:
Shock: ARDS- Renal failure.
DIC.
Hypocalcemia.
Hyperglycemia.

Carcinoma of pancreas

Management:
Pre-op preparation:
Surgical treatment: Same as periampullary carcinoma, becoz itself may be a
periampullary carcinoma.

110 SHARIFUL HALIM, K66, DMC


Sharifs personal notes

Curative:
o Whipples operation/Pancreaticoduodenectomy.
Palliative:
o Triple bypass: Choledoco-jejunostomy, gastro-jejunostomy,
jejuno-jejunostomy.
o Endoscopic or percutaneous stenting.

Periampullary carcinoma

Definition: Tumors these are located within 1cm of ampulla of vater.


Types:
Pancreatic adenocarcinoma.
Cholangiocarcinoma.
Adenocarcinoma of ampulla of vater.
Duodenal adenocarcinoma.
Treatment:
Pre-op preparation:
Surgery:
Curative:
o Pancreaticoduodenectomy.
Palliative:
o Biliary stenting (Endoscopic, Percutaneous).
o Triple bypass:
Gastro-jejunostomy.
Hepatico-jejunostomy.
Jejuno-jejunostomy.

Whipples operation

Resection of:
Whole part of duodenum up to 10cm of proximal jejunum.
Head and neck of pancreas.
Distal stomach.
Lower end of CBD.
GB and surrounding LN.
Anastomosis/Triple bypass:
Gastro-jejunostomy.
Hepatico-jejunostomy.
Jejuno-jejunostomy.
Another bypass: Pancreatico-jejunostomy.

111 SHARIFUL HALIM, K66, DMC


Sharifs personal notes

Pancreatic pseudocyst:

Q. What is pancreatic pseudocyst? Write its management.

Ans:

Definition: IMBDA a collection of amylase rich fluid enclosed in a wall of fibrous and granulation tissue.

Clinical features:

History of acute pancreatitis, trauma, surgery.


Small: Asymptomatic.
Large: Abdominal discomfort.
Very large: Visible swelling in the epigastrium, jaundice.

Investigations:

CBC with ESR: Leukocytosis.


S. amylase: Elevated.
Plain X-ray, USG, CT scan.
S. bilirubin and LFTs: If jaundice present.
ERCP.

Treatment:

Done in symptomatic cysts or size more than 5cm or any complications.


Minimally invasive:
USG guided aspiration.
Percutaneous drainage.
Invasive:
Excision.
Cystojejunostomy/cystoduodenostomy.

Spleen:

Q. What are the indications of splenectomy?

Ans:

Splenic rupture
Thalassemia.
Splenic neoplasm.
ITP.

112 SHARIFUL HALIM, K66, DMC


Sharifs personal notes

As a part of radical gastrectomy.

Q. A young boy admitted with left upper abdominal pain with shock following RTA?

a. Write down DDx/

b. What is your treatment plan?

Ans:

a. DDx: Splenic rupture, rupture of left kidney.


b. C/F:
H/O trauma.
Shock.
Abdominal distention, tenderness, muscle guarding.
Investigations:
Std. investigations: CBC with ESR, BUN, S. creatinine, S. electrolytes, ABD, CXR.
Plain X-Ray abdomen A/P view: Elevation of left dome of diaphragm, obliteration of
psoas shadow.
USG: Hematoma surrounding spleen.
CT scan of abdomen.
Treatment:
Conservative: Tt. Of shock.
Operative:
Splenectomy when there is continued bleeding despite resuscitation.

Q. What are the complications of splenectomy?

Ans:

Hemorrhage.
Injury.
Infection: Abscess, pancreatitis, pneumonia, septicemia; opportunistic infection.

UROLOGY

1. A 50 Y/O man presented with painless hematuria. What are the possible causes?

Ans:

Causes of painless hematuria:

Pre-renal:

Coagulopathy: Congenital or anticoagulant therapy.

113 SHARIFUL HALIM, K66, DMC


Sharifs personal notes

Sickle cell anemia.


Malaria.

Renal:

RCC.
Glomerulonephritis.
Renal TB.
Renal stone.
Renal infarction.
PCKD.
ATN.

Post-renal:

UB tumor/Cancer/Transitional cell CA.


BEP.
CA prostate.
Urethral neoplasm.

2. How IVU helps you to diagnose a case of painless hematuria?

Ans:

IVU in diagnosis of painless hematuria:

a. In IVU few minutes after injection of contrast medium e.g. urographin the first films are
taken which shows the renal parenchyma- Nephrogram phase.
b. Any distortion of renal outline or failure of part of kidney to function suggests a space
occupying lesion in kidney ie a stone, tumor/ TB.
c. After a few minutes the contrast is passed/excreted into collecting system then opacifying
the calyces & pelvis. Any distortion suggests a SOL.
d. Later films show the ureters.
e. At the end of the study the patient is asked to pass urine & final film is taken to show detail
of bladder area. If there is any distortion in outline- may suggest a SOL.

3. What is hydronephrosis?

Ans:

It is an aseptic dilatation of the kidney caused by obstruction to the outflow of urine.

4. What are the causes of unilateral & bilateral hydronephrosis?

Ans:

Causes of hydronephrosis:

114 SHARIFUL HALIM, K66, DMC


Sharifs personal notes

Cause Unilateral Bilateral


Ureteric: COMMON Uncommon.
Luminal: Stone, sloughed
papilla.
Mural: Pelviureteric junction
stenosis, Tumor, stricture.
Extra-Mural: CA cecum,
colon, rectum, cervix.
Bladder: Less common Common.
Stone, Tumor, bladder neck
scarring following surgery
Prostate: BEP, CA prostate Uncommon COMMON
Urethra: UNCOMMON Very COMMON
Congenital: Posterior
urethral valve, urethral
atresia, stricture, small
orifice. Acq: Stenosis,
phimosis.

5. Write down the C/F, investigation, treatment & complication of hydronephrosis.

Ans:

Clinical features:

In upper or lower obstruction:


Dull aching loin pain: Often with a sensation of dragging or heaviness which increases
with fluid intake.
Attacks of renal colic: If the cause of obstruction is stone.
If lower urinary tract obstruction:
Hesitancy.
Poor flow.
Dribbling of urine.
Sense of inadequate/incomplete voiding.
Due to azotemia:
Nausea.
Vomiting.
Irritability.

Investigations:

For finding out obstruction & its probable cause:


USG.
CT scan.
IVU.
Isotope renography.
S. PSA.
Trans-rectal USG.

115 SHARIFUL HALIM, K66, DMC


Sharifs personal notes

Retrograde uretero-pyelogram.
To observe effects/complications/the renal damage:
S. creatinine.
S. urea/ BUN.
S. electrolytes.
Urine R/M/E and C/S.

Treatment:

Aim: Removal of obstruction & drainage of urine.


If upper urinary tract obstruction: e.g. unilateral ureteric obstruction causing unilateral
hydronephrosis.
Pyeloplasty: E.g. Anderson-Hyness.
Endoscopic pyelolysis.
Ureteric stent.
Nephrostomy.
If lower urinary tract obstruction: ie bilateral hydronephrosis.
Insertion of urinary catheter.
If fails, Insertion of supra-pubic catheter/Supra-pubic cystostomy.

Complications:

Renal failure.
CKD.
Renal stone.
UTI.
Septicemia.
Death.

6. Classify renal tumors.

Ans:

Classification of renal tumors:

Benign:

Adenoma.
Angioma.
Angiomyolipoma.

Malignant:

Wilms tumor/nephroblastoma.
Grawitz tumor/adenocarcinoma/hypernephroma/Primary RCC.
Transitional cell carcinoma of the renal pelvis & collecting system.

116 SHARIFUL HALIM, K66, DMC


Sharifs personal notes

Squamous cell carcinoma of the renal pelvis.

7. What are the clinical features of RCC?

Ans:

CF of RCC:

Due to primary tumor:


Hematuria.
Dragging pain in the loin.
Mass detected by the patient.
HTN, Polycethemia, hypercalcemia.
Clot colic.
Rapidly developing varicocele esp. in the left side.
NAV.
Fever.
Due to distant metastasis:
Bones: Bone pain.
Lungs: Cough, hemoptysis.
Liver: Jaundice.
Other probable features:
Pyrexia.
Wt. loss.
Anemia.

8. A young man presented with palpable kidney. What are the possible causes?

Ans:

APKD= Adult polycystic kidney Disease.


Hydronephrosis due to any cause.
Pyelonephritis.
Simple renal cyst.
Renal tumors: Adenoma, angioma, angiomyolipoma.
Renal vein thrombosis.

9. Write down the C/F, investigation, treatment & complication of hydronephrosis.

Ans:

Clinical features:

117 SHARIFUL HALIM, K66, DMC


Sharifs personal notes

In upper or lower obstruction:


Dull aching loin pain: Often with a sensation of dragging or heaviness which increases
with fluid intake.
Attacks of renal colic: If the cause of obstruction is stone.
If lower urinary tract obstruction:
Hesitancy.
Poor flow.
Dribbling of urine.
Sense of inadequate/incomplete voiding.
Due to azotemia:
Nausea.
Vomiting.
Irritability.

Investigations:

For finding out obstruction & its probable cause:


USG.
IVU.
Isotope renography.
CT scan.
Retrograde uretero-pyelogram.
To observe effects/complications/the renal damage:
S. creatinine.
S. urea/ BUN.
S. electrolytes.
Urine R/M/E.

Treatment:

Aim: Removal of obstruction & drainage of urine.


If upper urinary tract obstruction:
Pyeloplasty: Ie Anderson-Hyness.
Endoscopic pyelolysis.
Ureteric stent.
Nephrostomy.
If lower urinary tract obstruction:
Insertion of urinary catheter.
Insertion of supra-pubic catheter.

Complications:

Renal failure.
UTI.
Septicemia.
Death.

118 SHARIFUL HALIM, K66, DMC


Sharifs personal notes

10. What are the clinical features of Wilms tumor?

Ans:

C/F of Wilm s tumor:

Due to primary tumor:

An abnormally large abdomen/ mass in the lumbar region.


Pain: in the anterior abdomen or loin.
Hematuria.
Fever.
Nausea & vomiting.
Wt. loss.
Pallor/ anemia.
HTN.

Due to distant metastasis:

Lung: Cough hemoptysis.


Bones: Bone pain.
Liver: Jaundice.
Brain: Focal symptoms, Early morning vomiting.
11. What are the investigations needed to be done in a suspected case of Wilms tumor?

Ans:

To diagnose the disease/tumor:

USG.
CT.
MRI.
IVU.

To detect metastasis:

CXR.
Radionuclide bone scans.
PET scan.
LFTs.

To see other effects of the tumor:

CBC with ESR.


Urine R/M/E.
S. creatinine.
S. urea.
BUN.
S. electrolytes.
12. How will you manage/treat a case of Wilms tumor?

119 SHARIFUL HALIM, K66, DMC


Sharifs personal notes

Ans:

Any case of Wilm s tumor should be treated in a pediatric oncology unit.


Treatment depends on the grading & staging of the tumor.
If low grade & very early stage disease only nephrectomy can be the curative treatment.
In higher grades & later stages treatment options may be:
Neo-adjuvant chemotherapy with regimen EE-4A or DD-4A followed by surgery ie
nephrectomy.
Nephrectomy followed by adjuvant chemotherapy with regimen EE-4A or DD-4A.
If bilateral tumor: Partial nephrectomy.
Before nephrectomy or chemotherapy:
Correction of anemia.
Correction of HTN.
Correction of fluid & electrolyte imbalance.
Control of infection.
Supporting the patient with parenteral nutrition.

13. What are the probable causes & investigation of a radio-opaque shadow on right renal region
on plain X-Ray?

Ans:

Probable causes of radio-opaque shadow on right renal region in plain X-Ray:

Renal stones.
Gallstones.
Calcified TB lesion in kidney
Calcified mesenteric lymph node.

th
Ossified tip of 12 rib.

Investigations to be done next (Clue think about each DD & probable investigations for them):

USG of W/A.
CT scan.
IVU.
Isotope renography.
Urine R/M/E.
S. ALP.
S. creatinine.
BUN.
S. urea
CXR.
Sputum for AFB.
MT.

120 SHARIFUL HALIM, K66, DMC


Sharifs personal notes

14. A 35 Y/O man presents with a right loin mass.


a. What are the renal & extra-renal cause?

Ans:

Renal cause:

Benign kidney tumor: Adenoma, angioma, angiomyolipoma.


Malignant tumor: Grawitz tumor, SCC, transitional cell CA.
Unilateral hydronephrosis.
Pyelonephritis.
APKD.
Simple renal cyst.
Renal abscess.
Renal TB.

Extra-renal cause:

Gut: CA ascending colon, Gut lymphoma.


Liver: Adenoma, HCC, abscess, hydatid cyst.
GB: Mucocele, GB CA.
Adrenals: CA adrenal.
15. What is anuria, write the common causes of renal anuria.

Ans:

Anuria:

th
Complete absence of urine production (B&L 25 ) for 12 hours or more.

Renal causes of anuria:

AGN.
Nephrotoxicity:
Drugs.
Poisons.
Contrast media.
Eclampsia.
Myoglobinuria/Rhabdomyolysis.
Incompatible blood transfusion.
DIC.
HUS.

16. What is TURP & TURBT? How will you manage a case of severe hematuria after TURP?

Ans:

121 SHARIFUL HALIM, K66, DMC


Sharifs personal notes

TURP: Means Trans-Urethral Resection of Prostate.

TURBT: Means Trans-Urethral Resection of Bladder Tumor.

Management of severe hematuria after TURP:

Hospital admission.
Catheterization of Urinary bladder.
Maintenance of hydration: >2.5 L of fluid/day.
Drugs: Finasteride, Tranexamic acid (1g TDS for 3 weeks).
Bed rest.
Regular diet.

17. What is hematuria? What are the causes of hematuria?

Ans:

Hematuria: Passage of blood in the urine.

Causes:

Pre-renal:
Purpura.
Anticoagulants.
Hemophilia.
Sickle cell anemia.
Renal:
AGN.
Renal stones.
Renal TB.
RCC.
Injury to the kidney.
Renal infarction
Post-renal:
UTI.
Stone in the ureter.
Ureteric tumor.
Stone in UB.
Transitional cell CA of bladder.
TB of UB.
Schistosomiasis.
BEP.
CA prostate.
Rupture of urethra.
Urethral tumor/malignancy.

122 SHARIFUL HALIM, K66, DMC


Sharifs personal notes

18. How will you investigate a case of hematuria in a middle aged patient?

Ans:

Urine R/M/E.
Urine for Gram staining, C/S.
X-Ray of KUB region.
USG of W/A.
CT scan.
IVU.
Flexible or rigid cystoscopy.
S. creatinine.
S. urea.
BUN.
S. electrolytes.

19. What are the causes of painful hematuria?

Ans:

Causes of painful hematuria:

UTI.
Pyelonephritis.
Stone in: Renal pelvis, ureter, bladder.
Ruptured urethra.
Loin pain hematuria syndrome.
Prostatitis.
Kidney injury.
Renal infarction.

Renal & ureteric stones:

Types:

1. Oxalate stones: Common, spiky, more hematuria.


2. Phosphate stones: Along with magnesium ammonium phosphate- Struvite stone, forms in
alkaline urine ie by infection with urea splitting organisms like proteus & some Strep. species.
May be large & Asymptomatic. May take a characteristic shape to be called a Staghorn
calculus.
3. Uric acid stones: Can be radiolucent.
4. Cysteine stone: Uncommon

Clinical features:

a. Constant pain: In the renal angle posteriorly or in the hypochondrium anteriorly.

123 SHARIFUL HALIM, K66, DMC


Sharifs personal notes

b. Ureteric/renal colic: Attack of severe agonizing colicky pain in the loin often radiating to
groin, testes, penis, inner aspects of thigh. May be associated with vomiting.
c. Hematuria.

Investigations:

1. Plain X- Ray KUB region A/P view.


2. Urine R/M/E.
3. Urine for Gram stain & Culture.
4. Contrast enhanced spiral CT scan.
5. USG of Abdomen.
6. IVU- Intravenous urography.
7. Assessment of renal function.

Q. What are the different methods of treatment of renal stones?


1. Renal stones:
a. Minimal invasive techniques:
ESWL- Extracorporeal Shock Wave lithotripsy.
Percutaneous nephrolithotomy: USG guided.
b. Invasive/ open procedures:
Nephrolithotomy: Stone in calyces.
Pyelolithotomy: Stone in extra-renal pelvis.
Extended Pyelolithotomy: Stone in intra-renal pelvis.

Q. What are the treatment modalities for ureteric stones?

Ureteric stone removal:


a. Minimally invasive:
Ureteric meatotomy: Close to meatus.
Dormia basket removal.
Ureteroscopic removal.
Lithotripsy in situ.
Push bang: High up stone.

b.
Invasive:
Ureterolithotomy.
20. What are the causes of renal stones?

Ans:

Less fluid intake/dehydration.


Dietary: Vitamin A deficiency.
Hyperparathyroidism: Causing hypercalcemia
Urinary infection: By urea splitting organisms such as proteus sp., streptococci &
staphylococci.
Urinary stasis.
Prolonged immobilization leading to skeletal decalcification.

124 SHARIFUL HALIM, K66, DMC


Sharifs personal notes

Q. What are the complications of renal stones?

Ans:

Hematuria.
Hydronephrosis.
Increased incidence of UTI.
Acute Renal failure.
CKD.

Q. How will you prevent the recurrence of renal stones?

Ans:

Prevention of recurrence of renal stones:

Adequate hydration.
More intake of vitamin A & C rich food.
Foods: Fiber rich diet, less protein intake.
Restriction of sodium intake.
More potassium containing food intake.
In people with hyper-uricemia: Avoidance of red meat & fish.
Ca-oxalate stone: Moderate intake milk product, spinach, strawberry.

Q. How will you manage a case of renal colic due to stone obstructing the pelvi-ureteric junction?

Ans:

Conservative management:

Analgesics.
Antibiotics.
Plenty of fluids.
Diet rich in vitamin A or C.

Surgical management: Pyelolithotomy/Push bang.

Q. What are the effects of long standing Staghorn calculus in renal pelvis?

Ans:

Chronic kidney disease.


Hydronephrosis.
Increased incidence of UTI.
Acute Renal failure.
Hematuria.

125 SHARIFUL HALIM, K66, DMC


Sharifs personal notes

21. What is retention of urine?

Ans: Inability to voluntarily void urine despite normal production.

22. What are the causes of acute retention of urine in a 5 Y/O child?

Ans:

Phimosis.
Paraphimosis.
Balanitis.
Posterior urethral valve.
Congenital stenosis of the urethra.
Foreign body in the urinary tract.
Ruptured urethra.
Circumcision.
23. What are the causes of acute urinary retention in adult?

Ans:

Bladder:

Bladder outlet obstruction due to:


Stone.
Blood clot.
CA bladder.
Neurogenic bladder: trauma to spinal cord. Post-operative.

Prostate:

BEP.
CA prostate.
Prostatitis.

Urethra:

Urethritis.
Ruptured urethra.
Impacted stone.
Tumor.

In women:

Retroverted gravid uterus.


Uterine prolapse causing BOO
Uterine mass ie fibroid, other gynecologic malignancy.

Drugs:

126 SHARIFUL HALIM, K66, DMC


Sharifs personal notes

Anticholinergics (eg, antipsychotic drugs, antidepressant agents, anticholinergic respiratory


agents).
Opioids and anaesthetics.

24. Write the management of acute retention of urine.

Ans:

Initial management:

Immediate and complete bladder decompression, usually with a Foley urinary catheter.

If this fails or is contra-indicated (eg, urethral trauma or stenosis), Alternatives include


suprapubic catheters/cystostomy.

Pharmacological treatment for postoperative retention (eg, cholinergics, intravesicle


[11]
prostaglandin) has been explored as an alternative to catheterisation.

As for any intimate examination, the patient should always have the option of a chaperone, although
[12][13]
many will decline.

Secondary management:

This is dependent on the cause of the AUR. For AUR caused by prostatic enlargement:

Alpha blockers & 5-alpha reductase inhibitors.

Prostatic surgery within a few days (emergency surgery) or a few weeks (elective surgery)
of a first AUR episode.

Trial without catheter (TWOC): This involves removing the catheter after 1-3 days: patients
are able to void in 23-40% of cases and surgery, Alpha-1 blockers given before catheter
removal increase the chances of a successful TWOC.

Other treatment options: According to cause of obstruction:

25. What are the causes of BOO- Bladder outlet obstruction?

Ans:

Impacted bladder stone.


Blood clots.
Bladder cancer.
Bladder neck stenosis.
Bladder neck hypertrophy.
Functional obstruction due to neuropathic conditions.
BPH.
CA prostate.

127 SHARIFUL HALIM, K66, DMC


Sharifs personal notes

Urethral stricture.
Posterior urethral valve.
Pelvic tumor: Cervix, uterus, rectum.

26. How will you manage post-operative retention of urine?

Ans:

Immediate:

Urethral catheterization.
Suprapubic cystostomy if catheterization fails.

Others:

Muscarinic agonists.
Naloxone if the patient had opioid analgesics.
Early mobility.
Warm bath.
Discouraging bed pans.
Avoidance of medication that can aggravate the condition:
Opiates.
Antipsychotics.
Antispasmodics.
Anti-parkinsonian.

27. How do you treat a patient having chronic retention of urine?

Ans:

Specific measures:

Catheterization Chronic indwelling catheterization, intermittent catheterization

Surgical interventions Male-specific etiologies: prostate surgeries


(etiology-specific)
Female-specific etiologies: pelvic organ prolapse repair;
adjustment to procedures for stress urinary incontinence

128 SHARIFUL HALIM, K66, DMC


Sharifs personal notes

Nonsex-specific etiologies: sacral nerve stimulation

Multiple etiologies: urinary diversion procedures

Pharmacological Alpha blockers (AB): doxazosin, prazosin, tamsulosin,


interventions terazosin, alfuzosin, and silodosin

5-Alpha reductase inhibitors (5-ARI): dutasteride and


finasteride

AB + 5-ARI combination therapy: tamsulosin/dutasteride

Neurogenic etiologies: botulinum toxin

General measures:

Stop any precipitating/aggravating medication.


General lifestyle advice such as:

Regulating fluid intake and avoiding evening drinking.

Reducing alcohol intake.

Reducing tea and coffee intake.

Preparation enabling access to toileting facilities.

28. Classify vesical calculi? What are the different methods for treating a bladder stone?

Ans:

Types of vesical calculi:

Oxalate stone.
Phosphate stone.
Uric acid stone.
Cysteine stone.
Xanthine stone.

Different methods of treating bladder stones:

Increasing fluid intake.


Cystoscopic removal/transurethral cystolitholapaxy: ie removal of stone under the view of
cystoscope using:
Ultrasonic energy.
Laser lithotripsy.

129 SHARIFUL HALIM, K66, DMC


Sharifs personal notes

Percutaneous supra-pubic cystolitholapaxy.


Suprapubic cystolithotomy.
Open cystostomy: an incision is made in the bladder and the stones are removed manually.
Treatment of underlying cause.
29. What is stranguary?

Ans:

Stranguary is the symptom of painful, frequent urination of small volumes that are expelled slowly
only by straining and despite a severe sense of urgency, usually with the residual feeling of incomplete
emptying.

These 'drops' of urine are 'squeezed out' in what sufferers describe as painful 'wrenching' spasms. The
pain is felt to arise in the suprapubic region, extends up to the root of the genitalia and in male
patients, to the tip of the penis. Also associated with few drops of blood at the end of micturition.

This distressing desire to fully void despite its impossibility is attributed to the irritation of urothelium
(epithelium lining the urinary tract) and subsequent spasm of muscles. It is seen in numerous
urological conditions including kidney stones (especially when a stone is impacted at the
vesicourethral junction), bladder inflammation (cystitis), and bladder cancer.

1. What are the secondary effects of BEP?

Ans:

Prostatic urethra:
Lengthening.
Distortion of urethra.
Exaggeration of posterior curve.
Bladder:
Muscle hypertrophy.
Trabecular appearance.
Shallow depression in between trabecular.
Increased blood flow.
Bladder muscle dysfunction.
Stone formation.
Cystitis.
Increased post-void residual volume.
High voiding pressure.
Ureter: Hydroureter.
Kidney: Hydronephrosis.
2. What are the complications of BEP?

Ans:

Acute urinary retention.


Chronic urinary retention.
Increased incidence of infection.

130 SHARIFUL HALIM, K66, DMC


Sharifs personal notes

Increase incidence of stone formation.


Hydronephrosis.
Acute renal failure.
CKD.
Bladder dysfunction: Muscular wall of bladder stretches weakens & no longer works
properly.
Hematuria.
Effects of straining: Hemorrhoids, hernia, rectal prolapse.

3. How will you manage a patient presenting with acute retention of urine due to BEP?

Ans:

Immediate:

Urinary Foley catheterization.


If catheterization fails- Suprapubic cystostomy.

Later on:

Drugs: Alpha blocker- Prazosin, terazosin, tamsulosin. 5-alpha reductase inhibitor-


Finasteride.
Surgery:
Prostetectomy: commonly TURP.
General advice:
Avoidance of medications than can exaggerate the condition: Opiates,
antispasmodics, anti-parkinsonian, antipsychotics.

Regulating fluid intake and avoiding evening drinking.

Reducing alcohol intake.

Reducing tea and coffee intake.

Preparation enabling access to toileting facilities.

4. What are the modes of presentation of BEP?

Ans:

Presentation of BEP:

Voiding/obstructive symptoms:

Hesitancy.
Poor flow.
Intermittent stream.
Sensation of poor bladder emptying.
Dribbling.

131 SHARIFUL HALIM, K66, DMC


Sharifs personal notes

Episode of near retention.

Storage/irritative symptoms:

Frequency.
Nocturia.
Urgency.
Urge incontinence.
5. What is LUTS/Prostatism? Enumerate them. How will you manage a patient with LUTS?/ Write a
short note on LUTS.

Ans:

LUTS= Means Lower Urinary Tract Symtoms. These are the symptoms that arise from obstruction or
irritation of lower urinary tract.

Common LUTS are:

Voiding/obstructive symptoms:

Hesitancy.
Poor flow.
Intermittent stream.
Dribbling.
Sensation of poor bladder emptying.
Episode of near retention.

Storage/irritative symptoms:

Frequency.
Nocturia.
Urgency.
Urge incontinence.

Management of a patient with LUTS:

Medical/pharmacological treatment:
Alpha blockers: Prazosin, terazosin, tamsulosin.
5 Alpha reductase inhibitor: Finasteride, dutasteride.
General advice:
Avoidance of medications than can exaggerate the condition: Opiates,
antispasmodics, anti-parkinsonian, antipsychotics.

Regulating fluid intake and avoiding evening drinking.

Reducing alcohol intake.

Reducing tea and coffee intake.

Preparation enabling access to toileting facilities.

132 SHARIFUL HALIM, K66, DMC


Sharifs personal notes

Specific treatment: According to cause.

6. What are the indications of prostatectomy?

Ans:

BEP with:

Acute retention
Chronic retention with renal impairment.
Complications of BOO: Stone, infection, diverticulum formation.
Hemorrhage.
Severe symptoms.
Flow rate <10ml/s.
PVR >100ml.
7. Write down the surgical treatment options for a patient with BPH.

Ans:

Minimal invasive procedures:

TUNA: Transurethral needle ablation.


TUMT: Transurethral microwave thermotherapy.

Invasive procedures:

TURP.
Transurethral incision of the prostate (TUIP)
RPP-Retropubic prostetectomy.
TVP- Transvesical prostetectomy.
Perineal prostetectomy.
8. What are DRE findings in BEP & CA prostate?

Condition DRE findings


BEP

Enlargement of the gland is usually symmetrical


Surface Smooth with no nodularity.
Prominent median sulcus.
Consistency is rubbery, or slightly elastic.
Rectal mucosa is free.
Examining finger: Not blood stained.

CA prostate

Asymmetric shape
Hard consistency
Discrete nodule may be palpable
Obliteration of the median sulcus.
Rectal mucosa may be fixed.
Examining finger: Blood stained.

133 SHARIFUL HALIM, K66, DMC


Sharifs personal notes

9. What are the post-op complications of prostatectomy?

Ans:

Local:

Hemorrhage: Primary, reactionary, secondary.


Injury: Perforation of bladder or prostatic capsule.
Infection: Wound infection.
Incontinence.
Retrograde ejaculation.
Impotency.
Urethral stricture.

Systemic:

Sepsis.
Water intoxication: TURP syndrome. Causes CCF and Hyponatremia.
Cardiorespiratory anesthetic complication: Pulmonary atelectasis, pneumonia, MI.
10. A patient of 60 years has presented with frequency, urgency & dribbling of urine:
a. Write the causes?
b. What investigations you advice for diagnosis?

Ans:

a. Causes:
BPH.
CA prostate.
Bladder neck stenosis.
Bladder cancer.
Urethral stricture.
UTI.
b. Investigations:
Urine R/M/E.
Urine for Gram staining & C/S.
Pressure-Flow Urodynamic study: Urine flow rate, residual volume, pressure of flow.
USG.
Trans-rectal USG.
Cystourethroscopy.
Prostatic biopsy.
IVU.
S. creatinine, S. urea, S. electrolytes.
PSA= Prostate specific antigen.

2. What are the causes of stricture urethra?

134 SHARIFUL HALIM, K66, DMC


Sharifs personal notes

Ans:

Congenital stricture or valve.


Urethritis: ie due to gonococcus.
Urethral injury:
Traumatic.
Instrumental: Indwelling catheter, urethral endoscopy.
Post-op: Open prostetectomy.
Urethral neoplasm.

3. What are C/F & treatment of stricture urethra?

Ans:

C/F: Obstructive features of LUTS.

Hesitancy.
Poor flow.
Intermittent flow.
Dribbling.
Sense of incomplete voiding.
Prolonged micturition.
Increased straining.
Increased frequency.
Acute retention of urine.

Treatment:

Dilation:
Gum-elastic bougie.
Metal sounds.
Self-dilation with Nelaton catheter.
Filiform & follower.
Urethrotomy:
Internal visual Urethrotomy.
Urethroplasty:
Excision & end-to-end anastomosis.
Patch Urethroplasty.

4. What are the dangers of fracture pelvis?

Ans:

Severe bleeding & Shock.


Sepsis.
Ruptured UB.
Ruptured membranous urethra.

135 SHARIFUL HALIM, K66, DMC


Sharifs personal notes

Thromboembolism.
Sexual dysfunction: Erectile dysfunction, loss of sensation.
Bowel dysfunction.
Chronic pain.
Degenerative arthritis.
5. How do you manage a case of rupture urethra?

Ans:

Clinical features:

H/O trauma to the perineum.


Urinary retention.
Perineal hematoma.
Bleeding per external urethral meatus.
Pain.
Displaced high prostate.

Investigations:

Retrograde urethrogram.
Flexible cystoscopy.
Plain X-Ray KUB region.

Treatment of ruptured urethra:

Initial treatment:

Discouraging from passing urine in a suspected rupture.


Full bladder drained by supra-pubic cystostomy.
Appropriate analgesics.
Administration of prophylactic antibiotics.

Specific treatment:

Open repair: excision of traumatized segment & end to end re-anastomosis of urethra.
Urethroscope guided catheterization: using a urethroscope & alignment of ends of the
urethra in same plane while healing occurs spontaneously.

Delayed Urethroplasty.

Suprapubic tube plus endoscopic realignment.

HERNIA

136 SHARIFUL HALIM, K66, DMC


Sharifs personal notes

Q. Define and classify hernia?

Ans:

Hernia: IMBDA an abnormal protrusion of a viscus or a part of it through any weak point of the wall
of the cavity within which it is contained with or without external manifestation.

Classification:

A. According to onset:
Congenital hernia.
Acquired hernia.
B. Clinical classification:
Reducible.
Irreducible.
Obstructed.
Strangulated.
Inflamed.
C. According to the anatomical site:
External: Inguinal, femoral, umbilical, incisional, epigastric, lumbar.
Internal: Hiatus hernia, diaphragmatic hernia.
D. According to contents:
Omentocele.
Enterocele.
Littres hernia: Content is Meckels diverticulum.
Richters hernia.

Q. Classify inguinal hernia.

Ans:

A. According to relation with inguinal canal:


Direct inguinal hernia: Contents come out thru posterior wall of inguinal canal.
Indirect inguinal hernia: Contents come out thru deep inguinal ring.
B. According to extent:
Incomplete: When contents don t reach the lower part of the scrotum.
Complete: When contents reach the lower part of the scrotum.

Q. Write down the management of a case of strangulated inguinal hernia.

Ans:

Symptoms:

H/O of an inguinoscrotal mass which bulges on coughing and reduces on lying.


Severe colicky abdominal pain followed by constant pain in the abdomen.
Protracted vomiting.
Constipation.
Fever.

137 SHARIFUL HALIM, K66, DMC


Sharifs personal notes

Signs:

G/E: Pt. is toxic, temperature raised, pulse- rapid and thready, BP-low.
P/A/E:
There is an inguinoscrotal swelling.
The overlying skin is red and edematous.
No visible cough impulse.
Tender.
Irreducible.

Treatment:

Immediate resuscitation followed by emergency operation:


NPO.
NG suction.
IV fluid.
Broad spectrum antibiotics.
Analgesics.
Urinary catheterization.
Operation:
Relief of obstruction.
If bowel is gangrenous/non-viable; excision of gangrenous segment and
end to end anastomosis.
Herniotomy with herniorrhaphy or hernioplasty is done.

Q. What are the complications of inguinal hernia?

Ans:

Irreducible hernia.
Incarcerated hernia.
Strangulated hernia.
Inflamed hernia.
Infection.
Testicular atrophy.
Calcification of the wall.
Torsion of omentum.

Q. What are the pathological changes that occur in strangulated hernia?

Ans:

Obstruction of the blood supply of part of bowel loop inside the hernial sac.
First venous obstruction occurs causing congestion and edema of the wall.
Followed by arterial obstruction which causes ischemia.
Ischemia causes gangrene/necrosis formation.
Proliferation of the bacteria causing infection.

138 SHARIFUL HALIM, K66, DMC


Sharifs personal notes

Peritonitis.
Shock and septicemia.

Q. What are the signs of viability of small gut following strangulation?

Ans:

Dark Color turns to light upon relieving strangulation.


The wall is firm.
Visible peristalsis may be seen.
Peritoneum is shiny.
Pulsation of mesenteric artery may be seen or felt.
Mesentery bleeds if pricked.

Q. What is herniotomy, herniorrhaphy and hernioplasty?

Ans:

Herniotomy: It is a surgical process involving dissection of the hernia sac, reduction of contents and
trans-fixation of the neck with removal of the remainder.

Done in congenital indirect inguinal hernia, congenital hydrocele.

Herniorrhaphy: Herniotomy plus repair of the deep inguinal ring and reinforcement of the posterior
wall of the inguinal canal.

Done in: Indirect/direct inguinal hernia.

Hernioplasty:

Repair of the posterior wall of inguinal canal with synthetic mesh.

Done in: D/I inguinal hernia.

Q. What is ventral hernia? What are the factors that influence development of ventral hernia?

Ans:

Ventral hernia: Any protrusion thru abdominal wall except inguino-femoral hernia.

Example: Incisional, umbilical, para-umbilical hernia, epigastric hernia.

Predisposing factors:

Coughing.
Constipation.
BEP.
Major operation.

139 SHARIFUL HALIM, K66, DMC


Sharifs personal notes

Congenital defect.
Weak abdominal musculature.
Others: Smoking, alcohol, obesity.

Q. What are the steps of operation of inguinal hernia of a 5 Y/O boy?

Ans:

Herniotomy.

HYDROCELE

Q. Write a short note on hydrocele.

Ans:

Definition: Abnormal collection of serous fluid in some part of processus vaginalis usually in the tunica.

Classification:

Congenital:
Congenital.
Infantile.
Vaginal.
Hydrocele of the cord.
Acquired:
Primary: PVH.
Secondary:
o Trauma.
o Tumor.
o TB.
o Torsion.
o Epididymo-orchitis.
o Post herniorrhaphy.
o Filarial hydrocele.

Pathogenesis:

Congenital: Impaired involution of processus vaginalis.


Acquired: Increased secretion, defective absorption, interference with lymphatic drainage.

Clinical features:

Painless cystic swelling in the scrotum or inguinoscrotal region.

140 SHARIFUL HALIM, K66, DMC


Sharifs personal notes

Swelling is:
Cystic.
Fluctuant.
Transilluminant.
Get above the swelling is possible.
Testes cant be palpated separately.

Treatment:

Vaginal hydrocele:
Lords operation: Sac is thin walled. Plication of redundant segment by a series of
interrupted absorbable suture.
Jabulays operation: Eversion of the sac and anchoring with sutures.
Congenital hydrocele: Herniotomy.
Infantile/ encysted hydrocele of cord: Excision of sac.
Secondary: According to coz.

Complications:

Rupture.
Hemorrhage and hematocele.
Pyocele.
Atrophy of testis.
Calcification of sac.
Herniation of hydrocele thru dartos muscles.

Q. How will you differentiate between hernia and hydrocele?

Ans:

Point Inguinal hernia Hydrocele


Definition
Region Inguinoscrotal Scrotal
Cough impulse Positive Negative
Reducibility Positive Negative
Get above the swelling Not possible Possible
Consistency Variable Cystic
Fluctuation, Negative Positive
Transillumination.
Testis Separately palpable Not separately palpable.

THYROID

Goitre: Generalized enlargement of thyroid gland.

141 SHARIFUL HALIM, K66, DMC


Sharifs personal notes

Solitary/Isolated nodule: Discrete swelling/nodule in one lobe with no palpable abnormality


elsewhere.
Dominant swelling: Swelling with evidence of abnormality elsewhere in the gland.
Classification of thyroid swelling:
Simple Goitre:
Endemic goiter/MNG.
Pubertal or pregnancy associated goiter.
Toxic goiter:
Diffuse toxic goiter: Graves disease.
Toxic multinodular goiter.
Toxic adenoma.
Neoplastic:
Benign: Follicular adenoma.
Malignant: Papillary, follicular, medullary, and anaplastic.
Inflammatory:
Autoimmune: Hashimotos thyroiditis.
De Quervains thyroiditis- Granulomatous.
Riedels thyroiditis- Fibrosing.
Infective: Acute- Pyogenic bacteria and viruses, and chronic infection- TB, syphilis.

Q. What are the differences between papillary and follicular thyroid carcinoma?

Ans:

Characteristics Papillary carcinoma Follicular carcinoma


% of total cancer of thyroid About 85% About 5-10%
Cause Previous exposure to ionizing Dietary iodine deficiency
radiation. (Long standing goitre)
Gross morphology Solitary/multifocal Solitary

Microscopy: Contain branching papillae with Fairly uniform cells forming


a fibrovascular stalk, ground small follicles containing
glass or orphan annie nuclei, colloid.
psammoma bodies.
Metastasis Predominantly lymphatic. hematologic
Cervical lymphadenopathy Common Not so common.

Prognosis Good Bad


Recurrence Less chance More chance.

Q. What are the limitations of FNAC in thyroid swelling?

Ans:

Follicular adenoma can t be differentiated from follicular carcinoma.


False negative results are common.
Aspiration of unsatisfactory samples.

142 SHARIFUL HALIM, K66, DMC


Sharifs personal notes

Q. A 20 Y/O unmarried female noticed a nodular swelling over her right thyroid region without
producing any signs or symptoms for six months. FNAC reveals follicular neoplasm. How will you
investigate and manage the case?

Ans:

Investigations:
Detailed history.
Elaborate clinical examination.
Lab investigations:
TFTs.
USG.
RAIU.
Investigations for staging: For liver, lung, bone, brain.
Investigations for G/A fitness.
Management:
Total thyroidectomy with preservation of RLN and Parathyroid gland followed by
suppressive therapy.
Radioiodine ablation therapy.
Thyroxin supplement.

HYPOTHYROIDISM:

Definition: Structural and functional derangement of thyroid gland leading to decreased


thyroid hormone secretion.
Classification/Causes:
Autoimmune thyroiditis: Hashimoto s thyroiditis.
Iatrogenic: After thyroidectomy, radioiodine ablation or drugs such as PAS, iodide in
excess, lithium, amiodarone.
Dyshormonogenesis.
Iodine deficiency.
Goitrogens.
Thyroid agenesis.
Secondary to pituitary or hypothalamic diseases.

Types of hypothyroidism:
Fetal or infantile hypothyroidism or cretinism./Congenital hypothyroidism.
Adult hypothyroidism/Myxoedema.
Cretinism/Congenital hypothyroidism:
Cause: Maternal or fetal iodine deficiency; thyroid agenesis.
Presentations:
Delayed passage of meconium.
Prolonged physiological jaundice/neonatal jaundice.
Coarse facies.
Hoarse cry.

143 SHARIFUL HALIM, K66, DMC


Sharifs personal notes

Pot belly abdomen.


Umbilical hernia.
Delayed physical and mental development.
Constipation.
Dry wrinkled skin.
Investigations: Heel prick blood test to estimate TSH, T4 level.
Tt: Immediate replacement with thyroxin for physical and mental development.
Adult hypothyroidism or myxedema:
Presentations/Symptoms:
Tiredness.
Mental lethargy.
Cold intolerance.
Wt. gain.
Constipation.
Menstrual disturbance- menorrhagia.
Carpal tunnel syndrome.
Signs:
Bradycardia.
Cold extremities.
Dry skin and coarse hair.
Periorbital puffiness.
Hoarseness of voice.
Bradykinesis- slow movement.
DELAYED RELAXATION OF ANKLE JERK- MOST DIAGNOSTIC.
Investigations:
TFTs.
Antibody against TPO.
Treatment:
Oral thyroxin: .1-.2 mg/day.
Maintenance dose: .05mg/day.

Q. What are the thyroid emergencies?

Ans:

Thyrotoxic crisis/Thyroid storm.


Myxoedema coma.
Obstruction of URT by large goitre.
Malignant ophthalmopathy.

HYPERTHYROIDISM- THYROTOXICOSIS.

Hyperthyroidism: Symptoms due to increased thyroid hormone level due to


hyperfunctioning of thyroid gland.
Thyrotoxicosis: Symptoms not explained by raised level of thyroid hormones.
Clinical types/cause:
Primary:

144 SHARIFUL HALIM, K66, DMC


Sharifs personal notes

Diffuse toxic goitre- Grave s disease.


Toxic Multinodular goitre.
Toxic nodule.
Toxic adenoma.

Secondary:
Hyperthyroidism due to other causes:
o Factitious thyrotoxicosis.
o Iodine induced.
o Struma ovary- Ectopic hormone secretion.
o Subacute thyroiditis (Granulomatous- De Quervain s, or lymphocytic)
Symptoms:
Loss of weight despite good appetite.
Recent preference for cold/Heat intolerance.
Unexplained diarrhea.
Palpitation.
Tiredness.
Anxiety, sleeplessness.
Signs:
Pt. is anxious looking.
Presence of goitre is common.
Exophthalmos: With lid retraction, lid lag.
Tachycardia.
Warm moist skin.
Fine tremor.
Bruit over thyroid gland: In Graves .
Irregularly irregular pulse/Afib: More common in thyrotoxicosis other than Graves .
Pretibial 145myxedema.
Investigations:
TFTs: S. T3, T4, TSH.
Thyroid autoantibody: TSH-R antibody increases in Graves disease.
RAIU.
Principle of treatment of thyrotoxicosis:
Rest and sedation.
Anti-thyroid drugs:
In patients under 40 years.
Carbimazole.
Propylthiouracil.
Beta blockers.
If one complication remembered- Agranulocytosis- Advise the patients
on Carbimazole therapy to immediately stop the treatment and consult
with doctor if any fever or sore throat happens.
Radioiodine therapy:
Contraindicated in pregnant women.
Given in patients over 45 years.
Surgery:
Done if large goitre with pressure symptoms, risk of malignancy and
cosmetic problems.

145 SHARIFUL HALIM, K66, DMC


Sharifs personal notes

Total or subtotal thyroidectomy preserving the RLN and parathyroid


gland followed by thyroid hormone supplementation.
Difference between primary and secondary thyrotoxicosis:

Point Primary thyrotoxicosis Secondary thyrotoxicosis.


Pathological process Hyperfunction of the gland Thyroid function is normal.
Enlargement of thyroid Present Normal Absent.
gland
Exophthalmos Present Absent.
Pretibial myxedema. Common Uncommon.
Thyroid bruit Common Absent
Neurological sign More common Less common.
CVS signs Less common More common.

Thyrotoxic crisis/thyroid storm:


Definition: An acute life threatening, hypermetabolic state induced by excessive
release of thyroid hormones.
Causes:
Infection.
Thyroid surgery.
Withdrawal of anti-thyroid drugs.
DKA.
Recent trauma.
Hypoglycemia.

Pathogenesis: In any stress there is increased release of catecholamines which


causes:
Increased conversion of T4 to T3.
Increased thyroid hormone synthesis and release.
Clinical features:
Hyperpyrexia: >41 Degree Celsius.
Profuse sweating.
HTN.
Tachycardia.
Cardiac arrhythmia.
Nausea, vomiting, abdominal pain.
Convulsion, coma.
Investigations:
T3, T4 and TSH.
ECG.
RBS.
ABG analysis.
LFTs, RFTs.
Tests to identify the source of infection if any.
Treatment:
Resuscitation:
o Oxygen inhalation.

146 SHARIFUL HALIM, K66, DMC


Sharifs personal notes

o IV infusion of normal saline.


Control of hyperpyrexia:
o Tepid sponging.
o Cooling blanket.
o Paracetamol.
Propranolol: First IV then oral.
IV hydrocortisone.
For CVS problems:
o Digoxin.
o Diuretics.
Correction of hyperthyroid state:
o Lugols iodine.
o Propylthiouracil.
o Sodium iodide.

THYROID SURGERY:

Q. What are the types and indication of thyroid operations?

Ans:

Type of operation Description Indication


Lobectomy Only one lobe is excised MNG, Follicular adenoma, cyst.
Hemithyroidectomy 1 lobe + Isthmus MNG, Follicular adenoma.
Subtotal thyroidectomy 2 gm. thyroid tissue left at MNG, Follicular adenoma.
upper pole of each lobe.
Near total thyroidectomy 2 gm. of thyroid tissue left MNG, follicular adenoma.
at upper pole of only one
lobe
Total thyroidectomy. Whole thyroid gland Diffuse MNG, thyroid
excised malignancy.

Q. What are the complications of thyroid operation?

Ans:

Anaesthetic: Intubation failure, cardiac arrest, respiratory arrest.

147 SHARIFUL HALIM, K66, DMC


Sharifs personal notes

Per-Operative:
Excessive bleeding: Primary hemorrhage.
Trachea, larynx.
Injury to recurrent laryngeal nerve.
Post-operative:
Reactionary hemorrhage within 24 hours of operation.
Bilateral recurrent laryngeal nerve palsy.
Tracheomalacia: Softening and thinning of trachea.
Hypothyroidism.
Hypoparathyroidism.

Q. What are the common post-operative complications of thyroid surgery which needs immediate
treatment?

Ans:

Hemorrhage/Hematoma with or without pressing over trachea.


Parathyroid insufficiency.
Thyrotoxic crisis.
Respiratory obstruction due to bilateral recurrent laryngeal nerve palsy.

Q. What is the blood supply of thyroid gland?

Ans:

Superior thyroid artery: Supply capsule and peripheral portion. Superficial bleeding.
Inferior thyroid artery: Supply remaining whole gland.

Q. What are the tumor markers in thyroid malignancies?

Ans:

Medullary carcinoma: Calcitonin.


Other differentiated carcinoma: Thyroglobulin.

Q. How will you manage a case of solitary thyroid nodule?

Ans:

DDx:

Thyroid adenoma.
Cyst.
Part of MNG.
Carcinoma.

Clinical features:

148 SHARIFUL HALIM, K66, DMC


Sharifs personal notes

Thyroid adenoma: painless, slow growing lump, soft to firm, not fixed with overlying or
underlying structure.
Cyst: Cystic in consistency, fluctuant, slows growing or non-progressive.
MNG: Surface bosselated, consistency variegated, features of upper airway obstruction may
be present. H/O Endemicity, iodine deficiency, intake of goitrogens.
Carcinoma: Lump- surface- irregular, hard in consistency, fixed, enlargement of regional LN.
Wt. loss, anemia, anorexia.

Investigations: Std. investigation.

Treatment:

Thyroid adenoma or cyst: Lobectomy.


MNG: Subtotal, near total or total thyroidectomy.
Carcinoma:
Papillary: Total thyroidectomy with neck dissection followed by suppressive
therapy.
Follicular: Total thyroidectomy followed by suppressive therapy with or without
chemotherapy.

BREAST

Benign breast diseases:

Considered as aberration rather than true disease.


Most occur during development, cyclical activity or involution.
Classification:
ANDI: Aberration of normal development and involution.
Age Normal process Aberration
<25 years Breast development Juvenile hypertrophy,
Fibroadenoma.
25-40 years Cyclical activity Cyclical nodularity, cyclical
mastalgia.
40-55 years Involution Cyst, Ductal ectasia.

Inflammatory or infectious process:


Abscess: Acute-Pyogenic, Chronic- TB, syphilis.
Periductal mastitis.
Fact necrosis.
Congenital disorder:
Inverted nipple.
Supernumerary breast/nipple.
Risk of development of malignancy in different benign breast disease:
No risk:
Adenosis.
Cyst.

149 SHARIFUL HALIM, K66, DMC


Sharifs personal notes

Fibroadenoma.
Duct ectasia.
Periductal or inflammatory mastitis.
Slightly increased risk:
Hyperplasia.
Papilloma.
Moderately increased risk:
Atypical hyperplasia.
Management of benign breast disease:
Conservative: Reassurance and follow up. E.g. Young patient (<30 years) with small
lump.
Surgery: Larger lump, new lump on older lump and older patient (>30 years):
removal of lump and biopsy.

Q. What are the causes of painful breast diseases?

Ans:

Mastitis.
Breast abscess.
Fibroadenosis.
Cyst.
CA breast.

Q. Write down the management of breast abscess?

Ans:

C/F:

Throbbing pain.
Fever with chills and rigors.
Tender lump in the breast.
Features of inflammation over the swelling.
Tender axillary lymphadenopathy.

Investigations:

CBC with ESR.


Pus for C/S.

Treatment:

Incision and drainage of abscess by radial or circumareolar incision.


Analgesics.
Antibiotics.
Antipyretics.
Regular dressing of the wound.
If lactating- rest to the breast.
Bed rest.

150 SHARIFUL HALIM, K66, DMC


Sharifs personal notes

Breast support.

Q. What are the causes of breast lump?

Ans:

Breast abscess.
Fibroadenoma.
Fibrocystic disease.
Ductal papilloma.
Fat necrosis.
CA of breast.

Q. A young lady of 30 years presented to you with a lump on her right breast recently. How will you
proceed to diagnose such a case?

Ans:

History:

Duration of lump.
Rate of enlargement.
Family history of CA breast.
Pain.
Blood stained discharge.
Local skin change.
Nipple retraction.
Mass in the axilla.
Wt. loss, anemia, anorexia.
Features of mets: Jaundice, bone pain, cough, hemoptysis, chest pain, focal symptoms.

Examination:

Examination of lump: Site, size, surface, shape, number, fixity, overlying skin, tenderness,
ulceration, discharge.
Examination of axillary lymph node: Enlarged/not, fixed/mobile.
Examination of Respiratory, HBS, MKS and nervous system: To see features of metastasis.
General examination: See anemia, jaundice, lymphadenopathy, dehydration, wt. loss.

Investigations:

USG of breast.
FNAC/ Core biopsy.
If LN enlargement- Biopsy or FNAC.

151 SHARIFUL HALIM, K66, DMC


Sharifs personal notes

To see evidence of mets: LFTs, USG of Abdomen, CXR, Bone scan.

Treatment: According to cause.

Q. What is triple assessment?

Ans:

It is a standardized procedure which helps to diagnose a case of breast lump and consists of:

Clinical assessment: Age, examination.


Radiological imaging: USG, mammography.
Tissue for cytological and histological analysis: FNAC, core-cut

CA breast

RF of breast cancer:
Age: Risk increases with age.
Gender: Female.
Genetic: Family history of CA breast; BRCA I, II positive.
obesity
Hormonal influences:
Early menarche and late menopause: increase duration of hormone
exposure.
OCP.
HRT.
SES: Higher class.
Avoidance of breastfeeding.
Diet:
Phytoestrogen is protective.
Saturated fat, red meat, drinking alcohol and increases the risk.
Previous benign disease: Atypical hyperplasia.
Geographical: More common in western countries.
H/O radiation.
Classification of breast cancer:
In situ/non-invasive:
Ductal carcinoma in situ (DCIS).
Lobular carcinoma in situ (LCIS).
Invasive:
Invasive ductal carcinoma/Scirrhous carcinoma.
Invasive lobular carcinoma.
Medullary carcinoma.
Papillary.
Colloid carcinoma.
Tubular carcinoma.
Spread of breast cancer:
Local spread: Skin of breast, chest wall, ribs.

152 SHARIFUL HALIM, K66, DMC


Sharifs personal notes

Lymphatic spread:
Blood borne metastasis:
Bones: Lumbar vertebra, thoracic vertebra, ribs, femur, humerous, skull.
Liver, lung, brain.
Clinical presentation:
Breast lump: Most commonly in upper and outer quadrant of breast.
Nipple retraction.
Nipple discharge: May be blood stained.
Ulcer over breast.
Edema: May present with Peau D orange.
Investigations:
For diagnosis:
USG of breast.
FNAC/Core biopsy.
For staging:
CXR: Secondary deposit or consolidation, pleural effusion, lytic lesion in
bony thorax.
USG of W/A: Liver mets, LN, ascites, pelvic deposit.
LFTs.
Radionuclide bone scan+ S. Calcium + S. ALP.
For G/A and operative fitness:
Common investigations.
Staging of breast cancer:
TNM staging:
T:
o T1: <2cm.
o T2: 2-5cm.
o T3: >5 cm.
o T4: Any size involving skin or chest wall.
N
o N0: No nodal metastasis.
o N1: Mobile axillary lymph node.
o N2: Fixed axillary LN.
o N3: Ipsilateral supraclavicular LN/internal mammary nodes.
M: M1 distant metastasis.

Manchester classification:
Stage I: Tumor <2cm.
Stage II: Tumor 2-5 cm with or without palpable mobile LN.
Stage IIIa: Tumor >5 cm with or without fixed palpable LN in axilla.
Stage IIIb:
o Skin involvement.
o Paeud orange.
o Fixation to underlying muscle.
o Palpable SC LN.
Stage IV= Distant metastasis.

153 SHARIFUL HALIM, K66, DMC


Sharifs personal notes

Prognostic factors of breast cancer:


Age: <35 years; poor prognosis.
Obesity: Bad.
Stage: Early stage-I, II: Good prognosis, Late stage- III,IV: Poor prognosis.
Grade: Grade: I, II- Good; Grade III- Bad.
Histologic type: Ductal- Good; Lobular- Bad.
Type of carcinoma: Invasive and inflammatory CA are bad.
Hormone receptor status:
HER2 positive- Poor prognosis.
ER positive- Good prognosis.
Treatment of breast cancer:
Stage I: <2cm.
Simple mastectomy with axillary sampling.
Adjuvant chemotherapy.
Hormone therapy: If ER +ve.
Stage II: 2-5 cm
Pateys or modified radical mastectomy.
Adjuvant chemotherapy: Chemotherapy: FAC= 5 FU, Adriamycin,
Cyclophosphamide. If any cardiac dysfunction Adriamycin is replaced by
epirubicin
Hormone therapy: If ER +ve: Hormone therapy: esp. if receptor status
positive. Drugs used are: Tamoxifen, DES, anastrazol.
Stage III: >5 cm
o With mobile axillary LN: Neo-adjuvant chemotherapy followed
by simple mastectomy with axillary clearance followed by
adjuvant chemotherapy. If Receptor status positive-Hormone
therapy.
o With fixed palpable LN: Only difference is axillary radiation.
o With fungating ulcer:
Toilet mastectomy.
Chemotherapy.
Hormone therapy.
Radiotherapy.

Metastatic CA breast: Stage IV


o Systemic chemotherapy.
o Hormone therapy.
o Immunotherapy: Transtujumab- Ab against HER 2 receptor.
o Radiotherapy: Controlling local disease or painful bony
metastasis.
o Surgery: toilet mastectomy.

Q. What are the investigations done in fresh specimen of breast?

Ans:

Frozen section biopsy.


Receptor status.
Tumor marker.

154 SHARIFUL HALIM, K66, DMC


Sharifs personal notes

Q. Name some incisions done in case of breast lump.

Ans:

Benign: Circular, radial, sub mammary.

Malignant: Stewart- Transverse elliptical, Haelsteadt- Elliptical.

Q. What are the parts of simple mastectomy?

Ans:

Removal of breast including nipple and areola with skin over tumor.
Axillary tail of breast.
Few nodes attached to axillary tail.

Q. What are the components of Pateys mastectomy/ Modified radical mastectomy?

Ans:

Whole breast.
Large portion of skin: Centre of which overlies the tumor, always including the nipple.
All of the fat fascia and LN of axilla.

Radical/Halsted mastectomy:

Breast.
Axillary lymph nodes.
Pectoralis major and minor muscles.

Q. What are the causes of nipple discharge?

Ans:

CA breast.
Fibrocystic disease.
Breast abscess.
Duct ectasia.
Pagets disease.

155 SHARIFUL HALIM, K66, DMC


Sharifs personal notes

SKIN

Q. Name the premalignant and malignant skin lesions?

Ans:

Pre-malignant:

Leukoplakia.
Bowens disease.
Dysplastic naevi.
Xeroderma pigmentosum.
Solar actinic keratitis.

Malignant lesions:

Squamous cell carcinoma.


Basal cell carcinoma.
Malignant melanoma.
Pagets disease of breast.
Merkel cell carcinoma.

Q. Write down the clinical features and management of basal cell carcinoma?

Ans:

C/F:

Site: Face above the line joining the angle of mouth to ear lobule.
Edge: Raised and rolled.
Floor: deep.
Discharge: Serosanguinous/ blood stained.
Base: Indurated, may be fixed with underlying structures.
Draining LN: May be enlarged.

Investigations: Biopsy for histopathology.

Treatment:

Surgical excision with .5 cm extra margin.


Radiotherapy.

Q. SN on SCC carcinoma.

Ans:

Clinical features:

156 SHARIFUL HALIM, K66, DMC


Sharifs personal notes

Edge: Raised and Everted.


Floor: Greyish white slough, may bleed on touch.
Discharge: Serosanguinous or blood stained.
Base: Indurated, may be fixed with underlying structure.
Draining LN: May be enlarged and hard.

Treatment:

Surgical excision: With 1cm free margin.

Q. How will you clinically differentiate between BCC and SCC?

Ans:

Point BCC SCC


Site Upper part of the face Any site
Edge Raised, rolled Raised, everted
Floor Deep Contains grayish white slough
Shape Circular Irregular.
Rate of growth Rapid Slower
Radiotherapy Sensitive Resistant .

Q. What are the criteria of a malignant mole/malignancy of benign mole?

Ans:

Major: Change in size, shape, color.

Minor: Inflammation, bleeding, itching.

FACE

Q. Write a short note on cleft lip.

Ans:

Definition: It is a developmental anomaly that results from failure of fusion between frontnasal and
maxillary process.

Classification:

Central:
Lateral: Uni/bilateral, complete/incomplete.

C/F:

Wt. loss, pallor.


Cleft lip in local examination.

Treatment:

157 SHARIFUL HALIM, K66, DMC


Sharifs personal notes

E & R.
Time of surgery:
All over world: Wt. 10 lbs., Hb-10g/dl, age-10 weeks.
BAN: Wt. 5 lbs., Hb-5g/dl, age- 5week.
Methods of surgery:
Tension triangular method.
Millard rotation.

Complications:

Disfigurement.
Difficulty is suckling.
Difficulty in speech.
Anemia.
Wt. loss.

PVD:

1. Involves: Artery, vein & lymphatics.


2. Etiology/ RFs:
Smoking.
DM.
HTN.
High cholesterol.
Age.
Obesity.
3. Types:
Arterial Disease:
Buergers disease/ Thromboangitis obliterans. (May also involve veins & sometimes
lymphatics):
Diabetic foot (Ulcer, gangrene, microangitis obliterans).
Pre-senile atherosclerosis.
Less common: Embolism, Raynauds disease, systemic vasculites.

Venous disease:

DVT.
Varicose vein.
Venous ulcer.

Lymphatic disease: Uncommon.

Lymphangitis.
Lymphedema.

Q. What is Buerger s disease/Thromboangitis obliterans?

158 SHARIFUL HALIM, K66, DMC


Sharifs personal notes

Ans:

It is a progressive peripheral veso-occlusive disease where Angitis/vasculites/non-specific inflammation


and thrombosis leads to obliteration of the lumen of the small to medium sized arteries.

Angitis- thrombosis- obliteration of lumen- ischemia- gangrene- ulceration.

Symptoms and signs:

Pain:
Intermittent claudication.
3 criteria:
i. No pain at onset of starting walking or exercise. (Pain at > Osteoarthritis.)
ii. Pain arises after a distance of walking or strenuous exercise.
iii. Subsides after stoppage of walking or taking rest. (Pain that
doesn t subside after taking rest & analgesics are needed=
Muscle/tendon pain).
Cramp like/bursting, uniform pain in the muscle ie in the calf of lower
limb & forearm muscles in case of upper limb.
Doesn t develop suddenly.
Doesn t relieve suddenly.
Not associated with systemic manifestations/constitutional symptoms.
Stages of claudication:
i. Mild: Patient can continue walking or exercise with pain. (Also
Claudication distance >500m)
ii. Moderate: Patient must take some rest to relive the pain.
(Claudication distance 500-100m)
iii. Severe: He must need any analgesic/ must go to bed. (
Claudication distance <100m)
Rest pain:
Pain is always present- symptom of severe ischemia.
Patient cant do anything.
Muscle wasting is primarily due to ischemia.
Site of pain: Most distal part of limb/more parts of whole of the limb
according to degree & level of obstruction.
Sleeplessness: Invariably present.
Patient attitude: Pt. hangs his leg to cause venous pooling in the lower
limb which causes 2 work:
i. Partial oxygenation of lower limb from venous blood.
ii. Inadequate wash out of lactic acid, K & NO which causes more
vasodilation & more pooling of blood causing more
oxygenation.
Pathogenesis of rest pain/ intermittent claudication:
Angitis, thrombosis & gradual obliteration of medium & small sized
vessels along with loss of contractile or relaxing power.
Exercise or walking increased muscle oxygen demand but the vessels
can t supply/meet the demands so muscles suffer from ischemia.
These causes pain in the muscle.

159 SHARIFUL HALIM, K66, DMC


Sharifs personal notes

Pre-gangrene:
Regular Features: Pain, pallor, Pulselessness, paresthesia, paralysis (These are
also cardinal features of acute ischemia), decreased temperature (Perishing cold).
Definite features: hair loss, loss of skin texture, color change.
If we can reperfuse the limb (ie by graft or stenting) at this stage, the limb can be
saved.
Gangrene:
Any type of gangrene may occur:
Dry gangrene: Only arterial involvement.
Moist/Wet gangrene: More venous involvement.
Mixed gangrene: Arterial & venous involvement.
Ulceration:
Types: Cutaneous, End ulceration.
Cutaneous: Commonly occurs at the dorsum of the foot. Occurs as a sequel of
cutaneous ischemia. We can heal the ulcer or give SSG.
End ulceration: Ie in toe. Occurs due to sloughing out of gangrenous part. NO
chance of healing or grafting.
Wasting of the limb.
Hair loss.
Numbness.

Difference between osteoarthritis & rest pain/Claudication:

Point Rest pain/Claudication Osteoarthritis.


Onset Claudication starts after Starts with initiation of
walking some distance walking.
Site of pain Muscle In joints.
ROM (Range of Normal Reduced.
movement)
Working capacity. Can t do anything. Can do something.
Muscle atrophy Primarily due to ischemia Due to disuse.
Age of pt. Middle age Elderly.

Ischemic ulcer:

Site:
Tip of the toes or fingers- end ulceration.
Dorsum of the foot- cutaneous ulceration.
Pressure points: ie heel, below first and fifth metatarsal.
Size: vary.
Shape: Elliptical/round.
Number.
Tenderness: May be present esp. in the surrounding region.
Edge: Punched out primarily, and then sloping in the healing stage- in case of cutaneous
ulcers.
Floor: Contains grayish yellow slough.
Base: May be slightly indurate.
Discharge: Can be clear fluid, serosanguinous or pus.

160 SHARIFUL HALIM, K66, DMC


Sharifs personal notes

Surrounding tissue: may be ischemic- cold, and tender. Often blackish and edematous, hair
loss, loss of texture.
Pulses: invariably absent up to some level.
Lymph nodes: not enlarged.
Neurological examination: Jerks normal or reduced, muscle wasting, power decreased
G/E may show evidence of cardiovascular or CNS manifestation of vascular disease.

Investigations:

a. Specific: Color Doppler USG/Duplex study/Digital Subtraction Angiography of Lower limb


vessels.
Site of occlusion.
Size.
Flowing of blood.
Condition of arterial wall.
Venous conditions.
b. Ankle brachial index: ABI <.9 indicates ischemia of the lower limb.
c. RBS.
d. For confirmation: Vessel biopsy.
e. Other tests: Common tests- CBC, CXR, ECG, ECHO, S. lipid profile, Blood RFTs.

Treatment of PVD:

1. Lifestyle modification:
i. STOP SMOKING: Pain is clearly advised about the importance of quitting. Quitting
halts the disease process & reduces pain.
ii. Exercise/Discipline: Ie esp. @ bed, ie raising, foot exercise, foot massage, exercises
in working place. Buergers exercise: Elevation of limb at 45 degree for 3 minute &
keep in horizontal for 2 minutes, repeated for about 30 minutes; done 2-3 times a
day. Aim is improvement of blood supply and reduction of body weight.
iii. Diet: Low in CHO & fat. High in protein- Increase in muscle bulk- Increased
pumping.
iv. Toe- nail must be carefully trimmed.
v. Regular use of custom made shoes: Soft and fitting.
2. Treatment of any comorbid condition: ie DM, HTN, Hyperlipidemia.
3. Drugs:
i. Simple analgesics: Start with Paracetamol, then aspirin, then ibuprofen, later
opioids.
ii. Peripheral vessel selective vasodilator: Pentoxyphylline, Nifedipine.
iii. Vitamin C& E: Work as antioxidant.
iv. Antibiotics.
4. Surgical:
i. In pre-gangrenous stage: Vascular graft (Vein or inert plastic material e.g.
Darcon), stenting, percutaneous transluminal angioplasty (PTA).
ii. Ulcer: Dressing, toileting, SSG or flap.

161 SHARIFUL HALIM, K66, DMC


Sharifs personal notes

iii. If unremitting claudication or rest pain that cant be controlled by analgesics:


Lumbar or thoracic sympathectomy. In lumbar sympathectomy: L 1, 2, 3, 4 are cut
in the same side of lesion (L1 is spared in opposite side to preserve erectile
function). Sympathectomy causes denervation of arteriole & venule so dilation of
cutaneous vessels & relief of pain occurs. It also limits the extent of amputation
required. Done thru a transverse loin incision.
iv. Removal of gangrenous part/amputation:
Indication: In extensive gangrene, rest pain.
Extensive gangrene: Done along with sympathectomy.
Rest pain: May be done done 4-6 weeks after sympathectomy.
The level of amputation is determined by local blood supply, which is
assessed by examination of pulse, temperature. It can be aided by
thermography and percutaneous oxygen measurement.

Q. What is Buergers angle?

Ans:

It is the angle between the elevated limb up to the appearance of pallor & the horizontal plane.

Q. Write a short note on amputation.

Ans:

Definition: Surgical removal of dead or non-functioning limb.

Indication:

Dead limb: DM gangrene, Buerger s disease.


Deadly: Moist gangrene, gas gangrene.
Dead loss: Lacerated injury, crushed injury.

DVT:

Q. Write a SN on DVT.

Ans:

Definition: Formation of semisolid thrombus in deep veins.

RFs:

Post-surgical: Hip/pelvis surgery, knee surgery, other major surgery.


Immobilization.
Age: More than 60 years.
Obesity.

162 SHARIFUL HALIM, K66, DMC


Sharifs personal notes

Pregnancy.
Varicose veins.
Polycythemia.
Nephrotic syndrome.

Clinical features;

Symptoms: Swelling, pain, fever, malaise if pulmonary embolization- cough, hemoptysis.


Signs:
Tender swelling.
Redness, edema, engorged superficial veins.
Homan s sign: Pain in the calf muscle upon dorsiflexion of foot/ankle.
Fever, tachycardia.

Investigations:

USG of limb.
Doppler study.
CXR.
CBC with ESR.
Coagulation profile.

Treatment:

LMW heparin for 5 days.


Warfarin 3-6 months.
If recur lifelong warfarin.
General: Antipyretic, analgesics, mobility, fluid intake increased.

Prevention:

Hydration.
Compression stocking.
Calf pumps.
Early mobilization.
Prophylactic LMW heparin.

Complications:

Local: Edema, ulcer, gangrene, varicose vein.


Pulmonary embolism.

163 SHARIFUL HALIM, K66, DMC


Sharifs personal notes

Q. Write a short note on CTEV/ Club foot.

Ans:

It is a congenital deformity of foot characterized by:

Talipes: Adduction and supination of forefoot.


Equinus: Plantar flexion of the ankle.
Varus: Internal rotation/inward twisting of hindfoot.

Pathology:

Small heel.
Short tendoachilles.
Soft tissue contracture of the foot: Especially in the posteromedial aspect.

Investigations:

X-Ray of foot in best possible correction A/P and Lateral view.

Treatment:

Conservative: By Ponsetti method:


Regular manual manipulation.
Serial plaster casting.
Splinting by Ponsetti brace shoe.
Surgical:
Postero-medial release.
Lengthening of the tendoachilles.

Q. What is fracture? Classify fracture.

Ans:

Fracture:

Soft tissue injury where bones happen to be broken.

Classification:

A. Clinical:
i. Simple or closed fracture.
ii. Compound or open.
B. According to fracture configuration:
i. Transverse.

164 SHARIFUL HALIM, K66, DMC


Sharifs personal notes

ii. Oblique.
iii. Spiral.
iv. Comminuted.
v. Avulsion.
C. According to cause:
i. Traumatic.
ii. Pathological.
iii. Stress- normal bone stressful condition- foot, metastases, tibial shin.
iv. Avulsion: sudden violent contraction of muscles against resistance- triceps and
olecranon, quadriceps and patella. Ie in Grand Mal seizure/ GTCS.

Q. What is Green-stick fracture?

Ans:

It is a childhood fracture of the young and soft bone where bone bends and partially breaks.

Q. What is pathological fracture?

Ans:

Definition: It is a type of fracture that occurs in bones weakened by local or systemic skeletal disease.

Causes:

A. Local disease:
I. INFECTION:
a. Pyogenic osteomyelitis.
b. Tubercular osteomyelitis.
II. BENIGN TUMOR:
a. Chondroma.
b. Giant cell tumor.
c. Simple bone cyst.
d. Aneurysmal bone cyst.
III. MALIGNANT TUMOR:
a. Osteosarcoma.
b. Ewings tumor.
c. Metastatic tumor.
B. Systemic disease:
i. Senile osteoporosis.
ii. Hyperparathyroidism.
iii. CKD.
iv. Multiple myeloma.
v. Rickets.
vi. Scurvy.

Q. What is compound fracture? How will you manage a case of compound fracture?

Ans:

165 SHARIFUL HALIM, K66, DMC


Sharifs personal notes

Compound fracture: IT is type of fracture where fracture hematoma communicates with the exterior.

Management of the compound fracture:

Primary survey and resuscitation:


Airway and cervical spine protection:
Breathing and ventilation: O2 inhalation.
Circulation and control of bleeding: IV cannulation, Fluid and Blood transfusion.
Disability.
Management of the fracture:
Recognition:
a) Fracture site and type.
b) Soft tissue injury.
c) Distal neurovascular status.
d) Contamination.
Reduction.
Retention: Immobilization by internal fixation (Less contamination, simple
fracture and less soft tissue loss) or external fixation (High contamination,
comminuted fracture, extensive tissue loss).
Rehabilitation: By physiotherapy.
Management of the wound:
Thorough surgical toileting.
Wound debridement.
Repair of any damaged structure: Vessel, tendons, nerves.
Broad spectrum antibiotics.
Analgesics.
TIG.
Anti-gas gangrene serum.
Wound closure:
Tissue injury, contamination and edema less: Primary or delayed
primary closure.
Extensive tissue loss and contamination: Secondary closure.

Q. What are the common fractures of the childhood?

Ans:

i. Clavicle fracture.
ii. Supra-condylar fracture.
iii. Monteggia fracture.
iv. Green stick fracture.
v. Hair line fracture.

Q. What are the common clinical features of fractures?

Ans:

A. Symptoms:
i. Pain.
ii. Swelling.

166 SHARIFUL HALIM, K66, DMC


Sharifs personal notes

iii. Deformity.
iv. Restriction of range of movement.
B. Signs:
i. Tenderness.
ii. Swelling.
iii. Deformity.
iv. Restriction of movement.

Q. Write down the principle of treatment of fracture.

Ans:

Primary survey and resuscitation:


Airway and cervical spine protection:
Breathing and ventilation: O2 inhalation.
Circulation and control of bleeding: IV cannulation, Fluid and Blood transfusion.
Disability.
Management of the fracture:
Recognition:
e) Fracture type.
f) Soft tissue injury.
g) Distal neurovascular status.
Reduction: Realignment of the fractured fragments to restore normal
anatomy. Open or closed method.
Retention: Maintenance of reduction:
a) Plaster cast.
b) Slab.
c) Traction: Cutaneous or skeletal.
d) Internal fixation.
e) External fixation.
Rehabilitation: By physiotherapy, psychotherapy and vocational
rehabilitation.
Management of the wound:
Thorough surgical toileting.
Wound debridement.
Repair of any damaged structure: Vessel, tendons, nerves.
Broad spectrum antibiotics.
Analgesics.
TIG.
Anti-gas gangrene serum.
Wound closure:
Tissue injury, contamination and edema less: Primary or delayed
primary closure.
Extensive tissue loss and contamination: Secondary closure.

Duration of immobilization in fracture:

Age group Upper limb. Lower limb.


Child 3 weeks 6 weeks.

167 SHARIFUL HALIM, K66, DMC


Sharifs personal notes

Adult 6 weeks 12 weeks.

Q. Write down the complications of fracture.

Ans:

I. Immediate:
a) Hemorrhage.
b) Shock.
c) ARDS.
d) Renal failure.
e) Injury to muscles, nerves, vessels, tendons.
f) Injury to internal organs.
g) Compartment syndrome.
h) Fat embolism.
i) Bone marrow embolism.
II. Delayed:
a) Osteomyelitis.
b) Septic arthritis.
c) Septicemia.
d) Mal-union.
e) Non-union.
f) Deformity.
g) Joint stiffness.
h) Volkmanns ischemic contracture.

**Rule of two in x rays:

a. Two joints: e.g. X ray of Rt. leg with knee and ankle joint.
b. Two views: AP and lateral. Any contusion X ray of opposite limb.
c. Two limbs: Compare any injury.
d. Two occasions: Neck fracture, scaphoid fracture.
e. Two physicians.

Q. What is dislocation?

Dislocation: loss of normal anatomical relationship between two bones forming a joint. Any
dislocation is an orthopedic emergency. Relocate or reduction.

Q. What is subluxation?

Ans: Partial loss of anatomical relationship between two bones forming a joint.

Q. Compare fracture and dislocation?

Ans:

Points Fracture Dislocation

168 SHARIFUL HALIM, K66, DMC


Sharifs personal notes

Definition Soft tissue injury where bones Loss of normal anatomical


happen to be broken relationships of bones forming a
joint
Loss of continuity of bones Occur NO
Disruption of joint Doesn t occur Occurs
Cause Trauma, pathology in the bone, Trauma, congenital
stress
Recurrence Uncommon Common
Treatment Resuscitation and recognition, Reduction and rest.
reduction, retention and
rehabilitation

Q. Write a short note on Erbs palsy.

Ans:

Definition: It is lesion in the upper trunk of the brachial plexus that occur due to movement of the
head to the opposite side and depression of the shoulder on same side.

th th
Site of lesion: 5 and 6 cervical cranial nerve fibers.

Involved muscles:

Deltoid.
Biceps.
Brachialis.
Brachioradialis.

Cause:

Birth injury: Shoulder dystocia.


Fall on shoulder.
Anesthetic complication.

Typical posture:

Police man tip hand/ waiter s tip hand.


Shoulder: Adducted and internally rotated.
Forearm: Pronated.
Wrist: Flexed.

Investigations:

EMG/ NCV.
MRI.

Treatment:

Passive range of motion exercise.

169 SHARIFUL HALIM, K66, DMC


Sharifs personal notes

Electrostimulation.
Microsurgical reconstruction.
Nerve graft.

Q. Write down the different methods of immobilization/retention?

Ans:

Splinting:
Plaster of Paris cast.
Slab.
Traction: Cutaneous or skeletal.
External fixation:
Uniplanar.
Multiplanar.
Illizarov external fixator.
Internal fixation:
Plate and screws.
Intramedullary nail.
Kirschner wire/ K-Wire.

Q. Write a short note on plaster of Paris.

Ans:

Plaster of Paris: It is an external splinting material containing calcium sulphate hemihydrate


(CaSO4.1/2 H20).

Mechanism of action: When it is soaked in water, it liberates heat and then hardens.

Use: As a casting material for immobilization in closed, un-displaced, simple and stable fractures.

Complications:

Compartment syndrome.
Volkmanns ischemic contracture.
Irritation of skin.
Itching.
Ulceration.
Muscle atropy.
Joint stiffness.
Osteoporosis.

Q. Write a short note on internal fixation.

Ans:

Methods:

Plates and screws.


Intramedullary nail.

170 SHARIFUL HALIM, K66, DMC


Sharifs personal notes

K- Wire.

Indication:

Open fracture: When less contaminated, less tissue loss, not comminuted.
Highly unstable fracture.
Displaced Fracture involving articular surface.
Fractures that cant be reduced.

Complications of internal fixation:

Hemorrhage.
Injury to soft tissues.
Injury to blood vessels and nerves.
Osteomyelitis.
Fat embolism.
Bone marrow embolism.
Arrest of bone growth.
Failure: Mal-union, non-union, delayed union.

Q. Write a short note on non-union of bone/fracture.

Ans:

Definition: When a minimum of nine months has passed after the initial injury and the fracture shows
no radiologically visible signs of progressive healing continuously for 3 months, the condition is
termed as non-union.

Types:

Atrophic: Fracture gap filled by fibrotic tissue.


Hypertrophic: Fracture gap filled by hypertrophied products of healing.
Infected.

Cause:

Local:
Improper reduction and retention technique.
Soft tissue interposition.
Inadequate blood supply.
Infection.
Foreign body.
Systemic:
Anemia.
Malnutrition.
DM.
Smoking.
Steroid.

171 SHARIFUL HALIM, K66, DMC


Sharifs personal notes

TB.
Osteoporosis.

SPECIFIC FRACTURES:

Q. How will you manage a case of fracture clavicle?

Ans:

Clinical features: Std. features.


Investigations: X-Ray of clavicle.
Treatment:
Un-displaced fracture: Triangular sling with elbow bag for 6 weeks in adult
and 3 weeks in child.
Displaced fracture: Reduction followed by immobilization.
Analgesics with PPI.
Advice: Progressive passive and then active range of movement of elbow and
shoulder.

Q. How will you manage a case of supracondylar fracture?

Ans:

Clinical features: Std features.


Investigation: X-Ray of elbow joint with lower part of humerous and upper ends of
radius-ulna A/P and lateral view.
Treatment:
Undisplaced fracture: Long arm back slab for 3 weeks in children.
Displaced fracture: Reduction by traction and counter traction under G/A
followed by long arm back slab with elbow and <90 degree for 3 weeks in
children.

Q. What are the complications of supracondylar fracture?

Ans:

Immediate:
Hemorrhage.
Injury to the brachial artery, radial nerve, median nerve.
Compartment syndrome.
Delayed:
Mal-union.
Non-union.
Joint stiffness.
Deformity.
Volkmanns ischemic contracture.

COLLES fracture:

172 SHARIFUL HALIM, K66, DMC


Sharifs personal notes

Case: A 50 Y/O lady fell on out-stretched hand and noticed swelling of the right fore-arm just above
the wrist. What is the most likely diagnosis? Write down the management of the patient.

Ans:

Diagnosis: Colles fracture.

Definition: Fracture of the lower end of the radius about one inch above the lower articular
surface.
Clinical features: Std. features+ Deformity-Dinner fork deformity.
Investigation: X-Ray wrist A/P and lateral views including distal parts of radius and ulna and
proximal part of hand.
Typical X-Ray findings:
Fracture and the lower part of radius.
Postero-lateral displacement of distal fragment.
Upward pulling of distal fragment.

Treatment:
Undisplaced fracture:
First 24 hours- Immobilization by short arm back slab.
Next 6 weeks: Colles full plaster- extends from elbow to knuckles of
hand.
Displaced fracture:
Closed reduction under G/A.
Colles plaster for 6 weeks.

FRACTURE NECK FEMUR:

Q. Classify fracture neck of femur.

Ans:

Intra-capsular fracture:
Garden Type I: Incomplete fracture.
Garden Type II: Complete fracture without displacement.
Garden Type III: Complete fracture with partial displacement.
Garden Type IV: Complete fracture with complete displacement.
Extra-capsular fracture:
Trochanteric fracture.
Sub-trochanteric fracture.

Q. Describe the management of a case of fracture neck of the femur.

Ans:

History: Of trauma.

173 SHARIFUL HALIM, K66, DMC


Sharifs personal notes

Symptoms:
Std. symptoms (Pain, swelling, deformity, restricted movement).
Signs:
Std. signs (Tenderness, swelling, deformity- Limb is abducted, externally rotated and
shortened, restriction of movement.
Investigations:
X-Ray of pelvis including both hip joints.
CT scan of pelvis.
Treatment:
Undisplaced fracture: Hip screw.
Displaced fracture:
ORIF: Open reduction and internal fixation.
Hemiarthroplasty.
Total hip replacement.

Q. What are the complications of fracture neck of the femur?

Ans:

Internal hemorrhage.
Shock.
Avascular necrosis of head of femur.
Stiffness of hip joint.
Limb shortening.
Deformity.
DVT and PE.

FRACTURE SHAFT OF FEMUR:

Q. How will you manage case of fracture shaft of femur?

Ans:

History: Trauma.
Symptoms: Std symptoms+ Shock.
Sign: Std signs+ Shock.
Investigation: Plain X-Ray of femur including hip and knee joint.
Treatment:
Undisplaced fracture:
Adult: Skeletal traction for 4 weeks followed by long leg full plaster for
8 weeks. (Total of 12 weeks)
Children: Surface traction followed by hip spika.
Displaced fracture: ORIF with intramedullary nail.

FRACTURE OF TIBIA:

Q. How will you manage a case of open fracture of tibia?

Ans:

174 SHARIFUL HALIM, K66, DMC


Sharifs personal notes

History: Trauma.
Symptoms and signs: Std. + Shock.
Investigation: Plain X-Ray of leg with knee and ankle joint.
Treatment:
Primary survey and resuscitation:
Airway and cervical spine protection:
Breathing and ventilation: O2 inhalation.
Circulation and control of bleeding: IV cannulation, Fluid and Blood transfusion.
Disability.
Management of the fracture:
Recognition:
h) Fracture type.
i) Soft tissue injury.
j) Distal neurovascular status.
Reduction: Under G/A.
Retention:
Less tissue loss, less contamination, non- comminuted fracture: ORIF
with plate and screw.
Excessive tissue loss, high contamination, comminuted fracture:
External fixation by Illizarov fixator.
Rehabilitation: By physiotherapy.
Management of the wound:
Thorough surgical toileting.
Wound debridement.
Repair of any damaged structure: Vessel, tendons, nerves.
Broad spectrum antibiotics.
Analgesics.
TIG.
Anti-gas gangrene serum.
Wound closure:
Tissue injury, contamination and edema less: Primary or delayed
primary closure.
Extensive tissue loss and contamination: Secondary closure.

OSTEOMYELITIS:

Q. What is osteomyelitis?

Ans:

Infection of the bone and bone marrow is called osteomyelitis.

Q. Classify osteomyelitis?

Ans:

1. Acute osteomyelitis: <2 weeks.


2. Sub-acute osteomyelitis: 2-3 weeks.

175 SHARIFUL HALIM, K66, DMC


Sharifs personal notes

3. Chronic osteomyelitis: >3 weeks.

Q. What are the causative organisms of osteomyelitis?

Ans:

1. Staphylococcus aureus.
2. Streptococcus.
3. Pneumococcus.
4. Hemophilus influezae.
5. Pseudomonas aeruginosa.
6. Proteus.
7. E. coli.
8. Klebsiella.
9. MTB.
10. Treponema pallidum.

Q. Write down the pathogenesis of acute osteomyelitis.

Ans:

Infection and suppuration at the metaphysical endosteum and cortex.


Percolation thru Haversian system to periosteum.
Subperiosteal abscess formation:
Lifts periosteum- ischemia and necrosis of cortex- dead bone/sequestrum formation.
May form a soft tissue abscess and a discharging sinus.

Q. How will you manage a case of acute osteomyelitis?

Ans:

Symptoms:
Bone pain.
Fever with chills and rigor.
Swelling.
Restriction of movement.
Sign:
Bony tenderness.
Swelling.
Discharging sinus.
Restriction of movement.
Fever, tachycardia.
Investigations:
CBC with ESR.
Blood C/S.
USG/ CT scan/ MRI/ Radionuclide scan of affected bone.
About 2 weeks after the onset of symptoms- X-Ray.
Discharge (If any) for C/S.
Treatment:
General:

176 SHARIFUL HALIM, K66, DMC


Sharifs personal notes

Bed rest.
Plenty of fluids.
Soft and nutritious diet.
Tepid sponging.
Antipyretics.
Analgesics.
Specific:
Empirical: Cloxacillin 500mg QDS + Gentamicin for 28 days.
Or antibiotics According to C/S.
If abscess: I&D= incision and drainage.

Q. What are the complications of acute osteomyelitis?

Ans:

Chronic osteomyelitis.
Pathological fracture.
Septic arthritis.
Septicemia, meningitis.
Deformity.
Arrest of bone growth.
Limb shortening.

BONE TUMORS:

Q. Classify bone tumors:

Ans:

1. Primary bone tumors:

Types Benign Malignant


Bone forming Osteoma, osteoid osteoma. Osteosarcoma.
Cartilage forming Chondroma Chondrosarcoma.
Bone marrow tumor Ewings sarcoma.
Multiple myeloma.
Giant cell tumor Giant cell tumor Malignant giant cell tumor
Fat forming Lipoma Liposarcoma
Fibrous Fibroma Fibrosarcoma
Vascular Hemangioma Angiosarcoma.

2. Secondary bone tumors: From


Lungs.

177 SHARIFUL HALIM, K66, DMC


Sharifs personal notes

Breast.
Prostate.
Kidneys.
Thyroid.

Q. Write a short note on Ewings sarcoma.

Ans:

Common site: At the diaphysis of the long bones.

Age: Less than 10 years.

Clinical features:

Pain.
Swelling.
Pathological fracture.
Restriction of movement.

Investigations:

X- Ray of the affected region:


Onio- peel appearance.
Widening of the shaft.
FNAC/ Biopsy with histopathology.

Treatment:

Combined radiotherapy and chemotherapy.

Q. What are the common sites of osteosarcoma?

Ans:

Around knee.
Upper end of humerous.
Lower end of radius.
Face.
Hip.

Q. Write down the management of a case of osteosarcoma.

Ans:

Clinical features:
Local: Pain, swelling, pathological fracture, neuromuscular damage.
Metastatic: Jaundice, cough, hemoptysis.
General: Wt. loss, anemia, anorexia, asthenia/weakness.
Investigations:
Plain X-Ray of the affected part:

178 SHARIFUL HALIM, K66, DMC


Sharifs personal notes

Mixed osteolytic and osteoblastic lesion at the metaphysis.


Periosteal reaction: Codmans triangle.
Sunray spicule/appearance.
CT scan/MRI.
Biopsy and histopathology.
Staging investigations:
CXR.
USG/CT scan of W/A.
LFTs.
Treatment:
Neo-adjuvant chemotherapy followed by amputation followed by adjuvant
chemotherapy.

Thoracic surgery:

Important topics to review:

1. Name some life threatening chest injuries.


Ans:
Tension pneumothorax.
Open pneumothorax.
Flail chest.
Massive Haemothorax.
Ruptured ventricle & Cardiac temponade.
Pulmonary contusion.
Aortic rupture.

2. What are the causes of airway obstruction following RTA?

Ans:

Causes of Airway obstruction following RTA:

Hemorrhage into the airway.


Expanding hematoma in the neck compressing the airway.
Foreign body.
Slipped artificial dentures.
Direct injury to the trachea & tracheal collapse.
Subcutaneous emphysema.

3. Describe the management procedure of airway obstruction in trauma.

Ans:

179 SHARIFUL HALIM, K66, DMC


Sharifs personal notes

Assessment of airway:

Fluent speech: If fluent speech production, no airway obstruction.


Stridor: Indicates upper airway obstruction.
Cough.

Treatment of airway obstruction:

Sequentially:

a. Suction & Clearance of the airway.


b. Head tilt- Chin lift maneuver.
c. Jaw thrust maneuver.
d. Protection of cervical spine.
e. Nasotracheal or orotracheal intubation: First line invasive management.
f. Crico-thyroidotomy: If intubation fails & no facility of tracheostomy.
g. Emergency tracheostomy.
h. Indirect or flexible laryngoscopy & removal of foreign body, if any.

Monitoring:

Speech.
Stridor.
Level of consciousness.
Pulse oximetry.
ABG.

Q. What are the components or presentations/complications of chest injury?

Ans:

Chest wall: Contusion, abrasion, bruise, rib fracture, flail chest, sternum fracture, vertebral fracture.

Pleural: Pneumothorax, Haemothorax, haemopneumothorax.

Lungs: Lung contusion.

Heart: Cardiac temponade, ventricular rupture.

Aorta: Dissection.

Esophagus: Rupture.

Diaphragm: Rupture.

Tracheo-bronchial tree: Penetrating wound, SC emphysema, pneumothorax.

180 SHARIFUL HALIM, K66, DMC


Sharifs personal notes

Q. Describe the boundaries of triangle of safety.

Ans:

Anterior to the mid-axillary line.


Above the level of nipple.
Below and lateral to the Pectoralis major.

Q. Define flail chest and paradoxical movement.

Ans:

Flail chest: It is a condition where several adjacent ribs are fractured in two sites in on side of the chest
or either side of sternum resulting in paradoxical movement of the flail segment.

Paradoxical movement: Movement of a flail segment of chest inward during inspiration and outward
during expiration.

Q. How will you manage a case of flail chest?

Ans:

Clinical features:

H/O Chest trauma.


Chest pain: Sharp localized, aggravated by breathing.
Respiratory distress.
Paradoxical movement of the flail segment.
Tenderness over the ribs.

Investigations:

CXR.
ABG.

Treatment:

ABC resuscitation.
Stabilization of the flail segment.
Thoracotomy and fracture fixation.
For lung contusion:
Use of colloids rather than crystalloids.
Diuretics.
If patient deteriorates: Mechanical ventilation with bilateral chest tubes.
Analgesics.
Antibiotics.
Wound care.

181 SHARIFUL HALIM, K66, DMC


Sharifs personal notes

Q. What is stove in chest?

Ans:

Depression of the chest wall due to multiple rib fracture.

Q. What is pneumothorax? What are the causes? Classify pneumothorax. Describe the management of
tension pneumothorax.

Ans:

Pneumothorax: Presence of air in the pleural space.

Causes:

Primary spontaneous: Rupture of apical pleural bleb.


Secondary spontaneous: TB, COPD.
Traumatic.
Iatrogenic: Bronchoscopy, mechanical ventilation, during chest tube insertion.

Classification:

Closed.
Open.
Tension pneumothorax.

Q. Write short note on tension pneumothorax.

Ans:

Definition:

Progressive accumulation of air in pleural space which causes shifting of mediastinum and
hemodynamic instability.

Pathogenesis:

Chest wall or lung itself acts a one way valve.


Air come into pleural space during inspiration and cant get out during expiration.
Progressive accumulation of air.
Shifting of mediastinum.
Hemodynamic instability.

Clinical features:

H/O: Trauma, COPD, TB, Intervention.

182 SHARIFUL HALIM, K66, DMC


Sharifs personal notes

Sx: Sudden severe chest pain, severe respiratory distress, features of shock.
Sign: Shock, cyanosis, shifting of apex and trachea to opposite site, hyper-resonant
percussion note and breath sound diminished or absent.

Investigations:

CXR P/A view.


ABG.
CT scan.

Treatment:

Immediate insertion of wide bore needle in the second intercostal space of affected site.
Then insertion of chest tube:
Kept for 24 hours and removed if lung is clinically and radiologically expanded.
If not- kept up to 1 week.
If still no response: Thoracotomy.
O2 inhalation.
IV fluid.
Antibiotics.
Analgesics.

Q. SN on Hemothorax.

Ans:

Definition: Accumulation of blood in pleural cavity.

Cause:

Trauma.
TB.
Tumor.
Pneumonia.
Iatrogenic.

Clinical features: Like pleural effusion.

Investigations: CXR, ABG, CBC with ESR.

Treatment:

General: ABC, antibiotics, analgesics, TT, wound management.


Specific:
No active bleeding: Aspiration.
Huge Hemothorax:
IT tube.
Thoracotomy:

183 SHARIFUL HALIM, K66, DMC


Sharifs personal notes

o Initial aspirate >1.5 L.


o Bleeding >200ml/h for 3 or more hour.

Q. What are the indications of thoracotomy?

Ans:

Initial aspirate >1.5 L.


Bleeding >200ml/h for 3 or more hour.
Rupture of bronchus, aorta, esophagus and diaphragm.
Cardiac temponade.

Q. What are the indications of chest drain tube?

Ans:

Pneumothorax.
Hemothorax.
Pyothorax.
Haemopneumothorax.
Massive Hydrothorax.
Hydropneumothorax.
Post-thoracotomy.

Q. What are the causes of SC emphysema/Surgical emphysema?

Ans:

Definition: Presence of air in tissues.

Causes:

Poorly managed chest tube drain.


Tracheostomy.
Rupture of esophagus.
Fracture of nasal bone.

Q. What is CABG? Write down indications of CABG? Name the conduit used for CABG?

Ans:

Definition:

It is a surgical procedure performed to relieve angina or other ACS by bypassing stenosed or


thrombosed coronary artery with vascular graft/conduit.

Indications:

Chronic stable angina.

184 SHARIFUL HALIM, K66, DMC


Sharifs personal notes

ACS and complications.


Combined with valve replacement.

Conduit:

Internal mammary artery.


Great saphenous vein.
Cephalic vein.
Radial artery.

NEUROSURGERY

Q. What are causes and cardinal features of raised ICP?

Ans:

Causes:

ICSOL: Brain tumor, brain abscess, IC hematoma, SD hematoma, lymphoma, tuberculoma.


Meningitis.
Encephalitis.
Hydrocephalus.
Severe HTN.

Cardinal features:

Cushing triad: Headache, vomiting, papilloedema.


Cushing reflex: Bradycardia, hypertension, Laboured/irregular respiration.
Other features: Blurred vision, Confusion, convulsion, coma.

Q. SN on ICSOL.

Ans:

Causes:

Brain tumor, brain abscess, IC hematoma, SD hematoma, lymphoma, tuberculoma.

CF: Raised ICP + Focal deficit.

Investigations:

CT scan.
MRI.
Screening for primary site.
Other investigations according to suspicion.

Q. Classify brain tumors.

Ans:

185 SHARIFUL HALIM, K66, DMC


Sharifs personal notes

Neuro-epithelial tumor: Astrocytoma, Oligodendroglioma.


Meningeal: Meningioma.
Nerve sheath: Acoustic neuroma.
Pituitary tumor.
Lymphoma.
Metastatic tumors.
Germ cell tumor: Teratoma.
Tumor like malformation: Craniopharyngioma.

Q. Write a SN on meningocele.

Ans:

Definition: It is a congenital defect in which meninges protrude from spine or skull.

Causes:

Spina bifida.
Defect in the development of skull.
Sacro-coccygeal teratoma.

CF:

Soft cyst covered by skin which is fluctuant, brilliantly transilluminant.

Investigations:

USG.
CT/MRI.

Treatment:

It is operated as soon as possible.

Q. Write a SN on hydrocephalus.

Ans:

Definition: Excessive accumulation of CSF in the ventricles due to increased production, defective
drainage or absorption and associated with raised ICP.

Causes:

Obstructive: Stenosis of cerebral aqueduct.


Communicative: CSF infection.

CF:

Children: Progressive macrocephaly, sunsetting eyes, bulging anterior frontanelle, dilated


scalp veins.
Adult: Raised ICP.

186 SHARIFUL HALIM, K66, DMC


Sharifs personal notes

Investigations:

X-ray skull: Copper beating of the skull.


CT scan.
MRI.
USG.

Treatment:

Medical CA inhibitor: Acetazolamide.


Surgical:
Removal of cause of obstruction.
VP shunt.
Ventriculo-atrial shunt.

GCS:

Q. Classify head injury according to GCS.

Ans:

According to severity:

Minor: 15 with no LOC.


Mild: 14-15 with LOC
Moderate: 9-13
Severe: 3-8.

Q. Classify intracranial hemorrhage.

Ans:

Epidural, subdural, Sub-Arachnoid, Intracerebral, intra-ventricular.

Q. Write a SN on extra-dural hematoma.

Ans:

Definition: Collection of blood between skull and dura matter.

Cause: Head injury.

Vessels responsible: Middle meningeal artery and vein.

Clinical features:

H/O head injury.


LOC and regain and then LOC: Lucid interval.
Features of Raised ICP.
Features of focal deficit: Commonly contralateral hemiparesis.
Features of shock.

187 SHARIFUL HALIM, K66, DMC


Sharifs personal notes

Investigations:

X-Ray skull.
CT scan of head.
S. electrolytes.

Treatment:

Immediate resuscitation: ABC.


Specific treatment:
Surgical: Evacuation of clot by Burr-hole operation.
Medical: Mannitol, steroid.
General:
Antibiotics, analgesics, antiemetic, anticonvulsant prophylaxis.
Care of the UC patient: Eye, mouth, bladder, bowel and skin.

Q. Write down the principle of management of head injury.

Ans:

Immediate resuscitation: ABCD.


Investigations: X-Ray skull, CT, MRI, RFTs.
Specific treatment:
Surgical: Evacuation of clot by Burr-hole operation in EDH, if fracture- management
of fracture.
Medical: Mannitol, steroid.
General:
Antibiotics, analgesics, antiemetic, anticonvulsant prophylaxis.
Care of the UC patient: Eye, mouth, bladder, bowel and skin.

Q. Write a short note on GCS.

Ans:

Definition: It is scale is used to measure the Level of consciousness.

Methods:

Eye opening:
Spontaneous: 4.
To speech: 3.
To pain: 2.
None: 1
Verbal response:
Oriented: 5.
Confused words: 4.
Inappropriate word: 3.
Incomprehensible sound: 2.

188 SHARIFUL HALIM, K66, DMC


Sharifs personal notes

None: 1.
Best motor response:
Obeys: 6.
Localizes: 5.
Withdraws: 4.
Abnormal flexion: 3.
Abnormal extension: 2.
None: 1.

Q. What is lucid interval?

Ans:

It is brief period of consciousness between 2 periods of unconsciousness.

Cause:

Head injury.
Dhatura poisoning.
Psychiatry.

189 SHARIFUL HALIM, K66, DMC

You might also like